Applied Three Final Module To Be Submitted
Applied Three Final Module To Be Submitted
Module Introduction
This module consists of five units. The first unit deals with ordinary differential equations. In
this unit we will look briefly at many terms and concepts related to ordinary differential
equations and discuss analytical methods of solving first and second order ordinary differential
equations. The second unit deals with Laplace transforms; Inverse Laplace Transform. The
second unit introduces and briefly discusses various methods of finding Laplace and Inverse
Laplace transforms and applications of Laplace Transform to solve homogenous and non-
homogenous differential equations and Integral equations will also be treated thoroughly in the
second unit. The third unit deals with Fourier series and Fourier integrals; and Fourier
transformations, which are the most useful type of mathematical tools in physical sciences, and
their applications. The fourth unit discusses on calculus of vector valued functions and vector
fields, specifically focuses on the limit, continuity and derivatives of vector functions and vector
fields and their applications. The fifth unit deals with calculus of functions of complex variables
together with their applications. By doing so students will be able to express terms and concepts
related to ordinary differential equations, Laplace Transform, Fourier series and Fourier
integrals; and Fourier transformations, calculus of vector functions, vector fields and functions
of complex variables
SOME FEATURES OF THE MODULE.
Quick Check Exercises: Each exercise set begins with approximately five practice exercises that
are designed to provide students with an immediate assessment of whether they have mastered key
ideas from the section.
Applicability : One of the good feature, primary goals of this module is to link calculus to the real
world and the student’s own experience. This theme is carried through in the examples and
exercises.
Career Preparation: This module is written at a mathematical level that will prepare students for
a wide variety of careers that require a sound mathematics background, including engineering, the
various sciences, and business.
Cooperative learning: One of the primary goals of this module is also to promote cooperative
learning, so that students share knowledge and skills through a lot of group discussions and
group activities given at each of new ideas introduced.
Module Objectives:
At the end of this module students will be able to:
Understand the concept of ordinary differential equations.
Define ordinary differential equations.
Correctly identify differential equations of various orders, degrees and type;
Compute general and particular solutions of linear differential equations
Demonstrate the application of differential equations together with solving exercises
Apply and Appreciate the concept of differential equations in solving real life
problems.
Define Laplace Transforms
Finding Laplace transforms of some functions using basic principles
Identify Different Properties of Differential Laplace Transforms
Apply Laplace transforms to solve differential equations modeling physical
problems
Distinguish Fourier series and Fourier integrals; Fourier transformation.
Demonstrate the applications of Fourier series, Fourier integrals, Fourier and Laplace
transformations together with solving exercise.
Apply the concept of Fourier series, Fourier integrals, Fourier and Laplace
transformations in solving real life problems
Define scalar and vector fields.
compute curl and divergence, and know what they have to say about the nature of a
vector field.
Evaluate line and surface integrals. Identify when a line integral is independent of
path and use the Fundamental Theorem of Line Integrals to solve applied problems
Find the curl and divergence of a vector field, the work done on an object moving in a
vector field, and the flux of a field through a surface. Use these ideas to solve applied
problems
Identify and use Green’s Theorem, the Divergence (Gauss’s) Theorem and Stokes’
Theorem.
UNIT I
Unit Introduction
This unit is divided into three sections. The first section presents Introduction to basic concepts
and ideas of differential equations. In the second section, we discuss various types of First-Order
differential equations that can be solved analytically. Particularly, the second section examines
Separable and non-separable differential equations, Exact and Inexact differential equations
(integrating factors), Linear and Some Special Non-linear Differential Equations of the first
order. Linear second order ordinary differential equations. The third section introduces linear
second order ordinary differential equations. In the third section second order homogeneous and
non-homogeneous ordinary differential equations are discussed and their solutions obtained with
a variety of techniques. Euler-Cauchy equation and systems of differential equations are also
parts of the third section. The learning activities in this unit include self-study, reading
assignments, group discussions, and problem solving.
Unit Objectives: Specific learning Objectives
By the end of this unit, the learner should be able to :
Define Differential Equations
Correctly identify differential equations of various orders, degrees and type;
Form a differential equation by elimination of arbitrary constants;
Mention Different Physical Examples of Differential equations
Demonstrate how to differentiate separable and non-separable, exact and non
exact, linear and nonlinear first order differential equations together with solving
exercises.
Apply the concepts of ordinary differential equations in real life problems.
Identify homogeneous and non-homogeneous second order differential equations.
Demonstrate how to compute general and particular solutions of second order
differential equations together with solving exercises.
Demonstrate how to transform systems of differential equations to second order
differential equations together with solving exercises.
Examples:
2u 2u 2u 2u u u u y
0, 2 2 and 2x z
x 2
y 2
x 2
t t y x x
are partial differential equations since the functions u f ( x, y) , u f ( x, t ) , y f ( x, z ) in each
equations depends on two variables.
Typical Examples of differential equations in applications are the following:
dR
i. The decay of a radioactive substance is described by an equation of the form kR ,
dt
where k is a constant.
d 2x dx
ii.The differential equation m F t , x, describes Newton's law for position xt of a
dt
2
dt
dx
particle acted on by a force F of t, x and ,the velocity.
dt
d 2x
m 2 k1 k 2 x F t where F t is a given function and
dx
iii. The differential equation
dt dt
m, k1 , k 2 are constants, governs the motion of a vibrating mass at the end of a vertical spring.
2u 2u
iv. c 2
( governs the motion of wave Equation in one dimension)
x 2 t 2
2u u
v. a 2
(governs the motion of Heat Equation in one dimension)
x 2 t
2u 2u
vi. 0 ( Laplace Equation in two dimension)
x 2 y 2
d 2q
q sin t models the charge q qt on a capacitor in an electrical circuit
dq 1
vii. L 2
R
dt dt C
containing an inductor, resistor and a capacitor, where the current is driven by a sinusoidal
electromotive force operating at the frequency .
Differential equations may be classified further according to their order and degree.
M ( x, y)dx N ( x, y)dy 0 .
For example, x y 2 3 y dx x 2 3x y dy 0 is a first order and first degree ode.
Quick Check Group Activity 1.1.1: Using the notes given in section 1.1.1, discuss in a small
group of 3-4 members, order and degree of the following differential equations.
2 3
dy d 3 y d2y dy
2 3x 1
2
a) 2 5 c)
dx dx 3 dx
dx
2 5
1 1 d3y d2y
d y
2 2
dy 3 d) 2 2 y e x
b) 2 x dx dx
dx dx
Instructor's role
Check and give feedback to their answers
All the differential equations to be discussed in the next two sections are ordinary differential
equations.
Notation: Throughout this module ordinary derivatives will be written by using either the
dy d 2 y d 3 y
Leibniz notation , 2 , 3 ,... or in the prime notation y, y ' , y '' ,... .
dx dx dx
In symbols we can express an nth-order ordinary differential equation in one dependent variable
by the general form: F x, y, y ' , y '' , y ''' ,..., y n 0 or in Leibnitz notation as
dy d 2 y d 3 y dny
F ( x, y, , 2 , 3 ,..., n ) 0 (*)
dx dx dx dx
where F is a real-valued function of n 2 variables: x, y, y ' , y '' ,..., y n .
On the assumption that it is possible to solve an ordinary differential equation in the form (*)
uniquely for the highest derivative y n in terms of the remaining n 1 variables, we can write (*)
as:
dny
dx n
f x, y, y ' , y '' , y ''' ,..., y n1 (**)
The differential equation (**) where f is a real-valued continuous function, is referred to as the
normal form of (*). Thus the normal forms to represent general first- and second-order ordinary
differential equations respectively are:
dy
dx
f x, y and
d2y
dx 2
f x, y , y '
For example, the normal form of the first-order equation 4 xy ' y x is y '
x y ; the normal
4x
form of the second-order equation y '' y'6 y 0 is y '' y'6 y .
Classification by Linearity:
Definition1.1.3: An nth-order ordinary differential equation F x, y ' , y '' , y ''' ,..., y n 0 is said to
be linear if F is linear in y, y ' , y '' ,..., y n . This means that an nth-order ODE is linear when it can
In the additive combination on the left-hand side of the above equation we see that the
characteristic properties of a linear ODE are as follows:
The dependent variable y and all its derivatives y, y ' , y '' ,..., y n are of the first degree, that
is, the power of each term involving y is 1.
The coefficients a0 , a1 , a2 ,..., an of y, y ' , y '' ,..., y n depend at most on the independent
variable x.
A nonlinear ordinary differential equation is simply one that is not linear. Nonlinear functions of
1
the dependent variable or its derivatives, such as sin y , e y , and so on or products of the
y
dependent variable or its derivatives , cannot appear in a linear equation.
are examples of nonlinear first-, second-, and fourth-order ordinary differential equations,
respectively.
Two important special cases of linear ODEs are linear first-order (n =1) and linear second order
(n= 2) DEs:
d2y
a1 x a0 x y g x and a2 x a1 x a0 x y g x
dy dy
2
dx dx dx
An n th order ordinary differential equation may have more than one solution which can be
represented by a single formula involving an arbitrary constants such a solution is called a
general solution.
Example: By integrating both sides of the first order equation y cos x we obtain
equation on the interval. A solution obtained by assigning definite values to the arbitrary
constants in the general solution is called a particular solution.
Although the prime problem in the study of differential equations is finding the solution to a
given differential equation, the converse problem is also interesting.That is the problem of
finding a differential equation which satisfies by a given group of functions, each one of which is
a solution of the ODE. The problem is solved by repeated differentiation and elimination of the
arbitrary constants.
A general solution is therefore a family of solutions defined on some interval I that contains all
solutions of the DE that are defined on I. A solution that is not a member of a family of solutions
of the equation that is, a solution that cannot be obtained by specializing any of the parameters in
the family of solutions is called a singular solution.
Solution Curve:
The graph of a solution of a differential equation is called an integral curve or a solution curve
for the equation, so the general solution y f ( x, c1 , c2 ,....cn ) of a differential equation produces
a family of integral curves corresponding to the different possible choices for the arbitrary
constants. Since y f x is a differentiable function, it is continuous on its interval I of
definition. Thus there may be a difference between the graph of the function and the graph of the
solution . Put another way, the domain of the function y f x need not be the same as the
interval I of definition (or domain) of the solution .
d 3 y dy
2
dx 3 dx
u 2 u u
3 4 2
t dx dy
2
d 3s d 2s
4 3 2 s 3t
dt dt
3
d y dy 2
2 2
5 1
dx 2 dx
Assessment
Check students participation in the group activity.
Give feedback to their answers
F x, y or
dy
dx
M ( x, y)dx N ( x, y)dy 0
where F x, y , M x, y and N x, y are in general functions of both x and y.
Definition: (Initial value problem)
A first order differential equation together with an initial condition, y( x0 ) y0 , is called the
initial value problem.
For example, y 2 xy x 1 0, y(1) 3 is an initial value problem.
Theorem 1.2.1: Existence of a unique solution ( Picards theorem)
For a differential equation of first order, viz.
f x, y
dy
dx
which satisfies the initial condition
y( x0 ) y0
there exists a unique solution y x , defined for all values of x in a certain region including
x0 , satisfying the given differential equation and taking on the value y0 when x x0 ,provided
f
that f x, y and are continuous single valued functions of in the region
y
x x0 , y y0
where 0 and 0 .
We shall, in the remainder of this section, discuss some analytical methods of solving various
types of First Order and First-Degree differential equations. However, despite the apparent
Kassahun Nigatu(MSC),Mesfin Teshome(MSC) and Yitagesu Daba(MSC) 12
Applied Mathematics III
simplicity of this equation, analytic solutions are usually possible only when F ( x, y) has simple
forms. Which of the two above forms is the more useful for finding a solution depends on the
type of equation being considered.
1.2.1 Separable Differential Equations; Reduction to Separable Form
Definition: A differential equation which can be written in the either form:
f ( x) g y Basic form)
dy
(1.2.1a)
dx
Since the variables x and y are now separable , we have ,from (1.2.1a),
dy
g ( y) f ( x)dx , (1.2.1c).
dy
dx
1 y2
Integrating both sides, we obtain
arctan y x c y tan(x c) .
b) xy y 0, y(1) 1
xy y
dy
x y
dx
dy dx
.
y x
Integrating both sides we obtain
ln y ln x c .
From this we obtain the general solution
c
y .
x
1
By using the initial condition, y(1) 1, we see that c 1 and hence y ( x) .
x
c) x 2 1 y 1 x y 2 0
dy
dx
This can be put into the form
dy y 2 1 x
dx 1 y x 2
1 y 1 x
dy 2 dx
2
y x
where the variables are separated.
Integrating both sides we get
1 1 1 1
y 2
dy 2 dx
y x x
which gives
1 1
ln y ln x ln c ,
y x
where c is an arbitrary constant. Simplification gives
y 1 1
ln
cx x y
which is the general solution of the differential equation.
d) x 1dy y 1dx 0 , y0 3 .
Separating the variables we obtain
dx dy
x 1 y 1
Integrating both sides we get
ln x 1 ln y 1 ln c,
or x 1 y 1 c
Substituting x 0, y 3 in the last equation we obtain c 2.
b) y e y x 2 , y0 ln 2 d) y x 1(1 y 2 )
Teachers role
Check and give feedback to their answers
Group Activity: 1.2.1
1. Solve the following differential equations.
a) y e x y 3 e) y 2 xy x 1 0, y(1) 3
b) x 3 1 dy ydx 0 f) ( x ln x) y y
c) yy sin 2 x, y(0) 3 x
h) y , y(2) 5
y
7
d) y y tanh x. y (0)
i) ( x 1) y' y 1 0, y(0) 1
5 2 2
dy dy
j) e x y x 2 e y k) e 2 x y x 3e y
dx dx
dI
g. L RI 0, I (0) I 0 , where L and R are cons tan ts
dt
2. (Newton’s Law of Cooling). A copper ball is heated to a temperature of 1000 c . Then at a
time t 0 it is placed in water which is maintained at a temperature of 300 c . At the end of 3
minutes the temperature of the ball is reduced to 700 c . Find the time at which the temperature of
the ball is reduced to 310 c . (Hint: the rate of change of the temperature T of the ball is
proportional to the difference between T and the temperature of the surrounding medium.)
dr
3. The stress P in thick cylinders of radius r is given by r 2 p 2c. Find P in terms of r.
dp
Assessment
Asking an answer for some of the questions.
Check students participation in the group activity.
Give feedback to their answers
y f (ax by c) , (*)
z a
f (z ) .
b
This can be separated as
dz
dx .
a bf ( z )
Examples: Solve the following differential equations
a) y ( x y 1) 2 2 b) y 1 x y 2 , y(0) 2
cot y x 1
dy
c)
dx
Solution:
a) Letting
z x y 1 ,
we have
y z 1 .
Substituting this in the given equation, we get
z 1 z 2 2 .
This implies
dz
z 2
1
dx ,
z 1
ln 2x c
z 1
z 1
ce 2 x .
z 1
Hence
1 ce 2 x
z .
1 ce 2 x
But since
z x y 1 ,
then
1 ce 2 x
y z x 1 x 1
1 ce 2 x
1 ce 2 x
y ( x) x 1 .
1 ce 2 x
b) y 1 x y 2, y(0) 2
Let
zx y2 (*)
y z x2
y z 1 .
Substituting this in the given equation, we get
z z.
This implies
dz
z
dx ,
2 z xc .
Substituting this in (*) we get
xc
x y2 ,
2
xc
2
x y2 .
2
Thus
xc
2
y ( x) x 2.
2
But y(0) 2 implies c 4
Thus
x 4
2
y ( x) x2.
2
cot y x 1 .
dy
c) Solve the differential equation
dx
Putting y x v, we obtain by differentiation of both sides we get
dy dv dy dv
1 or 1
dx dx dx dx
The given differential equation now becomes
dv dv
1 cot v 1 or cot v
dx dx
in which variables are separable, and we obtain
dv sin v
dx dv.
cot v cos v
Integration now gives
x ln cos v ln c ln
c c
ln ,
cos v cosx y
which is the required solution.
Quick Check Class Activity 1.2.2: Solve the following differential equations
cosx y 1
2
dy dy x y 1
a) c) .
dx dx x y 3
sin 2 x y 1
dy
b) dy
dx d) 4x 2 y 1
dx
Instructor's role
Check and give feedback to their answers
f tx, yt t n f x, y
that is, if x and y be replaced by tx and ty , t n factors out of the resulting function, and the
remaining factor is the original function. For example,
x x y 1
x 2 2 y 2 , sin , ln , x y xy ,
y x x y
are homogeneous of degrees 2, 0, 0, 1 and -1, respectively, since
tx 2 ty 2 t 2 x 2 2y 2
tx x
sin sin
ty y
tx ty x y
ln ln
tx y
yx ty txty t ( x y xy )
1 1
t 1
tx ty x y
An equation of the form
f x, y
dy
dx
is said to be homogeneous whenever the function f is homogeneous of degree zero and so is a
v ' x v g (v) ,
dv
thus x g (v ) v
dx
This can be separated as
dv dx
.
g (v ) v x
y
If we integrate this and replace v by , we obtain the general solution of (*) .
x
Examples:
a) Solve the differential equation 2 xyy y 2 x 2 0 .
Solution: Since
2 xyy y 2 x 2 0
can be re written as
2 xyy y 2 x 2
y x
2 y . (*)
x y
y
Now let v .
x
y vx
y v ' x v .
Then from ( * ) it follows that
1
2(v ' x v) v
v
2dv v2 1
x
dx v
2v dv dx
v 1
2
x
Integrating both sides yields
ln | v 2 1 | ln | x | c
c
v2 1
x
y
Now, setting v we obtain
x
y 2 x 2 cx .
Kassahun Nigatu(MSC), Mesfin Teshome(MSC) and Yitagesu Daba(MSC) 21
Applied Mathematics III
dy x 2 xy y 2
b) Solve the differential equation .
dx x2
1 v v
dv
vx
2
dx
1 v
dv 2
or
dx
On separating the variables, this gives
dx dv
x 1 v2
Integrating both sides, we obtain
y
ln x tan v c tan c
x
y
tan
Hence x c1e x
which is the general solution of the differential equation.
dy 2 xy
c) Solve the differential equation 2 and find a solution subject to the condition that
dx x y 2
y 1 when x 1.
Solution: The equation can be written as
y
2
dy x
2
dx y
1
x
which is homogeneous type.
dy dv
Putting y vx and v x the above differential equation becomes
dx dx
dv 2v
vx
dx 1 v 2
dv 2v v v3
x v
dx 1 v 2 1 v2
ln x ln v ln 1 v 2 ln c
y
cv cxy
Hence x c x 2 2
1 v 2
y x y2
1 2
x
or x 2 y 2 cy
Substituting x 1 and y 1 in the above, we obtain c 2 . Hence the particular solution is
x2 y2 2 y .
Quick Check Exercises: Solve the following differential equations.
a) ( x 2 y 2 )dx 2 xydy 0 dy y 2
c) x y
dx x
dy y 3 3x 2 y
b) dy y y
dx x 3 3xy 2 d) sin
dx x x
Instructor's role
Check and give feedback to their answers
dy aX bY ah bk c
dx a' X b'Y a' h b' k c'
Y
a b
dy aX bY X
dx a' X b'Y Y
a' b'
X
which is homogenous equation.
After solving this equation we replace X x h;Y y h, to get the solution of the given
equation.
a b a b
When '
' ( a case of failure) we put ' ' m ,(say),
a b a b
dy ax by c
Then becomes
dx a' x b' y c'
dy ax by c
dx m(ax by ) c'
Now by putting ax by v , the equation can be brought to variable separable form.
Examples:
dy x 2 y 3
a) Solve
dx 2 x y 3
a b 1 2
Solution: We have '
' i.e.,
a b 2 1
Put x X h; y Y h,
dy X 2Y h 2k 3
dx 2 X Y 2h k 3
dy X 2Y
dx 2 X Y
dY dV
which is homogeneous and may be solve by substitution Y VX and V X . Then
dX dX
we have
dV 1 2V
V X
dX 2 V
dV 1 2V 1V 2
X V
dX 2 V 2 V
2 V dV
dX
1V 2
X
Integrating yield
2 V dX
1V 2
dV
1V 2
dV
X
C
1V 1
ln
ln 1 V ln CX
2
1V 2
Substituting x X 1, y Y 1and simplifying the required general solution is
x y 2 K x y 3 .
dy x y 1
b) Solve
dx 2 x 2 y 1
Solution:
a b 1 1
Now we have '
' m m . The equation may be written as
a b 2 2
dy ( x y) 1
dx 2( x y ) 1
dv dy dy dv
Now by putting x y v and 1 1 , the equation can be brought to variable
dx dx dx dx
separable form. Use the method of separation of variables to solve (Class Exercise!!!)
Quick Check Exercises: Solve the following differential equations.
dy x y 2 dy 2 y x 5
a) b)
dx y x 4 dx 2 x y 4
Instructor's role
Check and give feedback to their answers
a) y
y 2 x 3 cos x 2
x
y
, y 0.
h) y sin 2 ( x y) 0
y2 2 x y
i) y
y x2
b) xy y x sec
x
j) y ( x y 2) 2
c) ( x3 3xy 2 )dx ( y 3 3x 2 y)dy 0
dy y y
k. tan
d) xyy 2 y 4 x , y(2) 4
2 2
dx x x
y l. ydx 2 y x dx 0
e) xy y x sec , y (1) .
2
x m. x y dx y x dy 0
yx n. ( x 2 y 2 )dx ( xy x 2 )dy
f) y , y (1) 1.
yx
dy x3
x y o. x y 2 , y (1) 2
g) y dx y
x
x
p. (ln x ln y 1)dy dx, y(1) e
y
x y
2. Find the curve that passes through the point ( e , e ) and has the slope
y x
1 2 y 4x
3. Solve y . (Hint: use v y 2 x ).
1 y 2x
Assessment
Asking an answer for some of the questions.
Check students participation in the group activity.
Give feedback to their answers
Definition:
A first order differential equation of the form
M ( x, y)dx N ( x, y)dy 0
is said to be exact if there exists a function, denote by U ( x, y) , such that
U U
dU dx M ( x, y)dx N ( x, y)dy .
x y
That is, if the first member of the equation is the exact differential of a function of x and y. The
differential equation then takes the form
dU 0
and its solutions are defined implicitly by
U x, y C
where C is an arbitrary constant.
U
Note: i) M U ( x, y) M x, y dx k y
x
U
ii) N U ( x, y) N x, y dy g x
y
iii) Every Separable equation is exact.
Theorem: The necessary and sufficient condition for the differential equation
M ( x, y)dx N ( x, y)dy 0 (*)
to be exact is
M N
.
y x
Proof: Exercise.
Procedure of Solution:
Kassahun Nigatu(MSC), Mesfin Teshome(MSC) and Yitagesu Daba(MSC) 27
Applied Mathematics III
M N
Step 1:Check whether differential equation written in the form (*) satisfies or not.
y x
M N
Step 2: If for given equation is satisfied then there exists a function U ( x, y) for
y x
which
U
M
x
Integrating with respect to x, while holding y constant, we get
U
M U ( x, y) M x, y dx k y (**)
x
Step 4: Integrate (***) with respect to y and substitute this value in (**) to obtain
U x, y C , the solution of the given equation.
Remark
a) We could just start the above mentioned procedure with the assumption that
U x, y
N x, y and by integrating N(x,y) with respect to y and differentiating the resultant
y
expression, we would find the analogues of (**) and (***) to be, respectively,
U
N U ( x, y) N x, y dy g x and
y
g ' x M x, y N x, y dy
x
Examples: Determine whether the following differential equations are exact. If they are exact
solve them by the procedure given in this section.
M N
3x 2 e y and 3x 2 e y
y x
Thus
M N
y x
which implies the differential equation is exact.
Apply procedure of solution above
Let
U
3x 2 e y
x
Integrating with respect to x, we obtain
U x, y x3 y xe y g ( y)
where g(y) is a constant of integration
U
x 3 y xe y g ' ( y)
y
This gives
U
N x, y x 3 y xe y g ' ( y)
y
or g ' y 2 y g y y 2
U x, y x3 y xe y y 2 c .
M ( x, y) 2 x sin 3 y , N ( x, y) 3x 2 cos 3 y 2 y
and
M N
6 x cos 3 y , 6 x cos 3 y
x x
we get
M N
y x
which implies the differential equation is exact.
But since
k ( y) 2 y k ( y) y 2 c1 .
u( x, y) x 2 sin 3 y y 2 c1 c2 .
x 2 sin 3 y y 2 c ,
This implies
x 2 sin 3 y y 2 c
where c c2 c1 is a constant.
1 x
Example 3: Solve the differential equation dx 2 dy 0 .
y y
x
Solution: Now M x, y and N x, y 2
1
y y
M 1 N 1
2 and 2
y y x y
M N
y x
which implies the differential equation is exact.
But since
U
M U ( x, y) M x, y dx k y g y
x
x y
U x, y
x
g ( y)
y
where g(y) is a constant of integration
U x, y
x
k c
y
x
Thus, C is the solution of the given differential equation.
y
Quick Check Class Exercises: Determine whether the following differential equations are
exact. If they are exact solve them by the procedure given in this section.
a) x2 x 2 y 2 x 2 2 y 2 y' 0 b) 2 xy dx x 2 y 2 dy 0
c) e y dx 2 y xe y dy 0
Instructor's role
Check and give feedback to their answers
M N
y x
are known as inexact equations.
Inexact equations can be made exact by multiplying them by some function F ( x, y) 0 . Thus
F x, y M ( x, y)dx F x, y N ( x, y)dy 0 become an exact equation. Such a multiplier (or a
function) is called an integrating factor. Although an equation of the form
M ( x, y)dx N ( x, y)dy 0 has integrating factors, but there is no general method of finding them.
We now use the following Theorems (rules) for finding integrating factors
Theorem 1.2.3.1:
1 M N
When is a constant, say k , or a function of x
N y x
e e
kdx f ( x ) dx
alone, say f (x) , then an integrating factor, F (x) , will be or .
Proof. Exercise.
M ( x, y) 4 x 3 y 2 , N ( x, y) 2 xy
we obtain
M N
6y , 2y
y x
M N
.
y x
Thus the differential equation is not exact.
But since
1 M N 1
by 2 y 2 ,
N y x 2 xy x
an integrating factor is
dx
2
F ( x) e x
x2 .
Thus
x 2 (4 x 3 y 2 )dx 2 xydy 0
(4 x 3 3x 2 y 2 ) dx 2 x 3 ydy 0
is an exact differential equation, since
y
4 x 3 3x 2 y 2 6 x 2 y
x
2x3 y .
Now then, since
u( x, y) (4 x 3 3x 2 y 2 )dx k ( y)
x 4 x 3 y 2 k ( y) (*)
u
and 2x3 y
y
we have
2 x 3 y k ( y) 2 x 3 y .
This implies
k ( y) 0 k ( y) c1 ,
u( x, y) x 4 x 3 y 2 c1 c2
x 4 x 3 y 2 c,
where c c2 c1 is a constant.
Therefore,
x 4 x3 y 2 c ,
is a required solution.
Example 2: Solve 2 y 2 3x dx 2 xydy 0
we get
U x, y x 2 y 2 x 3 g y ,
g ' y 0 and g y c .
Theorem 2:
1 N M
When is a constant, say k , or a function of y alone, say f ( y) , then an
M x y
e e
kdy f ( y ) dy
integrating factor, F ( y) , will be or .
Proof: Exercise
Solution: Since
M ( x, y) 2 xy , N ( x, y) 3x 2 ,
M N
we have 2 x, 6x .
y x
This implies
M N
y x
Thus the differential equation is not exact.
But since
1 N N 1
6 x 2 x 2 ,
M x x 2 xy y
an integrating factor is
dy
2
F ( y) e y
y2 .
Thus
y 2 2 xydx 3x 2 dy 0
2 xy 3 dx 3x 2 y 2 dy 0
is an exact differential equation, since
2 xy 3 6 xy 2 3x 2 y 2 . .
y x
u( x, y) (2 xy 3 )dx k ( y)
x 2 y 3 k ( y) ( *)
and
u
3x 2 y 2 ,
y
we obtain
3x 2 y 2 k ( y) 3x 2 y 2 .
k ( y) 0 k ( y) c1 ,
where c1 is a constant. Thus from ( * ) it follows that
u( x, y) x 2 y 3 c1 c2
x2 y3 c ,
M y Nx
leads us nowhere, as is a function of both x and y. However,
N
Nx M y 4x x 3
is a function of y only. Hence
M xy y
dy
3
e 3 ln y e ln y y 3 is an integrating factor.
3
y
e
2
3. Verify that y, xy 3 and x 2 y 5 are integrating factors of ydx 2 xdy 0 and solve.
Assessment
Asking an answer for some of the questions.
Check students participation in the group activity and Give feedback to their answers
1.2.4 Linear First Order Differential Equations; Reduction to Linear Form
1.2.4.1 Linear First Order Differential Equations
Definition:
A first order differential equation is said to be linear if it can be written in the form
y f ( x ) y r ( x) ,
Theorem: The general solution of the linear first order differential equation:
y f ( x) y r ( x) ,
y f ( x) y r ( x ) ( *)
in the form
f ( x) y r ( x)dx dy 0 ( **)
and, let
M ( x, y) f ( x) y( x) r ( x), N ( x, y) 1 ,
M N
then f ( x), 0.
y x
M N
y x
F ( x) e
f ( x ) dx
.
e h ( x ) ( y f ( x) y ) e h ( x ) r ( x) .
Since h( x) f ( x) , this may be writer as
d
dx
ye h e h r .
y( x)e h ( x ) e h ( x ) r ( x)dx c ,
that is,
y ( x) e h ( x ) e h( x)
r ( x)dx c , h( x) f ( x)dx . Or
f ( x ) dx f ( x ) dx r ( x)dx c
y ( x) e e .
Definition 2.3.2: I ( x) e
f ( x ) dx
is called the integrating factor of the standard form of a linear
differential equation .
Procedure of Solution :
Step 1:Put the equation in the standard form y f ( x) y r ( x) if it is not given in this form.
x x 4
dx dx
y ( x) e e dx c
x
ln x 1
ln x
e 4 e dx c
x
1
x
4 dx c
c
4 .
x
Therefore,
c
y ( x) 4
x
is the general solution.
3x 1 y e 3 x
dy
Example 2.: Find the general solution of : x
dx
Solution: Standard form is
dy 1 e 3 x
3 y
dx x x
1 e 3 x
f ( x) 3 r ( x)
x, x
1
3 dx
Integrating factor = (x) e
P(x)dx
= e x xe 3x
e 3 x
y.xe 3 x xe 3 x dx c x c
x
c
or y ( x) e 3 x 1 for 0<x<.
x
Quick Check Class Exercises 1.2.7
1. Solve the following linear differential equations given below
dy dy
a) 2 y 8e x c) x y1
dx dx x
dy
b) x3 3x 2 y cos x
dx
2. Solve the following initial value problem
dy dy
a) 5 y 20, y (0) 2 b) x y ex, y(1) 2
dx dx
Instructor's role
Check and give feedback to their answers
c) y 3 y e 2 x 6 g)
dy
y sinh x
dx
xy 2 y 2e x
2
d) dr
h) r sec θ cos θ
dθ
y
e) y 2 x 2e x dy
x i) ( y tan x cos x) 0
dx
dy 3x 2 y
f)
dx 1 x 3
c) xy (1 x) y , y(2) 6e 2
d) xy 4 y 8x 4 , y(1) 2
2
e) xy y sin x, y
2
3. Find the curve having the given slope y and passing through the
given point ( x0 , y0 ) if
x 1 x
a) y , (0,2) b) y , (1, 0)
4 4y 1 y
Assessment
Asking an answer for some of the questions.
Check students participation in the group activity.
Give feedback to their answers
Definition:
The differential equation of the form
Kassahun Nigatu(MSC),Mesfin Teshome(MSC) and Yitagesu Daba(MSC) 41
y p ( x) y g ( x) y a ,
Remark: For a 0 and a 1 the equation is linear, and otherwise it is non linear.
For a 0,1 if we set
u ( x) y ( x) 1 a
the equation will be reduced to the linear form
u (1 a) p( x) u (1 a) g ( x) ,
which will be easily solved.
Example 1: Solve the Bernoulli equation
y y y 2 .
Solution: Since a 2 , by setting
u y1 2 y 1 , (*)
we obtain
u y 2 y .
Thus
( y y y 2 ) y 2 .
From this it follows that
yy 2 y 1 1 ,
or
yy 2 y 1 1 ,
by multiplying both sides by 1 .
u u 1 .
With solution
u ( x) e e dx c
dx dx
1 ce x .
Using (*) we obtain
1 1
y ( x) .
u ( x) 1 ce x
This is the required general solution.
dy 1
Example 2: Solve the bernouli equations. y 3y3
dx x
Solution: Let v y1n y 2 (n=3)
dv dy
2 y 3
dx dx
dy 1 1 dv
or . 3
dx y 2 dx
Substituting these values into the given differential equation, we get
1 dv 1
v3
2 dx x
dv 2
or v 6
dx x
This equation is of the standard form, (2.7) and so the method of Section 2.3 is applicable.
Integrating factor ( x) e
P ( x ) dx
2
where P( x) . Therefore I x x 2
x
Solution is given by
v.x 2 6 x 2 dx c
v.x2 6x1 c
Since v y 2 we get y 2 6 x cx 2 or y 1
6xcx2
Quick Check Group Activity 1.4.2 Solve the following Bernoulli equations
1
a) y y xy 2 c) 3 y y (1 2 x) y 4
x
b) y xy xy 1 d) y x 3 y 2 xy
Instructor's role
Check and give feedback to their answers
1.2.5 Application Involving First Order Differential Equations
First order differential equations are often used to model rate processes. For example,
1.2.5.1.(Newton's law of Cooling).
This law states that the rate of temperature of a body is proportional to the difference between
the temperature of the body and that of the surrounding medium.
Suppose a body whose temperature is initially 𝜃1 ℃ is allowed to cool in air which is maintained
at a constant temperature of 𝜃2 ℃. It is required to find the temperature of the body as a function
of time 𝑡.
Let the temperature of the body be T C at time t. Then by Newton's law, we can write
k T 2
dT
(*)
dt
where 𝑘 is the constant of proportionality. The initial condition is given by T 1 at 𝑡 = 0.
Solving the equation, we obtain
kt logT 2 log C (**)
Initially we have
T 0 1
T 2 1 2 e kt ,
k T 40
dT
(i)
dt
Integration gives;
kt log( T 60
40
) (iii)
1
k log 3
4
Hence equation (iii) gives;
t
log 3 log( T 60
40
)
4
When T=50, we obtain
4 log 6
t 6.5 minutes
log 3
Example 2 Assuming that a building is a homogeneous point source without wind in its
homogeneous fluid surroundings of constant temperature. The internal temperature change of a
building can be calculated using Newton’s Law of Cooling.
The simplest form is the first order differential equation
dT
k (T Ts)
dt
where T(t) is the internal temperature of the building, S is the temperature of the surroundings,
and k is a growth constant. If we assume the initial condition T(t0)=T0, after integration and
substitution, we arrive at
T (t ) TS (T0 TS )e kt (1)
Therefore,
T ( t f ) TS k (t f t 0 )
T0 TS
e , (2)
implying
T ( t f ) TS
k (t f t0 ) ln( T0 TS
). (3)
Thus making it possible to find k with a second condition, 𝑇(𝑡𝑓) = 𝑇𝑧. Through substitution of
this second condition in (3) and after rearranging, we have
Now we can use the constant k to find the temperature at any given time, by inserting it into (2).
For example, a building with a constant surrounding temperature of 85°F has an initial condition,
T(0)= 75°F. Inserting this information into 2-4, we get
T ( t f ) 85 k ( t f 0 )
T (t ) 85 (75 85)e kt , 7585 e
T ( t f ) 85
k (t f 0) ln( 7585 )
k ( 15 ) ln( 32 )
k .08109
Then by inserting the k value into (1),
T (t ) 85 (75 85)e.08109t
1.2.5.2.(Radioactive decay)
Radioactive decay where the content of radioactive nuclei is denoted by the symbol, n, is
modeled by the following first-order differential equation.
dn
kn
dt
dn
For a positive constant, k, this equation tells us that the rate is negative and proportional to
dt
the amount of radioactive nuclei, n, present. If the initial content at t = 0 is n0 we can multiply
this equation by dt/n to obtain the following form that can be integrated directly.
dn
n
dn
t
n
n n 0 kdt ln n0 kt n n0e
kt
kdt k
n 0
The half-life for radioactive decay, t1/2, is defined as the time required for the initial radioactivity,
n0, to decrease to half its original value. Equation shows n n0 e kt us how to compute this half
life.
n0
1 ln( 2)
ln 2 ln ln( 2) kt1 / 2 t1 / 2
n0 2 k
With equation , we can rewrite the final version of equation to introduce the half life.
t
n n0 e t1 / 2 ln(2 )
Though only two applications involving first order equations are discussed in this module there
various laws governed by first order equations. For instance: Kirchhoff's electromotive force law
governing flow of electricity, Hooke's law which relates the tension and extension in elastic
string, the motion for a body falling in a Resisting medium are all governed by First order
equations.
Definition: The general linear second order differential equation is of the form
R( x) y" P( x) y'Q( x) y S ( x) ,
where P, Q, R, S are continuous functions. On any interval where Rx 0 ,we can divide the
equation by Rx and obtain the special linear differential equation
y f ( x) y g ( x) y r ( x) , (1 )
where f , g and r are continuous functions.
y f ( x) y g ( x) y 0 ( 2)
and is said to be homogeneous differential equation.
If r ( x) 0, then equation (1) is said to be non-homogeneous differential equation.
For example, y 4 y e x sin x is a non homogeneous linear differential equation, where as
(1 x 2 ) y 2 xy 6 y 0 and y 4 y 0 are homogeneous linear differential equations
Any differential equation of the second order which cannot be written in the form ( 1 ) is said to
be nonlinear.
Theorem 2: (Existence and Uniqueness Theorem for Linear Initial Value Problems)
Let f , g and r be continuous functions on some open interval I, and let k 0 , k1 be two
given constants. Then there exists a unique function y (x) that satisfies the linear differential
equation
y f ( x) y g ( x) y r ( x) (3)
on I and the two initial conditions
y( x0 ) k0 , y( x0 ) k1 (4)
Remark1: The conditions given in (4) both involve evaluations of the unknown function y
and its derivatives at the same point x0 . This is a crucial requirement for the existence and
uniqueness of a solution.
There is another type of problem involving the differential equations (3) in which conditions at
more than one point are given. For example, we might specify y( x1 ) d1 and y( x2 ) d 2 .
Conditions of this sort are called boundary conditions, and a differential equation together with a
set of boundary conditions is called a boundary value problem.
It is important to note that the existence and uniqueness of a solution guaranteed by Theorem1
for initial value problems, doesn’t hold for boundary value problems.
Example: Observe that the boundary value problem
y y 0, y(0) y( ) 0 ,
has infinitely many solutions:
y c sin x , for any constant c .
To check, note that sin 0 sin 0, so the boundary conditions are satisfied, and
(c sin x) c sin x c sin x c sin x 0 .
Where as the boundary value problem
y( x0 ) k1 , y( x0 ) k2 .
y c1 y1 c2 y2
and its derivatives,
y c1 y1 c2 y2 , y c2 y2 c2 y2
into ( 2 ) , we get
Remark: The above theorem doesn’t hold for non-homogeneous linear differential
equations or for nonlinear equations.
For example, substitution shows that:
1. The functions y1 1 cos x and y2 1 sin x are solutions of the non-homogeneous
linear differential equation y y 1, but 2(1 cos x), and (1 cos x) (1 sin x), are not
solutions of this differential equation.
2. y1 x 2 and y2 1 are solutions of the nonlinear differential equation. yy xy 0 , but
The point to taking linear combinations c1 y1 c2 y2 is to obtain more solutions from just two
just another constant multiple of y1 . In this event, y 2 is superfluous, providing nothing we did
not know from. This leads us to distinguish the case in which one solution is a constant multiple
of another from the case in which the two solutions are not multiples of each other.
Definition: Linear Dependence and Independence
Two functions, y1 x and y2 x , are linearly independent on an interval x0 , x1 ,whenever the
y1 x y2 x 0 or y1 x y2 x Cy2 x
c2 c2
c1 c1
Therefore: Two functions are linearly dependent on the interval if and only if one of the
functions is a constant multiple of the other for all x in the interval .
Examples:
1) y1 sin x and y2 cos x are linearly independent solutions of y y 0 , over the real line.
Neither of these functions is a constant multiple of the other. Indeed, if cos x k sin x for all x ,
2 2
then in particular cos k sin k so k must be 0. But then cos x sin x for all
4 2 4 2
x , a clear absurdity(for example x 0) . Now we know from theorem 1 above that
a sin x b cos x
is a solution for any numbers 𝑎 and 𝑏.Since the solutions are linearly independent, this linear
combination provides an infinity of solutions, instead of just constant multiple of one we already
know.
2x 3x
2) since y1 e 5 x consant , y1 e and y2 e are linearly independent solution of the
y2
Definition (Wronskian):
Let y1 ( x) and y 2 ( x) be any two solutions to the differential equation
y f ( x) y g ( x) y 0 .
The Wronkian of y1 and y1 is defined as
y1 ( x) y 2 ( x)
W ( y1 , y2 )( x) y1 ( x) y2 ( x) y1 ( x) y2 ( x) .
y1 ( x) y2 ( x)
Wronskian Test:
y f ( x) y g ( x) y 0
for x in an open interval I. Let f and g be continues on I. Then
Example:1 Show that y1 sin x and y2 cos x are linearly independent solutions of y y 0
Solution: The student may verify that y1 sin x and y2 cos x are solutions of y y 0 .
Now
sin x cos x
W y1 , y2 x sin 2 x cos 2 x 1 0
cos x sin x
W y1 , y2 x
e3x xe 3 x
e 3 x e 3 x 3xe 3 x 3e 3 x xe 3 x e 3 x e 3 x e 6 x
3e 3x
e 3xe
3x 3x
Since 𝑒 6𝑥 cannot be zero, the Wronskian for these two solutions is non-zero, showing the
linearly independence of the two solutions
Theorem 4:
Let y 1 ( x) and y 2 ( x) be linearly independent solutions of
y f ( x) y g ( x) y 0 (*)
and let y3 ( x) be another solution. Then there exist unique constant c1 and c 2
such that
y3 ( x) c1 y1 ( x) c2 y2 ( x) .
In other words, any solution of (*) can be written as a linear combination of two
given linearly independent solutions of (*) . Hence (*) does not have singular solution.
Proof: Exercise
y1
3 consant y1 e x and y2 3e x are linearly dependent, they don’t form basis .
y2
2x 3x
2. y1 e and y2 e are solutions of the differential equation y 5 y 6 y 0 .Now
y1
since e 5 x consant , y1 e 2 x and y2 e 3 x are linearly independent, thus form a
y2
basis of solutions of the equation for all x .
y f ( x) y g ( x) y 0 (1 )
y( x) c1 y1 ( x) c2 y2 ( x),
where c1 , and c2 are constants and y1 and y 2 form a basis (or a fundamental system)
Example 1: The student may verify that y1 e 2 x and y2 e 3 x solution of the differential
equation:
y 5 y 6 y 0
Solution : Using Wronskian test
e 2 x e 3 x
W y1 , y2 x 2 x 3 x
e 2 x . 3e 3 x e 3 x 2e 2 x e 5 x 0
2e 3e
y1
Now since y1 e 2 x and y2 e 3 x are linearly independent, e 5 x consant , they
y2
form a basis of solutions of the equation for all x and hence a general solution is
y( x) c1 y1 ( x) c2 y2 ( x) c1e 2 x c2e3 x
[Example 2] The student may verify that y1 sin x and y1 cos x are solutions of y y 0
Since
sin x cos x
W y1 , y2 x sin 2 x cos 2 x 1 0
cos x sin x
Thus the solutions are linearly independent. The general solution is then
y( x) c1 y1 ( x) c2 y2 ( x) c1 sin x c2 cos x
y f ( x) y g ( x) y 0
we want to seek another solution y2 such that y1 and y2 are linearly independent
y f ( x) y g ( x) y 0 (*)
is known on some interval I, the second linearly independent solution y 2 of the
equation can be obtained by the method of reduction of order and is given by
e
f ( x ) dx
Proof: Suppose y1 is a known nonzero solution of (*) , to get y 2 , since y1 and y 2 are linearly
y2
u ( x)
y1
must be a non-constant function of x , and y2 uy1 must satisfy (*) . Now, substituting
y2 u y1 ,
But since y1 is a solution of (*) , the expression in the last parentheses is zero.
Thus
u y
2 1 f .
u y1
z' y
2 1 f
z y1
dz dy
z
2 1
y1 f ( x)dx
ln z 2 ln y1 f ( x)dx .
e
1 f ( x ) dx
z ( x) 2
y ( x)
1
1 f ( x ) dx
u( x) e .
y12 ( x)
That is,
e
f ( x ) dx
u ( x) 2 dx .
y1 ( x)
e
f ( x ) dx
y2 uy1 y1 ( x) 2 dx
y1 ( x)
Example 1: Find general solutions for the differential equation
Solution: To use the formula, we need to write the differential equation in the following standard
form:
1 1
y y 2 y 0 ,
x x
and y1 x then
1
x dx
e
y2 x dx x ln x , for x 0 .
x2
Further more, since
y1 x , y2 x ln x, x0
are linearly independent, they are the basis for the given differential equation and the general
solution is
y( x) c1 x c2 x ln x, x 0
Example 2: Find general solutions for the differential equation
x 2 y 3xy 4 y 0, x 0 ,
if one solution is y1 x 2 .
Kassahun Nigatu(MSC),Mesfin Teshome(MSC) and Yitagesu Daba(MSC) 58
Applied Mathematics III
Solution: To use the formula, we need to write the differential equation in the following standard
form:
3 4
y y 2 y 0 ,
x x
and y1 x 2 then
3
dx
x
e
y2 x 2 dx x ln x , for x 0 .
x4
Further more, since
y1 x 2 , y2 x ln x, x0
are linearly independent, they are the basis for the given differential equation and the general
solution is
y( x) c1 x 2 c2 x ln x, x 0
Quick Check Group Class Activity Find the second linearly independent solution to
a) x y xy 2 y 0, x 0 if one solution is y1 x sin(ln x)
2
Discuss your solutions to questions above in small groups and see whether or not they match
with the solution suggested by your instructor.
Instructor's role
Check and give feedback to their answers
Group Activity 1.3
Discuss your solutions to questions above in small groups and see whether or not they match
with the solution suggested by your instructor.
1. Solve the following differential equations by the method of reduction of order if one
solution, y1 ,is given
a) x 2 y 5xy 9 y 0, y1 x3
3 4
b) y y 2 y 0, y1 x 2 , for x 0
x x
c)
y 4 y 0, y1 cos(2 x)
d) (2 x 2 1) y 4 xy 4 y 0, y1 x , for x 0
2. Find general solution of
2 2 ln x
y y 2 y , x 0
x x x
if the two solutions y1 x and y1 x 2 for the corresponding homogeneous equations are
given.
3. Application of reducible equations (Motion)
a) A small body moves on straight line so that the product of its velocity and acceleration is
constant, say, 1m 2 / sec3 .If at t 0 the body’s distance from the origin is 2 meters and its
velocity is 2m / sec , what are the distance and velocity at t 6 sec?
b) What happens in the above problem (prob.2a), if the acceleration equals the velocity (the
other data being as before)? Will the distance at t 6 be larger?
5. Find the curve through the origin in the xy -plan which satisfies y 2 y and whose
tangent at the origin has slope 1 .
Assessment
Asking an answer for some of the questions.
Check students participation in the group activity.
Give feedback to their answers
To solve ( 1 ) we remember from Unit One that a first - order linear differential equation
k x
y ky 0 with constant coefficient k has an exponential function as a solution, y e .
This gives us the idea to try as a solution of ( 1 ) the function
y ex . (2)
Substituting ( 2 ) and its derivatives
2 b c 0 . (3 )
Hence ( 2 ) is a solution of ( 1 ) , if is a solution of the quadratic equation (3 ) .
1
1
2
b b 2 4c , 2
1
2
b b 2 4c .
Our derivation shows that the functions
y1 e1 x and y 2 e 2 x
are solutions of ( 1 ) .
But since b and c are real constants, there are three possible situations for the roots of 1 and 2
if b 4c 0 .
2
Case III A real double root
y( x) c1e1x c2e2 x
where c1 and c 2 are constants.
1
1
2
1
9 1, 2
1
2
1 9 2
So that the general solution is
y( x) c1e2 x c2e x .
But since
y(0) 4, y(0) 1
and
y( x) 2c1e 2 x c2e x ,
then
y(0) c1 c2 4, y(0) 2c1 c2 1 .
From these it follows that
c1 1, c2 3
Therefore, the particular solution is
y( x) e 2 x 3e x
The student should check the answer.
Case II: Two Complex Roots
In this case, since b 2 4c 0 , then the roots are complex conjugates,
1 p iq, 2 p iq ,
where p and q are real constants and q 0 . But then since
s2 e( p iq ) x e px e qx e px cos qx ie px sin qx
then,
1 1
y1 ( s1 s2 ) e px cos qx , y2 ( s1 s2 ) e px sin qx .
2 2i
which are linearly independent solutions and therefore are the basis. Hence the corresponding
general solution is
1
1
2
2
36 1 3i , 2
1
2
2
36 1 3i
Hence
A 4, B 1 .
Therefore, the particular solution is
y( x ) e x (4 cos 3x sin 3x) .
Case III: A Real Double Root
In this case, since b 2 4c 0, then we get only one root
b
2
b
x
hence only one solution y1 e 2
.
To obtain a second independent solution y 2 (needed for a base), we use the reduction method
(discussed in Theorem 2, Sec. 2.2 ). Since one solution is known,
e bx e bx
b b
x x
y2 y1 ( x) 2 dx e 2
bx
dx e 2 dx
y1 ( x) e
b
x
xe 2
.
Thus
b b
x x
y1 e 2
, y 2 xe 2
are linearly independent solutions and therefore are the basis. Hence the corresponding general
solution is
b
x
y( x) (c1 c2 x)e 2
.
Example: Solve the following differential equation
a) y 8 y 16 y 0
b) y 4 y 4 y 0, y(0) 3, y(0) 1
Solution:
a) The characteristic equation is
2 8 16 0
Kassahun Nigatu(MSC),Mesfin Teshome(MSC) and Yitagesu Daba(MSC) 64
Applied Mathematics III
y( x) (c1 c2 x)e 4 x .
b) The characteristic equation is
2 4 4 0
and the root is
1
(4 0 ) 2
2
so that the general solution is
y( x) (c1 c2 x)e2 x .
But from the initial conditions, y(0) 3, y(0) 1 we obtain
c1 3, c2 5 .
Hence the particular solution is
y( x) (3 5x)e2 x .
Summary of cases I III
Roots of Basis of General solution of
Case
2 b c 0 y by cy 0 y by cy 0
Distinct real
I
1 , 2 e1x , e2 x y c1e1 x c2e2 x
Complex conjugate
e px cos qx ,
II 1 p iq y e px ( A cos qx B sin qx)
px
2 p iq
e sin qx
Quick Check Exercises: Write the auxiliary equations for the following differential equations
and hence solve the equations:
(d) y' '2 y' y 0, y(0) y' (0) 1.
(a) y // 3 y / 2 y 0
d2y d y
(b) 2
6 9y 0
dx dx
(c) y' '4 y 0, y(0) 1, y' (0) 0.
Instructor's role
Check and give feedback to their answers
Group Activity 2.3
1. Find general solutions for the following differential equations
a) y 4 y 3 y 0 d) y 16 y 0
b) y 4 y 0, e) y 4 y 0
c) y 2y 10y 0 f) y - 6 y 10y 0
2. Solve the following initial value problems
a) y 2 y y 0, y(0) 1, y(0) 0
Definition:
A differential equation of the form
x 2 y bxy cy 0 ,
(1 )
where b , c are constants, is called the Euler-Cauchy equation.
To solve the Euler-Cauchy equation, we let
y xm ( 2)
Substituting y and its derivatives,
y mxm 1 , y m(m 1) x m 2
in to (1), we find
x 2 m(m 1) x m2 bxmx m1 cx m
x m (m(m 1) x m (bm) cx m
x m (m2 (b 1)m c) 0 .
For x 0 , we obtain the characteristic (or auxiliary) equation
m2 (b 1)m c 0 ( 3)
for determining m in ( 2 ) .
The Three Cases of Solutions
Case I: Distinct Real Roots.
If the auxiliary equation ( 3 ) has distinct real roots m1 , m2 we get the basis of solutions
y1 x m 1 , y2 x m 2
and a corresponding general solution of (1 ) is
y( x) c1 x m1 c2 x m2 ,
for all x for which y1 and y 2 are defined.
Example: Solve the Euler - Cauchy equation
x 2 y 4 xy 6 y 0
Solution: The auxiliary equation is
m2 5m 6 0
The roots are
m1 3, m2 2 .
This gives
y1 x 3 , y2 x 2 ,
and the general solution is
y( x) c1 x 3 c2 x 2 .
Case II: Complex Conjugate Roots
If the roots of ( 3 ) are complex, they are conjugate, say,
m1 iv, m2 iv
and this gives
x m 1 x iv x xiv x eiv ln x
x cos(v ln x) i sin(v ln x)
x m 2 x iv x x iv x e iv ln x
1 m1
2
x x m2 x cos(v ln x)
and
1 m1
2i
x x m2 x sin(v ln x) ,
m1 3 9 13 3 2i,
m2 3 9 13 3 2i,
and the corresponding general solution for all positive x is
1
(1b ) dx
x 2
x
dx
1
(1b )
x 2
ln x .
That is,
1
(1b )
y2 x 2
ln x ,
and the corresponding general solution is
1
(1b )
y( x) (c1 c2 ln x) x 2
c) x 2 y xy 2 y 0 g) x 2 y 7 xy 9 y 0
2. Solve the following Euler-Cauchy initial value problems.
a) x 2 y xy 9 y 0, y(1) 2, y(1) 0
Assessment :
Asking an answer for some of the questions.
Check students participation in the group activity and Give feedback to their answers
1.3.2 Non Homogeneous Linear Differential Equations
y f ( x) y g ( x) y r ( x) , (1)
where f ( x), g ( x) and r (x) are continuous functions and r ( x) 0, and the corresponding
homogeneous equation
y f ( x) y g ( x) y 0 . (2)
Remark:
1. To solve the non homogeneous equation ( 1 ) , we have to solve the homogeneous equation
with constant coefficients and special r (x) , namely, exponential functions, polynomials,
cosines, sines, or sums or products of such functions. These r (x) have derivatives of a form
similar to r (x) itself.
The general procedure in this technique is to assume the particular integral y p of a form similar
to the right member r (x) in equation ( * ) . Choose for y p a form similar to that of r (x) and
If r (x) in ( * ) is one of the functions in the first column, in the table given below, choose the
corresponding function y p in the second column and determine its undetermined coefficients by
corresponding to (*) , then multiply your choice of y p by x repeatedly until no term of the
k k
product x y p is a solution of the homogeneous equation. Then use the product x y p to solve
choose for y p the sum of the functions in the corresponding lines of the second column.
i ke ax ce ax
ii kxn (n 0, 1, . . .) kn x n kn 1 x n 1 . . . k1 x k
d 2 y dy
2 y x2 ,
dx 2 dx
d 2 yc dyc
2 yc 0
dx 2 dx
the subscript c indicating this is the complementary solution. The auxiliary equation is:
2 2 0,
yc Ae x Be 2x .
We will now find the particular solution. As we have an x 2 on the RHS we will guess that the
particular solution is of the form:
y p Ax 2 Bx C.
To substitute this into the function we will also need the first and second derivative of y p :
y p ' 2 Ax B,
y p ' ' 2 A.
2 Ax 2 2 A 2Bx 2 A B 2C x2 .
Now we can equate the powers of x on both sides of the equal sign:
2 A 1,
2 A 2B 0,
2 A B 2C 0.
This is now a set of three simultaneous equations to be solved. In this case we can use the first
1 1
equation to find A , then substitute that into the second to find B and then use the
2 2
3
third equation to find C . This gives us our particular solution:
4
1 1 3
y p x2 x
2 2 4
Note that we must solve the boundary condition with the full solution and not just the
complementary solution! So before solving for A and B we form the full general solution:
yx yc y p Ae x Be 2 x
1 2 1 3
x x .
2 2 4
To solve the second boundary condition we will also need the first differential:
y' x Ae x 2 Be 2 x x .
1
2
y 0 A B
3
0,
4
y ' 0 A 2 B 0.
1
2
3 1 1
From the first we find A B ; using this in the second we find 3B 0 , so B and
4 4 12
2
A , giving us a final answer of:
3
2 x 1 2 x 1 2 1 3
y e e x x .
3 12 2 2 4
two, then
y p ( x) k2 x 2 k1 x k0 .
k2 2, k1 0, k0 1
Thus
y p ( x) 2 x 2 1 ,
y ( x) y n ( x) y p ( x)
A cos 2 x B sin 2 x 2 x 2 1 .
Example 2 – exponentials
a) Find the general solution to:
d2y
4 y e3 x
dx 2
Solution:
Again we start by finding solution to the homogenous equation:
d 2 yc
4 yc 0.
dx 2
the roots of this are purely complex, 2i , thus the complementary solution is given by:
yc A cos2 x B sin2 x .
We then find the particular solution. The only function which can differentiate to give a term
containing e3 x is Ae 3 x , so we will take this as our guess and substitute it in:
9 Ae 3 x 4 Ae 3 x e3 x ,
13 Ae 3 x e3 x
1
so we find that A and our particular solution is:
13
1 3x
yp e .
13
As this question didn’t come with any boundary conditions we cannot solve for A and B , so it
is sufficient to give the full general solution:
y yc y p ,
y A cos2 x B sin 2 x
1 3x
e .
13
y 4 y 4 y e 2 x
Solution: Since
yh ( x) c1e2 x c2 xe 2 x ,
2x
and both e and xe 2 x are solutions of homogeneous equation, then by modification rule ,
y p ( x) a x 2 e 2 x .
1
By substituting y p , yp and yp in the equation we find a and hence
2
1 2 2x
y p ( x) xe .
2
Thus
1 2 2x
y( x) c1e 2 x c2 xe 2 x x e
2
Example 3 – trigonometric
a) Find the general solution to:
d 2 y dy
2 y sin x
dx 2 dx
d 2 yc dyc
2 yc 0.
dx 2 dx
yc Ae 2 x Be x .
y p A sinx B cosx .
We must include both the sin x and cosx terms as one can differentiate into the other. In
order to substitute the particular solution into the equation we calculate the first and second
differentials:
sin x cosx .
3 1
yp
10 10
Hence the full general solution is:
sin x cosx .
3 1
y Ae 2 x Be x
10 10
Example 4 – Mixed
a) Find the solution to the equation:
y 3 y 2 y 4 x e3 x
d 2 yc dy
2
4 c 3 yc 0,
dx dx
2 4 3 0,
yh ( x) c1e 2 x c2e x ,
and since r (x) is given as the sum of forms (i ) and (ii ) , then
y p ( x) (ax b) ce 3 x
1
a 2, b 3, c .
2
Hence
1 3x
y p ( x) 2 x 3 e ,
2
and the general solution is
1 3x
y( x) c1e 2 x c2 e x e 2x 3 .
2
Quick Check Activity:
1. Determine the form of particular solution to the following non homogeneous differential
equation (Individual Activity)
a) y 2 y 2 y e 2 x cos x 3sin x
c) y 4 y x 2 3 sin 2 x
b) y 2 y 5 y e x cos 2 x
2. Solve the following non homogeneous differential equations by the Method of Undetermined
Coefficients (Group Activity)
(a) x 2 y 4 xy 6 y x 2 x
(b) y y x sin x c) y y 2 sin x 4 x cos x
b) y y 1 x x 2 g)
y 8 y 16 y 64 cosh4 x
a)
y y 2 y 10 sin x, y(0) 1, y(0) 3.
b)
y 6 y 13 y 4 e 3 x , y(0) 2, y (0) 4 .
c)
y 4 y e 2 x 2 x, y(0) 0, y(0) 4
c)
y y 2 2 x x 2 , y(0) 8, y(0) 1
Assessment
Asking an answer for some of the questions.
Check students participation in the group activity and Give feedback to their answers
1.3.2.2 The Method of Variation of Parameters
Method of solution by variation of parameters (VOP)
The method of undetermined coefficients discussed in the previous section is limited in its
application. We need another technique with wider application. The technique discussed in this
section is called the method of variation of parameters.
The VOP procedure consists of replacing the constants in the complementary function by
undetermined functions of the independent variable x and then determining these functions so
that when the modified complementary function is substituted into the differential equation,
f (x) will be obtained on the left side.
y f ( x) y g ( x) y r ( x) , (1)
y f ( x) y g ( x) y 0 . (2)
yh ( x) c1 y1 ( x) c2 y2 ( x) ,
where y1 , y2 form a basis of solutions, and the method of variation of parameters involves in
replacing the constants c1 and c2 (here regarded as “parameters” in y h ) by functions u (x) and
y p ( x) u( x) y1 ( x) v( x) y2 ( x) ( 3)
uy1 vy2 0 .
Then
yp ( x) uy1 vy2 .
Substituting the expressions for y p , yp and yp into ( 1 ) and collecting terms containing u and
uy1 vy2 r .
This gives a second equation relating u(x) and v(x), and we have the simultaneous equations
uy1 vy2 r ,
uy1 vy2 0.
Multiply the first equation by y2 and the second by y 2 and add to get
y2 r
u .
y1 y2 y2 y1
Now multiply the first equation by y1 and the second by y1 and add to get
y1r
Thus v .
y1 y2 y2 y1
Since y1 , y2 are linearly independent, we have Wronskian of y1 , y2 ,
W ( y1 , y2 ) y1 y2 y2 y1 0 .
By integration,
y2 r y1r
u dx, v dx .
y1 y2 y2 y1 y1 y2 y2 y1
y2 ( x) r ( x) y1 ( x)r ( x)
y p ( x) y1 ( x) dx y2 ( x) dx .
y1 ( x) y2 ( x) y1 ( x) y2 ( x) y1 ( x) y2 ( x) y1 ( x) y2 ( x)
a) y 4 y e
3x
Solution:
a) A basis of solutions of the homogeneous equation on any interval is
y1 e 2 x , y 2 e 2 x .
This gives
y1 2e 2 x , y2 2e 2 x .
and hence
y1 y2 y1 y2 4.
e 2 x . e3 x e 2 x . e3 x
y p ( x) e 2 x dx e 2 x dx
( 4) ( 4)
1 2x x 1
e e dx e 2 x e5 x dx
4 4
1 3x 1 3x
e e
4 20
1 3x
y p ( x) e .
5
But since
yh ( x) c1e2 x c2e2 x ,
1 3x
y( x) yh ( x) y p ( x) c1e 2 x c2e 2 x e .
5
b) Since
y1 x 3 , y2 x 2
are the basis of the solutions of the corresponding homogeneous equation on any interval and
y1 3x 2 , y2 2 x
then
y1 y2 y1 y2 x 4 .
Re writing
we get
4 6
y y y 21x 6 .
x x
Thus
r ( x) 21x 6 .
(21x 6 ) x 2 (21x 6 ) x 3
y p ( x) x 3 dx x 2
( x 4 ) dx
( x 4 )
1
x 4 .
2
But since
yh ( x) c1 x3 c2 x 2 ,
1 4
y( x) c1 x 3 c2 x 2 x .
2
c) Since
y1 cos 4 x, y2 sin 4 x
are the basis of the solutions of the corresponding homogeneous equation on any interval and
then
y1 y2 y1 y2 4 .
But since
yh ( x) A cos 4 x B sin 4 x ,
y( x) A cos 4 x B sin 4 x e x .
y(0) 6, y(0) 2 ,
and
it follows that
y(0) A 1 6 A5
3
y(0) 4 B 1 2 B .
4
Thus
3
y ( x) 5 cos 4 x sin 4 x e x .
4
Home take Group Activity: Solve the following differential equations by method of variation
of parameters:
(b) x 2 y // xy / 8 y x 4 3 ln x , x 0 (e) y // 5 y / 6 y x 2
(c)
y
Solve x y'' + y' - x = x ex (f) y // 4 y 3 sin x
j) y 2 y y e x
e)
y 2 y y e x ln x
2. Solve the following non homogeneous Euler- Cauchy differential equations.
3. Find the steady-state oscillation of the mass-spring system governed by the equation
given below.
a) y 3 y 2 y 20 cos 2t
b)
y 2 y 5 y 13sin 3t
Assessment
Asking an answer for some of the questions.
Check students participation in the group activity.
Give feedback to their answers
(b) y a2 x b2 y f 2 ,
x x y et , using (b) .
x x ( x x) et , using (*) .
2 x et
Thus x 2 x et
y x x ( 2c1e 2t
2c2e 2t
et ) (c1e 2t
c2e 2t
et )
x (t ) c1e 2t
c2e 2t
et
. y(t ) c1 ( 2 1)e 2t
c2 ( 2 1)e 2t
u u
M, N
x y
The necessary and sufficient condition for the differential equation
M ( x, y)dx N ( x, y)dy 0
to be exact is
M N
.
y x
If an equation of the form M ( x, y)dx N ( x, y)dy 0 is not exact, it can be made exact
by multiplying by an integrating factor.
1 M N k , or
If ,
N y x f ( x)
e k dx , or
then an integrating factor is F ( x) f ( x ) dx
e
1 N M c, or
y f ( y )
If ,
M x
e c dx , or
then an integrating factor is F ( y ) f ( y ) dy
e
f ( x ) dx f ( x ) dxr ( x)dx c
y ( x) e
e
The differential equation of the form y p( x) y g ( x) y a , where a is any real
number, is called the Bernoulli equation.
A differential equation of the form
y f ( x) y g ( x) y r ( x) , (1 )
where f , g and r are continuous functions, is called a linear differential equation of
y f ( x) y g ( x) y 0
and is said to be homogeneous differential equation.
If r ( x) 0, then equation (1) is said to be non homogeneous differential equation.
I Distinct real
1 , 2 e1 x , e2 x y c1e 1x c2 e 2 x
II Complex conjugate
e px cos qx ,
1 p iq y e px ( A cos qx B sin qx)
1 p iq e px sin qx
b e x , xe x y (c1 c2 x)e x
2
m 2 (b 1)m c 0 (2)
Distinct real
m1 , m2 y1 x m 1 , y2 x m 2 y( x) c1 x m1 c2 x m2
Complex conjugate
x cos(v ln x) , y(x)
II m1 iv,
x sin(v ln x) x A cos(v ln x) B sin(v ln x)
m2 iv
1
Real Double root (1 b )
2
x , 1
(1b )
III 1
(1 b) 1
y( x) (c1 c2 ln x) x 2
2 (1 b )
2
x ln x
y by cy r (x) , (1)
is given by
y ( x) y h ( x ) y p ( x) ,
equation y by cy 0 , and y p (x) is a particular solution of equation (1), given by the
formula
y 2 ( x) r ( x) y1 ( x)r ( x)
y p ( x) y1 dx y2 dx .
y1 ( x) y2 ( x) y1 ( x) y2 ( x) y1 ( x) y2 ( x) y1 ( x) y2 ( x)
y y
a) xy y 3 x 4 cos2 . b) xy y x 2 tan 0
x x
c) 9 y ( x y 1) 2 0
c) y (1 3x 1 ) y x 2, y(1) e 1
7. Solve the Bernoulli equation: 3 y y (1 2 x) y 4
1
1
1 sin x
a y y 1 2 y 0, y1 ( x) x 2 sin x c) xy 2 y xy 0, y1
x 4x x
10. Verify that, for any nonzero constant a , y1 ( x) e ax is a solution of y 2ay a 2 y 0 .
Find the general solution.
11. Solve the following differential equations
a) y y 2 y sin 2 x g)
y 2 y y xe x
e 2 x 2e x
b) y 4 y 4 y ; x 0 h) y 2 y y
x2 x3
13. Find the steady-state oscillation of the mass-spring system governed by the equation:
y 2 y 4 y sin 0.2t
14. Solve the following systems of differential equations.
a) x 2 x y sin t c) y x 3 cos t , x(0) 1, y(0) 1
b) x y cos t sin t d) y x cos t sin t
UNIT II
2. LAPLACE TRANSFORMS
UNIT INTRODUCTION
This unit is divided into five sections. The first section presents Definition to Laplace transforms,
Inverse Laplace Transform, discusses Laplace transforms of simple functions and establish
conditions for the existence of Laplace Transforms. The second section establish some useful
properties of Laplace Transforms and Applications of Laplace Transform to solve homogenous
and non-homogenous differential equations and Integral equations will be treated thoroughly in
the third section. Unit step functions and The Second shifting Theorem will be dealt in the third
section. The forth section presents Differentiation and Integration of Transforms. Convolution
Theorem and Integral equations will be studied in the fifth section.
Unit Objectives:
At the end of the unit students will be able to:
Define Laplace Transforms
Finding Laplace transforms of some functions using basic principles
Identify Different Properties of Differential Laplace Transforms
Find Laplace transforms of some functions using the standard Laplace Transforms
Tables and the properties of Laplace transforms
Find the Laplace Transforms of first and second order derivatives
Define Laplace Inverse Transforms
Find Laplace inverse transforms of some functions using basic principles using
the standard Laplace Transforms Tables and the properties of Laplace transforms
Solve differential equations using Laplace Transforms
Apply Laplace transforms to solve differential equations modeling physical
problems
Definition 2.1:
The Laplace transform of a given function f (t ) , that is defined for all t 0 , is defined as the
function F (s) such that
L[ f ](s) e st f (t )dt F ( s) (1)
0
for all s such that the integral exists, where L is referred to as the Laplace Transform
operator. Furthermore the original function f is called the inverse Laplace transform of F (s)
and is denoted by
f (t ) L1 F (s) or L1 F (s) f (t )
Remark 2.1
(a) Laplace transform takes a function f (t ) into a function F (s) of the parameter s .
(b) Original functions are denoted by lower case letters and their transforms by the same
letters in capitals, for instance F (s) denotes the transform of f (t ) . Thus we write
L f (t ) F s or F s e -st f t dt
(c) The defining equation for the Laplace transform is an improper integral, which is defined
as
N
Thus, the existence of the Laplace transform of f depends upon the existence of the limit.
Example: Find the Laplace transform of
a) f (t ) 1, t 0 b) f (t ) e a t , t 0
b) f t t n d) f t sin at
0
T
0
e -st dt
T
1 1 -ST
= lim e st - = lim - e e -s.0
T
s 0 T s
=
1
s
- lim e -st 1 = 0 1= , s>0
1
s
1
s
Therefore L f t = 1 where, s 0 , f t 1 .
s
L tn =
0
e st t n dt
L tn =
n(n - 1)
s2
L(t n2 )
L tn =
n(n - 1)(n - 2).....3.2.1.
sn
. Lt 0
Since L t 0 = L1
1
s
by part (a), we obtain
L tn =
n!
s n1
,
Lt
1
s2
(c) 0
L e at e st t at dt , by Definition
T
T 1 ( as ) t
lim 0 e
( as ) t
= dt = lim a s e dt
T T 0
1 1
= lim
T
-
a s a s
1 1
=- = provided a s 0 or s a .
as sa
T
T e s t 1 s T 1
lim
st
e dt = lim s = lim - s e s =
T
0
T 0 T
(d) By Definition
T
Lsin at = e st
sin atdt lim e st sin atdt
T
0 0
a s2
T
1 sT a sT
st
e sin atdt ( e sin T - 2 e cos aT 2 ) 2
0
s s s s a 2
a s2
Lsin at lim ( e sT sin aT - 2 e sT cos aT 2 ) 2
1 a
T s s s s a 2
Lsin at
a
s a2
2
Instructor's role
Check and give feedback to their answers
Theorem ( Linearity ): Suppose L[ f ](s) F (s) and L[ g ](s) G(s) are defined for s a ,
and a, and b are real numbers. Then
and
L1 c1 F1 s c2 F2 s c1 L1 F1 s c2 L1 F2 s
b) f t 2 sin 3t e 2t
Solution
a) By Linearity Property we have
L{2t 2 4t 1} 2L{t 2 } 4L{t} L1
1! 1 4 4s s
2
2!
2( 3 ) 4 2
s s s s3
b) By Linearity Property we have
Lsinh at L
1
2
1 1
L e at L e at
1
2 s a s a
Lsinh at 2
a
s a2
Example 2 Find the following inverse Laplace transform.
1 72
(i) L1 2 (ii)
s 2 s5
L1
2 3
3 s5
(iii) (v) L1 2
( s 1) s
( s - 2s - 3
- 2s 6 s 1
(iv) L1 2 L1 2
( s 4)
(vi)
( s - 4s
Solution
(i) For k 2 and the linearity of the inverse Laplace transform we get
1 1 2 1
L1 2 = L1 2 = sin 2t
s 2 2 s 2 2
ii. Look for the Laplace transform of basic functions on the right hand side which gives this
denominator. The only possibility is L t n =
n!
s n 1
where n must be 4. Therefore for n 4
72 72.4! 72 1 4! 72 4
L1 5 L1 41
L 41 t 3t 4
s 4!.s 4! s 24
iii. Look for the Laplace transform of basic functions on the right hand side giving a
denominator of ( s 1) and giving a denominator of s 3 .
2 3 1 1 1 3 2
L1 3 2 L1 t
3L 3 2e t
s 1 s s 1 s 2
- 2s 6 1 - 2 s 6
L1 2 = L 2 2
s 4 s 4 s 4
s 1
= 2 L1 2 + 6 L1 2
s 4 s 4
By the Linearity of the inverse transform,
6 1 2
2 cos 2t L 2 cos 2t 3 sin 2t
2 s2 4
In (v) and (vi) but, there is a problem however, and that is that the denominator of F s is not in
form of our transform of basic functions. To proceed, we need the method of partial fractions,
which will allow us to rewrite F s Y in a form we can use.
s5 s5 A B
= = +
s - 2s 3 ( s - 3)( s 1)
2
s -3 s 1
s5 ( A B ) s ( A - 3B )
=
( s - 3)( s 1) ( s - 3)( s 1)
s 5 A Bs A 3B
s5 1 2 -1
L1 2 = L
s - 2s - 3 s - 3 s 1
1 1 1
= 2 L1 L using linearity of L1
s - 3 s 1
Hence
s5
L1 2 2e e
3t t
s - 2 s - 3
s 1 s 1 1 1 5 1
vi. By Partial Fractions = = +
2
s - 4s s ( s - 4) 4 s 4 s4
Therefore
s 1 1 1 1 5 1 1 1 5 4t
L1 2 =- L L = - .1+ e
s - 4s 4 s 4 s - 4 4 4
Theorem 1.2 (First shifting theorem, or shifting in the s variable): Let L[ f ](s) F (s)
for s b 0 . Let a be any number. Then
L e at f t F ( s a) for s a b
and
L1F s a e at f t
at
L e f (t ) = e
st
[e f (t )] dt = e ( s a )t f (t )dt F ( s - a).
at
0 0
and
F ( s a) e
s ( s a )t
f (t )dt e st e at f (t ) dt L (e a t f (t )) .
0 0
If F (s) exist (i.e., is finite) for s greater than some k , then our first integral exists for s a k
Now take the inverse on both sides to obtain the second formula in the theorem.
s sa
For instance, Since L[cos(bt )] , then L[e at cos(bt )] .
s b2
2
( s a) 2 b 2
A short list of Important Transforms
The following table gives a short list of transforms that are basic. Form these transforms we can
obtain nearly all the other transforms that we shall need in this chapter.
SN f (t ) L( f ) f (t ) L( f )
1 w
1 1 8 sin wt
s s w2
2
1 s
2 t 9 coshat
s2 s a2
2
2
2! a2
3 t 10 sinh at
s3 s2 a2
t n (n 0, 1,) n! n!
4 11 e at t n
s n 1
( s a) n 1
(a 1) sa
5 ta, a 0 12 e at cos wt
sa 1 ( s a) 2 w 2
1 w
6. e at sa
13 e at sin wt ( s a) 2 w 2
s
7 cos wt
s w2
2
Important note: We must know how to use the Tables of Laplace Transforms
Examples: Using tables to find Laplace Transforms and Inverse Transforms
Example 1: Find the Laplace Transform of each of the following function
(a) f t e 2t sin 3t c) f t e 2t sinh 3t
So using Linearity
2 3 1
L{2t 2 e 3t 3 sin(3t ) 3} 2 3 2
3 2
3
( s 3) s 3 s
4 9 3
2
( s 3) 3
s 3 2
s
Solutions:
a) We look at the (s 2) 2 9 term on the bottom and find it in the Laplace Table. We find,
e 2t sin 3t
3
( s 2) 2 9
both have (s 2) 9 on bottom.
2
s2
e 2t cos 3t
( s 2) 2 9
Now we want to write the top line in the problem, 5s 2 , in terms of the two top lines “3” and
“s - 2” from the table and put
5s 2 H (3) K (s 2)
This is like our partial fractions problems:
Choose s = 2 8 H (3) H 83
Equate s 5K
So that
5s 2 H (3) K ( s 2) ( s 2)
F s
8 3
5
s 2 2
9 s 2 2
9 3 s 2 9
2
s 22 9
b) In this example, there is one quadratic factor on the bottom line. The first thing to do is
complete the square for this factor, i.e.
s 2 6s 13 s 3 32 13 s 3 9 13 s 3 4
2 2 2
3s 2
F ( s)
s 3 2
4
This can now be done like the previous example (get H = 3 and K 72 ).
c) We begin by completing the square of the denominator, which gives
4 4
We have
s 4s 20 ( s 2) 2 6
2
F s 2
4
( s 2) 2 16
L e-2t sin 4t F ( s 2) F s 2
4
=
( s 2) 2 16
and therefore
e sin 4t
4
L1 2t
( s 2) 16
2
(b) f t 1 2t
2
d) t 5 4 cos(3t ) 6t 2
2. . Find the inverse Laplace Transform of each of the following
9 4 s
a. 2 c.
s 16 s 16
2
( s 2)(s 1)
4s 2
b. d.
s 25
2
s( s 3) 2
Before discussing the possible existence of F s e -st f t dt , it is helpful to define certain
0
terms
Definition 2.2 A function f is said to be piecewise continuous on the closed interval [𝑎, 𝑏] if
the interval can be divided into a finite number of open subintervals (𝑐, 𝑑) = {𝑡 [𝑎. 𝑏]/ 𝑐 <
𝑡 < 𝑑} such that
ii. The function f has a finite limit as t approaches each endpoint from within the
interval; that is, lim f t and lim f t exist.
t d t c
The condition (ii) means that a piecewise continuous function f may contain finite or jump
discontinuities.
Definition 2.3 A function f is said to be of exponential order if there exist real numbers a,
f t Me at for t t 0 .
Now we prove the following basic existence theorem for the Laplace transform of a function f.
for all t 0 and for some constants a, and M 0 . Then the Laplace transform of f (t )
exists for all s a .
Proof: Since the function f is of exponential order, we know that there are constants a, and t 0
Hence
Me ( s a )t
L f t | e
st
f (t ) | dt Me ( s a ) t
dt -
0 0
s a 0
Me ( s a )t M
= - lim
t sa sa
Since first term is zero for 𝑠 > 𝑎, we have
M
le
st
f (t )dt ,sa
0
sa
which implies the existence of the improper integral defining the Laplace transform of f
and completes the proof.
Corollary 2.1 Let f be a piecewise continuous function of exponential order defined on [0,),
L f t e
M
Proof : st
f (t )dt le st f (t )dt , as seen in the proof of Theorem 2.1.
0 0
s-a
a) f (t ) t 2 e 3t b) f (t ) t 2 cost c) f (t ) e t sin 2 t
5
1 1 cos at
d) f (t ) t 2 e) f (t ) 2e cos ( t )
t 2
f) f (t ) , a const.
2 t
h) f (t ) 5e sinh 2t
t
g) f (t ) e cos t i) f (t ) te sin t
2t t
4. Find f (t ) if L ( f ) is
s4 1 7s 1
a) b) c)
s2 4 ( s 1)( s 2)( s 3) ( s 1) 2
5
s ak
d)
1
2 e)
4
s 2s 3
2
f) sk
k 1
2
s 1
2
3 s9
g) i)
s 6s 18
2
s 6s 13
2
finite interval in the range t 0 . Then the Laplace transform of the derivative f (t )
L f s Lf f (0) .
(*) Proof: Exercise
Examples:
(a) Using the Laplace transform of f ' ' find Lsin kt .
f ' 0 k . Therefore by above theorem since L f " (t ) s 2 F s sf 0 f ' 0 we have:
L f " (t ) s 2 F s k L - k 2 sin kt s 2 F s k
Lsin kt
k
s k2
2
Solution (b) Let f t sin 2 t , then f ' t 2 sin t cos t sin 2t , f 0 0 .
Therefore
L f ' (t ) sF s f 0 L f ' (t ) Lsin 2t sF s
where F s L f t L sin 2 t . But we also know that Lsin 2t
2
s 4
2
Therefore
L f ' t L{sin 2t} sL sin 2 t 2
s 4
2
Solving for L sin 2 t L we get
Lsin t
2
s s 4
2
Quick check Exercises
1. a) Using the Laplace transform of f ' ' find L(cos at )
b) Using the Laplace transform of f ' ' find L(t cos t )
Instructor's role
Check and give feedback to their answers
Then
s 2 bs c L ( y) sy(0) by(0) y(0) L (r ) .
So
( s b) y(0) y(0) L (r )
L ( y) .
s 2 bs c
But since
y(0) k0 , y(0) k1,
we get
( s b)k0 k1 L (r )
L ( y) .
s 2 bs c
c. We must be able to find L 1 of the right hand side of the expression in order to
determine the solution y of the initial value problem.
Thus Laplace transform methods are primarily intended for the solution of the linear initial value
problems with constant coefficients.
Example: Solve the following initial value problems by means of Laplace transform
a) y 4 y 1, y(0) 1
b) y 3 y 2 y 4t 6, y(0) 1, y(0) 3
Solution:
a) Taking the Laplace transform of both sides we get
L ( y) 4 L ( y) L (1),
that is,
1
s L ( y) y 0 4 L ( y) , .
s
Hence
1 1 5 1 11
L ( y) .
s 4 s( s 4) 4 s 4 4 s
Taking the inverse on both sides we get
5 1 1 -1 1
y (t ) L -1 L .
4 s 4 4 s
Or
1
y (t ) (5e4t 1) .
4
b) Taking the Laplace transform in both sides and using the differentiation property, we
have,
s L ( y) s 3 3 s L ( y) 1 2 L ( y) s4
2
2
6
s
.
So
4 6 s 3 6s 4
s 3s 2 L ( y) s 2
2
s s s2
.
s 3 6s 4
L ( y) 2 2 2
s 2 s 2 2s 2
s s 3s 2
s s 2 s 1
s 2 2s 2 1 2
2.
s ( s 1)
2
s 1 s
That is,
1 1 1
y (t ) L 1 2L 2 .
s 1 s
Or
y(t ) et 2t .
Example 3
Solution:
(1) Before switched from 1 to 2 at t=0
4
i 2 A i (0 ) 2 A
2
(2) System equation (t>0)
di(t )
L Ri (t ) 0 ( L 1H ) ( R 2ohm)
dt
KVL:
di(t )
2i(t ) 0
dt
di (t ) di (t )
L 2i (t ) L 2 L[i (t )]
dt dt
sI ( s ) i (0 ) 2 I ( s )
( s 2) I ( s ) 2 0
2
I ( s)
s2
i (t ) 2e 2t u (t ) A
a) y 4 y 1, y(0) 1
b) y 3 y 2 y 4t 6, y(0) 1, y(0) 3
c) y // (t ) y / (t ) 1 , y(0) 1, y / (0) 0
Let F (s) be the Laplace transform of f (t ) . If f is piecewise continuous and satisfies the
inequality f (t ) Me t , for all t 0, then
t
L f (u )ds
0
1
s
F ( s)
Proof: Exercise
1 1
Example: Find f (t ) if a) L( f ) b) L( f )
s s
2
s s
3
Solution:
a) Since
Kassahun Nigatu(MSC), Mesfin Teshome(MSC) and Yitagesu Daba(MSC) 114
Applied Mathematics III
1 1 1
s s s s 1
2
and
1
L 1 t
e ,
s 1
we get
1
1 1 1
t x
f (t ) L 1 2 e dx 1 e .
t
L
s s
s s 1 0
b) Observe that
1 1
s s s ( s 1)
3 2
and
1
L 1 2 sinh t.
s 1
Hence
1 1 1 t
f (t ) L 1 3 L 1 2 sinh xdx
s s s s 1 0
cosht 1 .
Group Exercises 2.3.2
1) Solve the following initial value problems by Laplace transform
a) y 2 y e t , y(0) 1
b) y 3 y 10 sin t , y(0) 0
c) y 9 y t 2 , y(0) 0 y(0)
e) y y 2cos t , y(0) 3, y(0) 4
g) y 4 y 3 y 6t 8, y(0) 0, y(0) 0
2. Find f (t ) if L ( f ) is given
1 4 1
a) b) c)
s(s ) s ( s 1) s 1
2 2 2
Definition:
The unit step function (or Heaviside function) H (t a) is defined by
0 if t a
H (t a) . (1)
1 if t a
In particular if a 0 , then
0 if t 0
H (t ) .
1 if t 0
Let H (t a) be the unit constant function for t 0 (also called the Heaviside Function
H (t a)
By definition, we have
a
e a s
L u (t a) e u (t a)dt e u (t a)dt e dt e dt
st st st
, st
s0
0 0 a a s
e as e 0.s 1
In Particular if a 0 , L{H t a } LH t =
s s s
a) The Heaviside function is used to “switch on” inputs.
If 𝑓(𝑡) is defined for all t values then H t f t 0 if 𝑡 < 0 and H t f t f t if t 0 i.e.
𝑓(𝑡) has been ignored if 𝑡 < 0 and switched on for t 0 . This is a very handy function for
looking at the response of a system to an input.
0 if t a 0 if t a
-- On-off effect: H (t a) g (t ) , H (t a) g (t a)
g (t ) if t a g (t a) if t a
b) The Unit step function can also be used to write piecewise defined functions in a compact
form(single formula).
g t ,0 t a
f t
ht , t a
Similarly
g t , a t b
f t ht , b t c
k t , t c
L[ H (t a) f (t )](s) e as L f t a .
Proof LH (t - a) f (t - a) = e st H (t - a) f (t - a) dt
0
= e st f (t - a) dt
a
because H (t a) 0 for t < a and H (t a) 1 for t a. Now let u t a in the last integral.
We get
L H (t - a) f (t - a) = e s
( a u )
f (u )du
0
= e as e st f (u )du e as F s
0
Examples:
1) Compute L f (t ) when
e t 0 t 2 ,
b) f (t ) t
e cos t , t 2
Solution:
1. a) By the Second Shifting Theorem (Theorem 1) we get
L f (t ) L sin a(t b)u(t b) eb s L (sin at )
ae bs
.
s2 b2
b) Since f (t ) has a jump discontinuity at t 2 and
0, t 2 ,
u (t 2 ) cos(t 2
cos(t 2 ) cos, t 2 ,
we have
f (t ) e t u(t 2 ) cos(t 2 ) .
Thus, by the second shifting theorem, we get
L f (t ) L (et ) L u(t 2 )cos(t 2 )
1
e2 s L (cos t )
s 1
1 se 2 s
.
s 1 1 s 2
Example 2: Compute
s
b) L t 2u (t 3)
1 e 2
a) L 1
1 s 2
Solution: a) Observe that
s
2s
L 1
1 e 2 L 1 1 L 1 e
1 s 2 1 s 2 1 s 2
= sin t u (t )sin(t )
2 2
= sin t u (t ) cos t .
2
b) Let f (t 3) t .Then, since
2
t (t 3) 3 (t 3) 3,
we have
f (t ) (t 3) 2 t 2 6t 9 .
2 6 9
Thus L (t 2 u (t 3)) e 3s L (t 2 6 t 9) e 3s 3 2 .
s s s
0,0 t 2
Example 3. Compute Lg t ,where g t 2
t 1, t 2
Lg t L H t 2 t 2 1 e 2 s
s
L t 2 1
2
Lg t
e 2 s
s
L t 2 4t 5 e as
s
L t 2 4 Lt 5L1
e 2 s 2 4 5
Lg t
s s 3
s 2 s
2,0 t
Example 4. Compute L f t where f t 0, t 2
sin t , t 2
s
e 2s
L{ f t } 2
2 e
s s s
The unit step function can be used as a building block in the construction of other functions; for
example,
f1 (t ) u(t a) u(t 2a) u(t 3a), a 0 ,
Yields a three - step staircase. By the linearity property of Laplace transforms we obtain
L f1 (t ) L u(t a) L u(t 2a) L u(t 3a , a 0
e e2 as e3as .
1 as
s
Consider an infinite staircase
1
x
n0
n
1 x x 2
1 x
, x 1.
1
s
1 e as e2 as e3as . . .
1
s
1 e a s (e a s ) 2 ( e a s ) 3 . . .
1
s 1 e as
.
(t a)dt 1
1 e s
s L ( y) L ( y)
2
.
s s
Or
s
s 1 L ( y) 1se
2
.
Hence
1 e s
L ( y) ,
s( s 2 1)
1 1 s
and since 2
s( s 1) s s 1
2
We have
1 s e s se s
L ( y) 2 2 .
s s 1 s s 1
Using the second shifting theorem (Theorem1), we obtain
y(t ) 1 cos t u(t 1) u(t 1) cos(t 1)
s 2 L ( y) 2s 3 2 s L ( y) 2 L ( y) e s ,
Or
s 2
2s 1 L ( y) 2s 7 e s .
Hence we get
2s 7 e s 2( s 1) 5 e s
L ( y)
s 2 2s 1 ( s 1)2 ( s 1)2 ( s 1)2
2 5 es
.
s 1 ( s 1) 2 ( s 1) 2
1
Since L (tet ) , it follows from Theorem 1 that
( s 1)2
e s
e s L te t L (t 1)e ( t 1) u (t 1) .
( s 1) 2
Thus
1 1 2 2 1
a) L tu (t 1) e
s
s
2
s
2
s
b) L t u (t 1) e
s
3
2
s s
2. Compute the inverse of the given Laplace transform
e s e 3 s e 3 s s se s
a) b) c) d)
1 s2 s3 ( s 1) 3 1 s 2
3. Using the Laplace transform, solve the following problems
F ( s) L ( f ) e st f (t )dt .
0
with respect to s can be obtained by differentiating under the integral sign with respect to s ;
thus
dF ( s) d st
F ' ( s) e f (t ) e st (tf (t )) dt L (tf (t )) .
ds 0 ds 0
Theorem:
If L ( f ) F ( s) exists for s a, then L tf (t ) exists for s a and
L tf (t ) L f (t ) F ' ( s) .
d
ds
(1)
d
Or L 1 L f (t ) t f (t ) .
ds
Proof: Exercise
Note that differentiation of the transform of a function corresponds to the multiplication of the
function by t .
Using equation (1) repeatedly, we obtain
L t f (t ) L t f (t ) (1) L t f (t ) (1)
2 n
d 2 d n d
n n -1 n-2
L f(t) .
ds ds 2 ds n
s s2 - k 2
For instance: Since L[cos(kt)] , then L[t cos( kt)]
s2 k 2 (s 2 k 2 ) 2
Examples:
1. Find the laplace transform of
a) L[t sin t ] b) L[t 2 cos t ] c) L[te 2t cos 5t ]
Solution:
1
a) Since L[sin t ] . Hence
s 1
2
d 1
L[t sin t ] 1
2s
2 2
ds s 1 s 1 2
s
b) We have L[cos t ]
s 1
2
d 2 s 2s 3 6s
L[t 2 cos t ] 1
2
ds 2 s 2 1 s 2 1 3
c) By shifting Property
s2
L[e 2t cos 5t ]
s 22 25
Hence using (C)
d s2 s 22 25
L[te 2t cos 5t ] 1
ds s 22 25 s 22 25 2
2.Find
s-a 2ks
a. ln b.
s -b (s - k 2 ) 2
2
Solution:
a. Let F s ln ln s a ln s b . Then
s-a
s -b
F ' s
1 1
s a s b
Therefore
1 1
L1{F ' s } L1 e e
at bt
s a s b
Using Inverse Laplace Transform of derivatives property, we have
L-1 F ' s (1)tf t
1 1
L1{F ' s } L1 e e tf t
at bt
s a s b
e bt e at
f t
t
b. Let Gs
2ks
.
(s - k 2 ) 2
2
where F s
k
.
s -k2
2
Therefore
d k 1 1 k
L1 Gs L1{F ' s } L1 2 2
t L 2 2
t sinh kt where
ds s k s k
Example 2: Solve the differential equation.
ty ty y 0, y(0) 0, y (0) 3
Solution: Since
d 2
L ( y) L ( y) 0 .
ds s 1
Separating variables, we have
dL ( y ) 2
ds ,
L ( y) s 1
and an integration yields
ln L ( y) 2 ln s 1 c1 .
Or
c
L ( y) .
( s 1) 2
That is,
y(t ) c t e t .
y(t ) 3tet .
Remark: Observe that all variable coefficients equations can not be solved by this method.
It works only when
a) The coefficients are polynomials in t ;
b) The differential equation involving L( y) can be solved; and
c) The inverse transform of L( y) can be found.
Quick Check Group exercises
1. Find the Laplace Transform of the following functions
a. te 2t sin t d. t cosh at
b. t 2 3t 2 sin 3t e. te t sinh t
c. t sinh at
f. t 2 4 cos 2t
2. Find the Laplace inverse transform
a 1 (s 2 b 2 ) sa
a. arctan b. ln c. cot 1
s 2 (s 2 a 2 ) b
Theorem:
f (t )
If L ( f ) F ( s) exists for s a , and lim exists, then
t 0 t
f (t )
L F ( s )ds ( s a) .
* *
(2)
t s
f (t )
Or L 1 F ( s*)ds *
s t
Remark: If g (t ) f (u ) du ,
0
then
b) L f (t ) L (g(t)) sL g (t ) g (0);
c) g (0) 0 .
1
Finally, using parts (b) and (c), we get L g (t ) L f (t ) .
s
t 1
Thus we conclude that L f (u )du L f(t) .
0 s
Kassahun Nigatu(MSC), Mesfin Teshome(MSC) and Yitagesu Daba(MSC) 128
Applied Mathematics III
we obtain
sin t du
L 2 lim(arctan u ) lim(arctan u)
t s u 1 u u s
arctan s .
2
t sin u 1 sin t 1
b) L du L L (sin t )dt
0 u s t ss
1 du
2
s s u 1
1
limarctan u limarctan
s u u s
u
1
arctan s .
s 2
Group Exercise 2.3.4
1 cos au
t
a)
2
t sin 2t b)
sin 3t
t
c)
1 cos at
t
d)
0
u
du
1 s s
a) b) c)
( s 3) 3 ( s a) 2
2
( s 4) 2
2
Definition:
If f and g are piecewise continuous functions, then the convolution of f and g ,
written f g , is defined by
t
f g t f ( x) g (t x)dx .
0
f (t u) g (u)du
0
t
g (u ) f (t u )du ( g f )(t ) .
0
Hence ( f g )(t ) ( g f ) (t ) .
Note that we can take the convolution in either order without altering the result.
Proof: By definition
su sv
F ( s)G( s) L ( f ) L ( g ) e f (u )du e g (v)dv
0 0
e
s (u v )
f (u ) g (v)dvdu (2)
0 0
If we make the change of variables t u v, then v t u and dv dt, and integral (1) is
equal to
u=t
Region of integration
u
dtdu dudt ,
0 u 0 u
0 u
t
e f (u ) g (t u )dudt
s t
=
0 0
t
= e g (t u ) f (u )du dt
s t
0 0
Kassahun Nigatu(MSC), Mesfin Teshome(MSC) and Yitagesu Daba(MSC) 131
Applied Mathematics III
= e s t ( g f )(t )dt
0
= e s t ( f g )(t )dt . = L ( f g ) .
0
Hence L ( f g) L ( f )L (g) .
L 1 ( F (s)G(s)) f g .
That is, L 1 ( L ( f ) L ( g )) f g .
This result can sometimes be used to find the inverse Laplace transform of a function that is a
product of factors whose inverse transforms are known
Note: It is no true that
L 1 F (s)G(s) L 1 ( F ) L 1 (G) .
1 1 1
For example, if F ( s ) and G ( s ) 2 then F ( s ) G ( s) 3
s s s
but
1 t
2
L 1
( FG) L 3 ,
1
L 1 ( F ) 1, L 1 (G) t
s 2
and, clearly L 1 ( FG) L 1 ( F ) L 1 (G) .
Examples:
1 s
1. Compute a) L
1
2
b) L
1
2
2
s( s 4) (s 1)
Solution:
1. a) Since
1 1 1 1
L 1
2
L
s ( s 4) 2
s( s 4)
Kassahun Nigatu(MSC), Mesfin Teshome(MSC) and Yitagesu Daba(MSC) 132
Applied Mathematics III
and
1 1
L 1 1, L 1 te 4t ,
2
s ( s 4)
then
1 1 1 1 t
L
1
L
2
1 * te xe 4 x dx
2
4t
s( s 4) s ( s 4) 0
1 1 1
= te 4t e 4t .
4 16 16
b) Since
s s s
L 1 2 L 1 2
2
( s 1) s 1 s 2
1
and
s 1
L 1 2 cost , L 1 2 sin t ,
( s 1) s 1
then
s s s
L 1 2 L 1 2
2
cost * sin t
( s 1) s 1 s 2
1
t
= cos x sin(t x)dx
0
t sin t
= .
t
2. Taking Laplace transforms
L y 2 L ( y) 8 L ( y) L ( f ) .
That is
s 2 L ( y ) s 2s L ( y ) 2 8 L ( y ) L ( f ) .
Or
( s 2 - 2s - 8) L ( y) L ( f ) s 2 .
Thus
s2
L y
1
L( f ) 2
s 2s 8
2
s 2s 8
1 1 1 1 1 1 2 1
= L( f ) L( f ) .
6 s-4 6 s2 3 s4 3 s 2
6
1
6
y(t ) L 1 e 4 t L f L 1 L(e 2t )L f e 4 t e 2 t
1 1
3
2
3
1 1 1 2
e 4t f (t ) e 2 t f (t ) e 4 t e 2 t .
6 6 3 3
4t 2 t
This solution is valid for any function f (t ) having a convolution with e and e
2.5.2 Integral Equation
Although the convolution theorem is obviously very useful in calculating inverse transforms, it
also helps in solving certain integral equations, that is, equations in which the unknown function
y (t ) appears under the integral (and perhaps also out side of it).
Example: Solve
t
a) y (t ) 1 y ( x)dx .
0
t
b) y(t ) sin 2t y( x) sin 2(t x)dx .
0
Solution:
t
a) y (t ) 1 y ( x) 1
0
y (t ) 1
Or
1
L ( y) .
s 1
From this it follows that
1
y (t ) L - 1 e t .
s 1
t
b) y (t ) sin 2t y ( x) sin 2(t x)dx sin 2t y (t ) sin 2t .
0
Thus
y(t ) 2 sin 2t .
e
2 t 2x
e cos(3x)dx.
0
b) To show that
1
t x
L 2 (s) f ( x)dx
1
s 0 0
t F(s) t
L f ( x)dx , where F (s) L ( f ) . H int : f ( x)dx (1 f )(t )
0 s 0
4. Find L ( f ) if
t
a) f (t ) (t x) 3 sin xdx
0
t
b) f (t ) e ( t x ) cos 2 xdx
0
t
c) f (t ) e17( t x ) x19 dx
0
1, t
a) y 4 y 13 y f ( x), y(0) y(0) 0, where f (t )
0, t
2, t
b) y 6 y 18 y f ( x), y(0) y(0) 0, where f (t )
0, t
6. Find the inverse Laplace transform of each of the following functions by using
convolution.
2 1 1
a) b) c)
s ( s 2 4)
2
( s 1) ( s 2 9)
2
( s 1) 2
2
L ( f ) F ( s) e
st
f (t )dt
0
and
L1 c1 F1 s c2 F2 s c1 L1 F1 s c2 L1 F2 s
Let L[ f ](s) F (s) for s b 0 . Let a be any number. Then
L e at f t F ( s a) for s a b and L1F s a eat f t
0, t a
u (t a) (a 0)
1, t a
Then the Laplace transform of the derivative f (t ) exists when s , and
L f s Lf f (0) .
If f and g are piecewise continuous functions, then the convolution of f and g ,
t
written f g , is defined by f g t f ( x) g (t x)dx .
0
3. Find
a) L sin t
2
b) L cos
2
t
c) L te sin t t
2
d) L t cos3t
4. Find f (t ) if L ( f ) is given
9 s 1 1 s 1
s 2 s 2 9
a) b)
s2 s 1
s4 4
c) d)
s2 4 s 2s 2
3
1 1
e) f)
( s 3)3 s5 s3
1 e 4 s s2
g) h)
s5 s2 3
5. Using the Laplace transform, solve the following problems
b) y (t ) t sin(t x) y ( x)dx
3
8. Find the inverse Laplace transform of each of the following functions by using convolution
ea s a s
d) e) f)
s ( s 2) s (s a 2 )
2 2
(s 2 ) 2
2
e 3s 3 s
g) h) j)
s3 s 4 ( s 2 1) ( s 2 2) 2
t
1
6 0
b) y (t ) t (t x) 3 y ( x)dx
t
c) y (t ) 2t 4 y ( x) sin 2(t x)dx
0
t
d) y (t ) 1 sinh t (1 x) y (t x)dx
0
t
e) y (t ) e 2 cos (t x) y ( x)dx
t
UNIT III
FOURIER TRANSFORMATIONS
Unit Introduction
This unit deals with Fourier series and Fourier integrals; and Fourier transformations. The unit is
divided into three sections. The first section presents periodic functions and trigonometric
Fourier series. In the second section we will deal with the Fourier series. Fourier Integrals and
The Fourier Transforms are treated in the third and fourth sections respectively.. By doing so
students will be able to express terms and concepts related to Fourier series and integrals; and
Fourier transformations.
Unit Objectives:
At the end of this unit students will be able to:
Define Fourier series, Fourier integrals, Fourier transformations
Identify Fourier series of even and odd functions
Demonstrate how to differentiate Fourier series of even and odd functions
together with solving exercises
Distinguish Fourier series and Fourier integrals; Fourier transformation and
demonstrate together with solving exercises.
Compute Fourier cosine transform, Fourier sine transform and Fourier transform of a
function.
Demonstrate the applications of Fourier series, Fourier integrals, Fourier and
transformations together with solving exercise.
Apply the concept of Fourier series, Fourier integrals, Fourier and Laplace
transformations in solving real life problems.
Definition:
A function f is said to be periodic if it is defined for all real number x and if there is
some positive number p such that
y
T
f t dt f t dt
a 0
Example 3.1-1 will use this property to integrate a 2-periodic function shown in Figure 3.1-2.
Example 3.1-1. Let f be the 2-periodic function and N is a positive integer. Compute
N
N
f 2 ( x )dx if f(x) = x + 1 on the interval 0 x 2
y
1
x
-3 -2 -1 1 2 3
-1
Figure 3.1-2. A 2-periodic function.
Solution
N N 2 N 4 N
N
f 2 ( x )dx = N
f 2 ( x )dx +
N 2
f 2 ( x )dx + +
N 2
f 2 ( x )dx
2
N N 2 1
= N f ( x )dx = N ( x 1) dx = N ( x 1) 3
2 2 2
f ( x )dx
N N 2 0
3 0
N N 2
N
f 2 ( x )dx =
3
[ 1 1] = N
3
The most important periodic functions are those in the (2-period) trigonometric system
Such a series is called a trigonometric series, and a n and bn are called the coefficients of the
series.
Note that each term of the series (2) has the period 2 . Hence if the series (2) converges, its
sum will be a function of period 2 .
Activity1.1
1. Find the smallest positive period p of
cos x, sin x, cos 2 x, sin 2 x, cosx, sin x, cos 2x, sin 2x
2. If f (x) and g (x) have period p, show that h( x) af ( x) bg( x) ,
(a, b cons tan ts) also has the period p .
3. Show that the function f ( x) c , where c is constant is periodic function of period p
for every positive p .
f ( x) cos x, a b 2 .
= for m = n
= 2 , m n 0
sin mx sin nxdx
= 0 for m n
= for m = n
= 0, m n 0
cos mx sin nxdx
= 0 for all m and n
b
Definition: If
a
f ( x ) g ( x )dx = 0 then f and g are said to be orthogonal over the interval [a, b].
In general, if any two members ψn, ψm of a set of functions {ψi} satisfy the condition
b
x x dx 0
a
n m
then ψn and ψm are said to be orthogonal, and (1.7) is known as the orthogonal condition in the
interval between a and b. The set {ψi} is an orthogonal set over the same interval.
Thus if the members of the set of trigonometric functions are
1, cos x, sin x, cos 2x, sin 2x, cos 3x, sin 3x, . . . ,
then this is an orthogonal set in the interval from −π to π.
It is a tool in abstract analysis and electromagnetism and statistics and radio communication.
People have even tried to use it to analyse the stock market. (It didn't help.) The representation of
Kassahun Nigatu(MSC), Mesfin Teshome(MSC) and Yitagesu Daba(MSC) 144
Applied Mathematics III
Assume that f (x) is periodic function of period 2 is integrable over a period and can be
represented by a trigonometric series,
f ( x ) a0
n 1
(a n cos nx bn sin nx) ; (1)
that is, we assume that this series converges and has f (x) as its sum. Given such a function
f (x) ,we want to determine the coefficients a n and bn of the corresponding series (1) . Because
of orthogonality between the trig functions, one has an easy way to evaluate the a n and bn in
equation (1).
Determination of a0 .
Determination of a n .
Multiplying (1) by cosmx, where m is any fixed positive integer, and integrating from
to :
f ( x) cos mxdx a0 a n cos nx bn sin nx cos mxdx
n 1
ao cos mxdx an cos nx cos mxdx bn sin nx cos mxdx ,
n 1
by integrating term-by term.
a if n m
m .
0 if n m
Thus
1
am
f ( x) cos mxdx,
m 1, 2, .
Determination of bn .
Multiplying (1) by sin mx, where m is any fixed positive integer, and then integrating from
to :
f ( x) sin mxdx a0 a n cos nx bn sin nx sin mxdx
n 1
a0 sin mxdx an cos nx sin mvdx bn sin nx sin mxdx
n 1
b , n m
m .
0, otherwise
Thus
1
bm
f ( x) sin mxdx,
m 1, 2, .
1
a0 f ( x)dx
2
1
an
f ( x) cos nxdx,
n 1, 2, , (2)
1
bn
f ( x) sin nxdx,
n 1, 2, ,
These numbers, given by (2) , are called the Fourier coefficients of f (x) .
Definition:
The trigonometric series
a0
n 1
a n cos nx bn sin nx
with coefficients given by (2) is called the Fourier series expansion of f (x) .
Since the series in (1) hold good for any function of period 2 , the above formula also hold
good for the range 0 to 2 .
Example 1: Find the Fourier series of
k if x 0
f ( x) f ( x 2 ) f ( x), k 0
k if 0 x ,
Solutions:
y
k
x
-3 -2 - 2 3
-k
Since the value of f (x) at a single point does not affect the integral, we can leave f (x)
undefined at x 0 and x .
1 cos nx cos nx
0
k k
n n 0
2k
1 cos n
n
4k
for n odd,
n .
0 for n even
Hence
4k 4k 4k
b1 , b2 0, b3 , b4 0, b5 .
3 5
and therefore, the Fourier series of f (x) is
4k 1 1
sin x sin 3x sin 5 x .
3 5
4k sin(2n 1)
n 1 2n 1
.
2 2
1 1 1 1 2 2
a0
0 f(x) dx
0 ( - x ). dx
x - x
2 0
1
(2 2 2 2 ) 0
2 2
1 1 nx x cos n
an
0 f(x) dx
0 ( - x ).cos
dx
1
-1 sin nx
2 n
1
0 ( - x ).cos nx dx -1
0 cos nx
n2
1 x 1 2
sin nx - cos nx
n n2 0
1
(1 1) 0
n 2
2 2
1 1 nx
bn
0 f(x) dx
0 ( - x ).sin
dx
2
1 x sin nx
0 ( - x ).sin nx dx -1
-1 cos nx
n
1 x 1 2
cos nx - sin nx -1
n n 2 0
0
n
2
sin nx
1 2
( 2 )
n n
and therefore, the Fourier series of f (x) is
sin 2 x 1 1
2 sin x sin 3x sin 4 x .
2 3 4
sin nx
2 .
n 1 nx
0 if x 0
a. f ( x) f ( x 2 ) f ( x),
1 if 0 x ,
0 if x 0
b. f ( x) f ( x 2 ) f ( x),
t if 0 x ,
2 if x 0
c. f ( x) f ( x 2 ) f ( x),
1 if 0 x ,
0 if x 0
d. f ( x) f ( x 2 ) f ( x),
sin t if 0 x ,
Instructor's role
Check and give feedback to their answers
We must be careful at this stage not to conclude that we have proved that every periodic function
f (x) has a Fourier expansion that converges to it. The convergence problem can be solved by
the famous theorem of Dirichlet.
Definition:
A function f (x) is piecewise continuous on a finite interval a x b if it is defined
on that interval and is such that the interval can be subdivided into finitely many intervals in
each of which f (x) is continuous and has finite limits as x approaches either endpoint of the
interval of subdivision from the interior.
It follows from this definition that finite jumps are the only discontinuities that a piecewise
continuous function may have.
Example 1: Consider the function
4, x 0
f ( x)
4, 0 x
It is a piece wise continuous and has a jump at x 0 . Its left - hand limit there is 4 and its
right - hand is 4 . So the average of these limits is 0 . Thus the Fourier series of f converge to
this value when x 0 because then all its terms are 0 . Similarly for other jumps.
Definition:
A function f t is said to satisfy the Dirichlet conditions on an interval I if and only if
f t is bounded and has at most a finite number of local maxima and minima and a finite
number of discontinuities on I.
right-hand derivatives at each point of that interval, then the Fourier series (1) of f
(with coefficients (2) ) is convergent. Its sum is f (x) , except at a point x0 at which
f (x) is discontinuous and the sum of the series is the average of the left and right-hand
limits of f (x) at x0 .
Proof: Exercise
The conditions of Theorem above under which a periodic function possesses a valid Fourier
expansion are referred to collectively as the Dirichlet conditions. Note that this theorem gives a
sufficient condition for a Fourier series to converge to f .
Example 1: Obtain the Fourier expansion of
f x x in - < x <
1
2
Solution: We have,
1 1 1
2 ( x)dx
a0 f ( x)dx
1 x2
= x
2 2
1 1 1
an
f ( x) cos nxdx
2 ( x) cos nxdx
1
0 0
2
1 1
bn
2 ( x) sin nxdx
cos nx sin nx
x n
1
(1)
2 n
2
(1) n
n
Using the values of a0 , an and bn in the Fourier expansion, since f x is continous
a0
f ( x) an cos nx bn sin nx
2 n 1 n 1
we get,
(1) n
f ( x) sin nx
2 n 1 n
This is the required Fourier expansion of the given function.
Example 2. Obtain the Fourier expansion of
, x 0
f(x) =
x,0 x
Deduce that
2 1 1
1 ......
8 32 52
Here,
1
0
a0 dx xdx
0 2
1
0
an cos nxdx x cos nxdx
0
1
n
2
(1) n 1
1
0
bn sin nxdx x sin nxdx
0
1
n
1 2(1) n
Fourier series is
f(x) =
1 1
(1) 1 cos nx
n
1 2(1) n
sin nx
4 n 1 n 2 n 1 n
Note that the point x=0 is a point of discontinuity of f(x). Here f(x+) =0, f(x-)=- at x=0. Hence
[ f ( x ) f ( x )] 0
1 1
2 2 2
The Fourier expansion of f(x) at x=0 becomes
1 1
[(1) n 1]
2 4 n 1 n 2
2
1
or 2
[(1) n 1]
4 n 1 n
Simplifying we get,
2 1 1
1 ......
8 32 52
Quick Check Exercises
Q1 Find the Fourier series to represent the function f (x) , given by
x for 0 x
f ( x)
2 x for x 2
1 1 1 2
Deduce that ............
12 2 2 32 8
Q2 Obtain the Fourier expansion
, if x 0 2
1
f ( x) and hence deduce that 1
x, if 0 x 8 n 1 ( 2n 1)
2
0, if x 0
Q3 If f ( x) . Prove that
sin x, if 0 x
n n
f ( x) a0 (a
n 1
n cos
L
x bn sin
L
x) (1)
1 L
2 L L
(a) a0 f ( x)dx
1 L n
(b) an
L L
f ( x) cos
L
xdx n, 1, 2, 3, . (2)
1 L n
(c ) bn f ( x) sin xdx, n 1, 2, 3, .
L L L
Remark: We may replace the interval of integration by any interval of length p 2 L , for
0 if 2 x 1
a) f ( x) k if 1 x 1 , p 4
0 if 1 x 2
b) f ( x) 0 if 2 x 0 , p 4
1 if 0 x2
Solution:
y
a)
x
-3 -2 -1 1 2 3
12 1 1 1 2
k
a0 f ( x)dx 0dx kdx 0dx ,
4 2 4 2 1 1 2
12 nx k1 nx 2k n
an
2 2
f ( x ) cos
2
dx
2 1
cos
2
dx
n
sin
2
,
12 nx k1 nx
bn f ( x) sin dx sin dx 0 .
2 2 2 2 1 2
Then the Fourier series is
n
sin
k 2k 2 cos nx
f ( x)
2
n 1 n 2
k 2k 1 3 1 5
cos x cos x cos x .
2 2 3 2 5 2
b) Since p 2L 4 then L 2 and
12 12 1
a0 f ( x)dx dx ,
4 2 40 2
12 nx 12 n
an
2 2
f ( x ) cos
2
dx
20
cos
2
xdx 0,
2
12 nx 12 n 1 (1) n , n odd
bn f ( x) sin dx sin xdx n .
2 2 2 20 2 n 0, n even
Then the Fourier series is
sin(2n 1) x
1 2
f ( x) 2 .
2 n 1 2n 1
Solution : Y
8
-4 -2 0 2 4 X
-8
1
4 2 4
1
a0
2
0
f ( x)dx 8dx 8dx 0 .
2 0 2
1
4 2 4
1
an
0
f ( x) cos nxdx
0
8 cos nxdx
2
8 cos nxdx 0 .
1
4 2 4
1
bn
0
f ( x) sin nxdx 8 sin nxdx 8innxdx
0 2
1 cos nx cos nx
2 4
8 8
n 0 n 2
2k
1 cos n
n
4k
for n odd,
n .
0 for n even
Hence
4k 4k 4k
b1 , b2 0, b3 , b4 0, b5 .
3 5
and therefore, the Fourier series of f (x) is
4k 1 1
sin x sin 3x sin 5 x .
3 5
4k sin(2n 1)
n 1 2n 1
.
1 if 1 x 0
a) f ( x) , p2
1 if 0 x 1
b) f ( x) x, 2 x 2, p4
Instructor's role
Check and give feedback to their answers
y y
a) f (x) equals b) f (x) equals
k
1
x x
c) f ( x) x, x d) f ( x) x 2 , x
e) f ( x) x, 0 x 2 f) f ( x) x x , x
1 if 2 x 2 x if 2 x 2
g) f ( x) h) f ( x)
1 if x 3 0 if x 3
2 2 2 2
2 1 1 1
i) f ( x) x , x . Show that 1
8 9 25 49
2. Find the Fourier series of the periodic function f (x) of period p 2 L .
x if 1 x 0
a) f ( x) , p2
x 2 if 0 x 1
0 if 2 x 0
b) f ( x) , p4
2 if 0 x 2
x if 0 x 2
c) f ( x) , p4
3 if 2 x 4
d) f ( x) x , 2 x 2, p4
e) f ( x) 3x 2 , 1 x 1, p2
1 1
x if x 0
f) f ( x) 2 2 , p 1
1
x if 0 x 1
2 2
3. Find the Fourier series of the periodic function that is obtained by passing
the voltage v(t ) vo cos100 t through a half - wave rectifier.
Definition:
A function f is said to be even if f ( x) f ( x) , for all x .
-L X
L
Even function
Definition:
A function g is said to be odd if g ( x) g ( x), for all x .
X
-L L
Odd function
Kassahun Nigatu(MSC), Mesfin Teshome(MSC) and Yitagesu Daba(MSC) 159
Applied Mathematics III
Remarks:
1. If f is an even function on [ L, L] , then
L L
f ( x)dx 2 f ( x)dx
L 0
f ( x)dx 0
L
f ( x), 0 x L
g ( x)
f ( x), L x 0
Similarly we can extend it as odd function:
f ( x), 0 x L
h( x )
f ( x), L x 0.
4. The product q fg of an even function f and an odd function g is odd. Hence if
n
f (x) is even, then f ( x) sin x is odd, which implies
L
n
L
1
bn
L f ( x) sin
L
x dx 0 .
L
1 L n
Similarly, if f (x) is odd then a0
2L L
f ( x)dx 0 and f ( x) cos x is odd, which intern
L
implies
1 L n
a n f ( x) cos x dx 0 .
L L L
Thus we proved the following theorem.
with coefficients
1L 2L n
L 0
a0 f ( x)dx, an f ( x) cos xdx, n 1, 2 , .
L0 L
The Fourier series of an odd function f (x) of period p 2L is a Fourier sine series
n
f ( x) b
n 1
n sin
L
x,
with coefficients
2L n
bn
L0
f ( x) sin
L
xdx, n 1, 2 , .
In particular, the Fourier series of an even function f of period 2 is a Fourier cosine series
f ( x) a 0 a
n 1
n cos nx
with coefficients
1 2
a0
f ( x)dx,
0
an
f ( x) cos nxdx,
0
n 1, 2, .
Similarly, the Fourier series of an odd function f of period 2 is a Fourier sine series
f ( x) b
n 1
n sin nx ,
with coefficients
2
bn
f ( x) sin nxdx,
0
n 1, 2, .
Fourier coefficients of f .
Proof: Exercise
Example 1: Find the Fourier series of the function
a) f ( x) x if x and f ( x 2 ) f ( x)
b) f x x, L x L
Solution:
a) Since
f ( x) x f ( x), for all x,
thus f is odd and hence
a0 an 0,
2 cos n 2
bn
x sin nxdx 2
0 n
(1) n 1 .
n
Hence the Fourier series is
(1) n 1
f ( x) 2 sin nx
n 1 n
1 1
2 sin x sin 2 x sin 3x .
2 3
b) Since this is an odd function, the Bns will = 0.
1 L n x 2 L n x
An
L L
x sin
L
dx
L 0
x sin
L
dx
n x
Let y,
L
2 n L L 2L n
L 0 n
An y sin y dy y sin y dy
n n 2 0
n
2L
sin y y cos y
n 2
0
2L
n 2
0 n 1 0 0 n2L 1
n n 1
n 1
n x
f x x 1
2L
sin
n 1 n L
Graph of 1st term and the sum of the first 4 terms of equation.
Example 2. Obtain the Fourier expansion of f(x) = x2 over the interval (-, ). Deduce that
2 1 1
1 ......
6 22 32
Solution:
The function f(x) is even. Hence
1 2
a0 =
f ( x)dx = f ( x)dx 0
2 2 x3
= x dx 2
0 3 0
2 2
a0
or 3
1
an
f ( x) cos nxdx
2
=
f ( x) cos nxdx,
0
since f(x)cosnx is even
2
x
2
= cos nxdx
0
1
Also, bn
f ( x) sin nxdx 0
since f(x)sinnx is odd.
Thus
2
(1) n cos nx
f ( x) 4
3 n 1 n2
2
1
2 4 2
3 n 1 n
1 2
1 n 2 6
2 1 1
Hence, 1 .....
6 22 32
3. Obtain the Fourier series of f(x) = 1-x2 over the interval (-1,1).
Solution:
The given function is even, as f(-x) = f(x). Also period of f(x) is 1-(-1)=2
Here
1 1
1
a0 = f ( x)dx = 2 f ( x)dx
1 1 0
1
1
x3
= 2 (1 x )dx 2 x
2
0 3 0
4
3
1
1
1 1
an f ( x) cos(nx)dx
1
2 f ( x) cos(nx)dx as f(x) cos(nx) is even
0
1
= 2 (1 x 2 ) cos(nx)dx
0
4(1) n1
=
n 2 2
1
1
bn f ( x) sin(nx)dx =0, since f(x)sin(nx) is odd.
1 1
The Fourier series of f(x) is
2 4 (1) n 1
f(x) = 2 cos(nx)
3 n 1 n 2
Quick Check Class Exercises
1. Obtain the Fourier series for the function f(x) given by
2x
1 , if x 0
a) f ( x)
1 2 x , if 0 x
Hence deduce that
1 1 1 2
............
12 32 5 2 8
b) f ( x) x, 2 x 2, p4
c) f ( x) 1 x , 2 x 2, p4
Instructor's role
Check and give feedback to their answers
Fourier sine series. These two series are called the two half - range expansions of f , which is
given only on “ half the range” (half the interval). The form of these series is given in Theorem
1 (sec. 1.2.4).
X
-L L
X
-L L
X
-L L
Example1
(d) : Represent f (x) by a Fourier series, f (x) = x, 0 < x < L.
a0 2nx 2nx
Let f ( x ) ( an cos bn sin )
2 n 1 L L
2 L 2 L
L 0
a0 f ( x )dx xdx L
L 0
2 L 2nx 2 L 2nx
an
L 0
f ( x ) cos
L
dx x cos
L 0 L
dx
2 L 2nx 2 L 2nx
bn
L 0
f ( x ) sin
L
dx x sin
L 0 L
dx
1
n =3
n =5
0.8
0.6
n =1
0.4
0.2
0
0 0.2 0.4 0.6 0.8 1
y
L
(2)Fourier cosine series x
2L L L 2L
a0 nx
Let f ( x ) a n cos
2 n 1 L
2 L
L 0
a0 xdx L
y
L
2L L L x
(3)Fourier sine series
2L
L
nx
Let f ( x ) bn sin
n 1 L
nx nx nx
L
2 L 2 L
bn x sin dx ( x cos cos dx )
L 0 L n L 0
0 L
nx ( 1) n 1 2 L
L
2L 2L
( 1) n 2 2 sin
n n L 0 n
n 1
2L
( 1) nx
f ( x)
n 1 n
sin
L
1
5 terms Cosine
0.8
Full range
0.6
0.4
Sine
0.2
0
0 0.2 0.4 0.6 0.8 1
k
x
2L n 23 n
an
L0
f ( x ) cos
L
xdx
30
k cos xdx 0 .
3
Hence the Fourier cosine series is f ( x) k.
x
6
4k
1 (2n 1)
f ( x)
2n 1 sin
n 1 3
x
4k 1 1 5
sin x sin x sin x .
3 3 5 3
k
x
-k
a0 a
e inx e inx e inx e inx
f ( x) ( a n cos nx bn sin nx ) 0 ( a n bn )
2 n 1 2 n 1 2 2i
a 0 a n ibn inx a n ibn inx
[( )e ( )e ] c0 ( cn e inx c n e inx ]
2 n 1 2 2 n 1
where
a0 1
2 2
c0 f ( x )dx
a ibn 1 1
cn n
2
2 f ( x )(cos nx i sin nx )dx
2
f ( x )e inx dx
a ibn 1 1
cn n
2
2
f ( x )(cos nx i sin nx )dx
2
f ( x )e inx dx
Hence
f ( x) c e
n
n
inx
x
where
1
2
cn f ( x )e inx dx
nx
cn e
i
f ( x) L
L x L (2.8)
n
where
nx
1 L i
cn
2 L L
f ( x )e L
dx
Example 1: Find the complex Fourier series of f (x) = ex, x if and f (x + 2 ) = f (x).
Solution : Let f ( x) c e
n
n
inx
x
1 1 1 1
f ( x )e e e e
inx inx (1in ) x
cn dx x
dx dx e (1in ) x
2 2 2 2 (1 in )
1 in
[e (1in ) e (1in ) ]
2 (1 n )
2
1 in
[e (cos n i sin n ) e (cos n i sin n )]
2 (1 n 2 )
1 in 1 in
(1) n (e e ) (1) n sinh
2 (1 n )
2
(1 n )
2
sinh
1 in
ex
(1 n
n
2
)
(1) n e inx x
Example 2: Let f (x) = x for 0 x 2 , find the complex form of Fourier series of f (x) on the
Solution : Let f ( x) c e
n
n
inx
, where
2
1 2 1 2 1 x2 1 4 2
c0
2 0
f ( x)dx
2 0
xdx
2 2
2 2
0
2 2 1 inx 2
e inx dx
1 1 1 2
cn
2 0
f ( x)e inx dx
2 0
xe inx dx xe
2 in 0 0
2
1 1 i 2 n e inx 1 1 e i 2 n 1 1 i
2e 2
2 in in 0 2 in in in n
1
f ( x) i e inx , 0 x 2
n n
n0
b) f ( x) x, 0 x e) f ( x) e x , 0 x 2
c) f ( x) x 2 , 0 x L f) f ( x) x, 0 x L
1. a) Show that the complex Fourier series of a periodic function f
of period 2 is given by
f ( x) c e
n
n
inx
,
1
where cn f ( x)e dx, n 0, 1, 2,
inx
2
f ( x) c
n
n e L
,
where
in x
1 L
cn
2L L
f ( x) e L dx, n 0, 1, 2,
ii) f ( x) x, x , f ( x 2 ) f ( x)
1, x 0
iii) f ( x) , f ( x 2 ) f ( x)
1, 0 x
0, x 0
iv) f ( x) , f ( x 2 ) f ( x)
1, 0 x
d) Show that the complex Fourier Coefficients of an even function are pure
imaginary.
left-hand derivative at every point and if f (x) is absolutely integrable, that is, the integral
f ( x) dx
where
1 1
A( )
f (v) cos vdv,
B( )
f (v) sin vdv (2)
with taking all values. At appoint where f (x) is discontinuous the value of the Fourier
integral equals the average of the left - and right - hand limits of f (x) at that point.
Proof: Exercise.
Example: Find the Fourier integral representation of the function
1, x 0
f ( x) 1, 0x
0, x
Solution: Since
1
10
A( ) f (v) cosvdv (1) cosv dv cos vdv 0 ,
0
Kassahun Nigatu(MSC), Mesfin Teshome(MSC) and Yitagesu Daba(MSC) 176
Applied Mathematics III
and
1
10
B( )
f (v) sin vdv
( 1) sin vdv 0 sin vdv
2 1 cos
,
the Fourier integral representation is
2 1 cos
f ( x)
0
sin xdw.
Indeed, in Theorem 1 above, if f (x) is an even function, then B( ) 0 in (2) , and
2
A( )
f (v) cosvdv .
0
(3)
The Fourier integral (1) then reduces to the Fourier cosine integral
2
B( )
f (v) sin vdv .
0
(5)
The Fourier integral (1) then reduces to the Fourier sine integral
f ( x) B( ) sin xd ( f odd). (6)
0
Observe that these simplifications are quite similar to those in the case of a Fourier series
discussed earlier.
Remark: The Fourier integral representations may also be used for evaluating integrals.
Examples:
1. Find the Fourier cosine and sine integrals of
1, 0 x 1
f ( x) 2, 1 x 4
0, x 4
2. Find the Fourier integral representation of the function
x
f ( x) e ,
and evaluate the integral
1
1
0
2
d .
Solution: 1.
(a) Fourier cosine integral.
Since
2 2 1 4
A( )
0
f (v) cosvdv cosvdv 2 cosvdv
0 1
2
2 sin 4 sin .
the Fourier cosine integral representation is
2 2 sin 4 sin
f ( x)
0
cos xd .
2 2 1 4
B( )
0
f (v) sin vdv sin vdv 2 sin vdv
0 1
2
(1 cos 2 cos 4 )
the Fourier sine integral representation is
2 1 cos 2 cos 4
f ( x)
0
sin xd .
2. Since
x e x , x 0
f ( x) e x
e , x 0
and
f ( x) f ( x) , for all x ,
f ( x) e
x
is even and so B( ) 0 .
Thus
2 2
A( ) f (v) cos vdv e v
cos vdv
0 0
2 1
.
1 2
Therefore, the Fourier integral of f (x) is the Fourier cosine integral
2 cos x
f ( x) d .
0 1 2
But since
2 d
f (0) 1 ,
0 1 2
we obtain
d
0 1 2 2 .
1, x 1
sin
b) f ( x)
0, x 1
, and evaluate the integral
0
d .
x, x
c) f ( x)
0, | x |
k , 10 x 10
d) f ( x) , k cons tan t
0, | x | 10
1 if 0 x 1 x if 0 x a
a) f ( x) c) f ( x)
0 if x a 0 if x a
e x if 0 x 1
b) f ( x)
0 if x 1
5. a) Show that the complex Fourier integral of a non periodic function f is given by
c( )e
ix
f ( x) d , where
1
f (v )e
iwv
c( w) dv .
2
1
f (v ) e
i ( x v )
f ( x) dvd .
2
Or it can be given by
a) f ( x) e
x
sin x, 5 x 5
c) f ( x)
x 0, x 5.
b) f ( x) xe
where
2
A( w)
f (v) cos wvdv
0
2 2 2 2
f (v) cos wvdv f (v) cos wvdv .
0
0
We now set
2
fˆc ( w) f (v) cos wvdv ,
0
2 ˆ
A( w) f c ( w) .
Writing v x , we have
2
Fc ( f ) fˆC ( w) f ( x) cos wxdx . (2)
0
Which is called the Fourier cosine transform of f (x) , and from (1) , we obtain
2 ˆ
f ( x)
f C (w) cos wxdw .
0
This formula gives us back f (x) from fˆC ( w) , and we therefore call f (x) the inverse
The process of obtaining the transform Fc fˆc from a given function f is called the Fourier
cosine transform.
3.4.1.2 Fourier Sine Transformation
For an odd function f (x) , the Fourier integral is the Fourier sine integral
f ( x) B( w) sin wxdw (3)
0
where
2
B( w)
f (v) sin wvdv
0
2 2 2 2
f (v) sin wvdv f ( v ) sin wvdv .
0 0
We now set
2
fˆs ( w) f (v) sin wvdv ,
0
2 ˆ
B( w) f s ( w) .
Writing v x , we have
2
Fs ( f ) fˆs ( w) f ( x) sin wxdx .
0
Which is called the Fourier cosine transform of f (x) , and from (3) , we obtain
2 ˆ
f ( x)
f s (w) sin wxdw .
0
This is called the inverse Fourier sine transform of fˆs ( w) .The process of obtaining Fs fˆs ( w)
from f (x) is called the Fourier sine transform. Similarly for the inverse process.
1
Note that Fc 1 and F s are the inverses of Fc and Fs , respectively where “ c ” and “ s ”
x, 0 x a
f ( x)
0, x a
b) Find Fc (e x )
Solution:
2a 2 a sin wa cos wa 1
a) Fc ( f ) x cos wxdx .
0 w w2
2a 2 sin wa a cos wa
Fs ( f ) x sin wxdx .
0 w2 w
Fc ( f )
2
e
0
x
cos wxdx
2 1
1 w 2 x
lim e x w sin wx cos wx 1
2 1
.
1 w2
continuous on every finite interval, then the Fourier cosine and sine transforms of f (x) exist.
Theorem:
The Fourier cosine and sine transforms are linear operators,
that is,
a) Fc (af bg) aFc ( f ) bFc ( g )
b) Fs (af bg) aFs ( f ) bFs ( g )
where a and b are constants.
2
Proof: a) Fc (af bg)
af ( x) bg( x)cos wxdx
0
2 2
a
f ( x) cos wxdx b
0 g ( x) cos wxdx
0
aFc ( f ) bFc ( g ) .
Thus
Fc (af bg) aFc ( f ) bFc ( g ) .
Similarly for (b).
Fc f ( x) wFs f ( x)
2
f (0) , ( 4a )
Fs f ( x) wFc f ( x) (4b)
2
Fs f ( x) f ( x) cos wxdx
0
2
lim f ( x) cos wx f (0) w f ( x) sin wxdx
x 0
2
lim f ( x) cos wx f (0) w f ( x) sin wxdx
x 0
wFs f ( x)
2
f (0) .
Similarly,
2
Fs f ( x) f ( x) sin wxdx
0
2
lim f ( x) sin wx w f ( x) cos wxdx
x
0
wFc f ( x) .
Fc f ( x) w 2 Fc f ( x)
2
f (0) ,
Fs f ( x) w 2 Fc f ( x)
2
w f (0) .
Example: Find the Fourier cosine transform of
f ( x) e ax , a 0 .
Solution: By differentiation,
f ( x) (e ax ) a 2e ax a 2 f ( x) .
Thus
a 2 Fc ( f ) Fc ( f ) w2 Fc f ( x)
2
f (0)
w2 Fc f ( x) a
2
.
From this it follows that
2
(a 2 w 2 ) Fc ( f ) a .
2 a
Or Fc ( f ) , a 0.
a 2 w2
x2 s2
1
Q18. Show that Fourier cosine transform of e 2
is e 4 .
2
Activity 3.4.3
1. Find the Fourier cosine and sine transforms of
cos x, 0 x b
a) f ( x)
0, xb
b) f ( x) e x , x .
c) f ( x) xe a x , a 0
d) f ( x) xe x cos x.
1, 0 x 1 1 cos w
e) f ( x) , show that Sinwdw
0, x 1 w 4
0
a) f ( x) xe ax
x , 1 x 1
b) f ( x)
0, otherwise
sin x, | x |
c) f ( x)
0, | x | .
1
1 1
iwx
f ( x) f (v)e iw( x v ) dvdw
iwv
2
f ( v ) e dv e dw . (2)
2 2
The expression in brackets is a function of w , which is denoted by F ( f ) fˆ ( w) ,and is called
a x
Example: Find the Fourier transform of f ( x) e , a 0.
Solution:
a x e ax , x0
f ( x) e ax , a 0.
e , x0
Then
1 0 ( a iw)
0
a x
F (e ) e dx e ( a iw )
dx
2
1
2
1
a iw 1 xlim
e ( a iw) x
1
lim e ( a iw) x 1 .
a iw x
But since
and
Hence
a x 1 1 1
F (e )
2 a iw a iw
1 2a
.
2 a w
2 2
Exercises:
e s / 2 .
2
Proof: This follows from the fact that integration is a linear operation.
1 1
a f ( x) e dx b g ( x )e
iwx iwx
dx
2 2
aF ( f ) bF ( g ) .
F ( f )
1
lim
2 x
f ( x ) e iwx
lim
x
f ( x ) e iwx
iw
f( x ) e iwx
dx
.
But since lim f ( x) 0 , the expression in the brackets is zero. Thus
x
1
F ( f ) iw f ( x )e dx iwF ( f )
iwx
.
2
F ( f ) iwF ( f ) (iw) 2 F ( f ) w2 F ( f ) .
And in general
F ( f ( n ) ) (iw) n F ( f ) , n Z .
Activity 3.4.4
1. Find the Fourier cosine and sine transforms of
cos x, 0 x b
a) f ( x) b) f ( x) e x , x .
0, xb
c) f ( x) xe a x , a 0 d) f ( x) xe x cos x.
1, 0 x 1 1 cos w
e) f ( x) , show that Sinwdw
0, x 1 w 4
0
x , 1 x 1
a) f ( x) xe ax b) f ( x)
0, otherwise
sin x, | x |
c) f ( x)
0, | x | .
3. Find the Fourier transform of the following functions.
1 if a x b e x if x 0
a) f ( x) b) f ( x) x
0 otherwise e if x 0
e x if a x b x if 0 x a
c) f ( x) d) f ( x)
0 otherwise 0 otherwise
e kx if x 0, k 0 x if 1 x 1
e) f ( x) f) f ( x)
0 if x 0 0 otherwise
sin x, a x a
f ( x) x, | x | 1
g) 0, | x | a, a constant h) f ( x)
0, | x | 1
1
a0 f ( x)dx ,
2
1
an
f ( x) cos nxdx,
n 1, 2, ,
and
1
bn
f ( x) sin nxdx,
n 1, 2, ,
where
1 1
A( )
f (v) cos vdv, B( )
f (v) sin vdv
2
Fc ( f )
f ( x) cos wxdx .
0
2
Fs ( f )
f ( x) sin wxdx .
0
x if 2 x 0
c) f ( x) sin x , x d) f ( x)
x if 0 x 2
e) f ( x) 2 x , x f) f ( x) x 2 , x
2. Find the complex Fourier series of
a) f ( x) 2 x, 0 x 3, f ( x 3 ) f ( x) ,
0, 0 x 1
b) f ( x) , f ( x 4 ) f ( x)
1, 1 x 4
3. Find the Fourier integral representation of the function
1, | x | a
f ( x) a0
0, | x | a ,
4. Find the Fourier cosine integral of
x 2 if 0 x 1
f ( x)
0 if x 1
5. Find the Fourier sine integral of
e x if 0 x 1
f ( x)
0 if x 1
6. Find the complex Fourier integral (if it exists) for
x , 2 x 2
f ( x)
0, | x | 2
x if 1 x a
6. Find the Fourier cosine transform of f ( x)
0 otherwise
x 1 if 0 x 1
7. Find the Fourier sine transform of f ( x)
0 otherwise
8. Find the Fourier transform of
x 2 x if x 0 kx if a x b
a) f ( x) b) f ( x)
0 if x 0 0 otherwise
xe x if x 0 x 2 if 0 x 1
c) f ( x) d) f ( x)
0 if x 0 0 otherwise
UNIT FOUR
Unit introduction:
In this unit we study the calculus of vector fields. (These are functions that assign vectors to
points in space.) In particular we define line integrals (which can be used to find the work done
by a force field in moving an object along a curve). Then we define surface integrals (which can
be used to find the rate of fluid flow across a surface). The connections between these new types
of integrals and the single, double, and triple integrals that we have already met are given by the
higher-dimensional versions of the Fundamental Theorem of Calculus: Green’s Theorem,
Stokes’ Theorem, and the Divergence Theorem.
Unit Objectives:
At the end of this unit students will be able to:
Define scalar and vector fields.
Describe Vector Point functions graphically
Define Line integrals, surface integrals
Differentiate and integrate vector-valued functions. For a position vector function of
time, interpret these as velocity and acceleration.
Evaluate line and surface integrals. Identify when a line integral is independent of
path and use the Fundamental Theorem of Line Integrals to solve applied problems
Find the curl and divergence of a vector field, the work done on an object moving in a
vector field, and the flux of a field through a surface. Use these ideas to solve applied
problems
Identify and use Green’s Theorem, the Divergence (Gauss’s) Theorem and Stokes’
Theorem.
y x
x
F (x, y, z) =
F (x, y, z) = x i y j
Example 1: The gravitational force F (x, y, z) exerted by a point mass m at the origin on a unit
mass located at point (x, y, z) (0, 0, 0) is given by:
Gm
F ( x, y, z ) u ( x, y, z )
x y2 z2
2
where G is a gravitational constant, u ( x, y, z ) is the unit vector emanating from (x, y, z) and
directed towards the origin. Hence the vector field is called the gravitational field of the point
mass.
xi y j z k
Then u ( x, y, z ) = .
x y z
2 2 2
Hence, F (x, y, z) =
Gm
x i y j z k .
x
3
2
y 2
z2 2
If a point (x, y, z) in space is represented by the vector r , then the gravitational field can be
written as:
Gm
F (x, y, z) = r , where r = x i y j z k .
3
r
Example 2: Imagine a fluid flowing steadily along a pipe and let V(x,y,z) be the velocity vector
at a point (x,y,z). Then V assigns a vector to each point (x, y, z) in a certain domain E (the
interior of the pipe) and so V is a vector field on R 3 called a velocity field. The speed at any
given point is indicated by the length of the arrow.
A vector field F can be expressed in terms of its components, say M, N and P as follows:
F (x, y, z) = M ( x, y, z ) i N ( x, y, z ) j P ( x, y, z ) k
In short we can write
F (x, y, z) = M i N j P k .
Note that: M, N and P are scalar fields.
Gm
Solution F (x, y, z) = r , where r = x i y j z k .
3
r
x Gm
Then M dx = G m dx + k (y, z) = + k (y, z)
3
x2 y2 z 2
x2 y 2 z 2 2
y Gm
N dy = G m dy + ℓ(x, z) = + ℓ(x, z)
3
x y2 z2
2
x2 y 2 z 2 2
z Gm
and P dz = G m dz + q (x, y) = + q (x, y)
3
x2 y2 z 2
x2 y 2 z 2 2
Gm
Now let f (x, y, z) = , then F = grad f.
x y2 z2
2
M N P
Definition 2.16 Let F = M i N j P k be a vector field such that , and
x y z
exists. Then the divergence of F, denoted div F or F is the function defined by
div F (x, y, z) = F ( x, y, z )
M N P
= ( x, y, z ) ( x, y, z ) ( x, y, z )
x y z
Divergence is used to measure the density of a vector field in order to see how quickly matter is
moving into and out of a region.
Example 1: Find the divergence of the vector field F, where
F ( x, y, z) ( y z ) i ( x z) j ( x y) k
( y z ) ( x z ) ( x y)
Solution: div F (x, y, z) = = 0.
x y z
Therefore, div F = 0.
Note that: If div F = 0, then F is said to be divergence free or solenoidal (incompressible).
( x e y ) ( y e x ) (sin yz )
Solution div F (x, y, z) = .
x y z
= e y e x y cos yz
Definition 2.17 Let F = M i N j P k be a vector field such that the first partial
derivatives of M, N and P all exist. Then the curl of F, which is denoted curl F or
F is the function defined by
curl F (x, y, z) = F ( x, y, z)
P N M P N M
= i j k
y z z x x y
i j k
Curl F =
x y z
M N P
Curl is used to determine the amount of rotation of a vector field. If the curl of a vector field is
zero, then it is said to be ‘irrotational’, i.e. it does not rotate.
Solution M = y + z, N = x + z and P = x + y.
P N M P N M
Then = = 1, = = 1 and = = 1.
y z z x x y
Therefore, curl F = 0.
P P N M N M
Then = x e xy , = y e xy and = = = = 0.
y x z z x y
Therefore, curl F = x e xy i y e xy j .
Quick check Exercises : Find the divergence and curl of the following vector fields:
a) F x 2 y iˆ z 3 yjˆ xkˆ
b) F y 2 z iˆ x3 z 2 ˆj 3 ykˆ
d) F ze xy iˆ z 2e x ˆj xe3 y kˆ
2 f 2 f 2 f
( f ) = div (grad f) =
x 2 y 2 z 2
The right side of this formula is the Laplacian of f usually denoted by 2 f . A function that
satisfies the equation
2 f = 0
f f N M
grad f (x, y) = i j , curl F (x, y) = k
x y x y
M N 2 f 2 f
div F (x, y) = and 2 f (x, y) =
x y x 2 y 2
Note that: (*) holds for a vector field F = M i N j P k need not imply that F is conservative.
If the domain of F is 3 and if (*) holds, then there is a function f such that F = grad f.
F (x, y) = M ( x, y) i N ( x, y) j
the conditions in (*) reduce to
N M
x y
and the corresponding statements in the theorem holds for such vector fields.
Example 1 Let F ( x, y, z ) yz i xz j xy k
and G ( x, y, z ) ( x 2 y 2 ) i ( y 2 z 2 ) j ( x 2 z 2 ) k .
Show that F is the gradient of some function but G is not the gradient of any function.
y, z) on a curve in space. These integrals are called line integrals and are denoted by C
f ( x, y)ds
or C
f ( x, y, z )ds . They were invented in the early 19th century to solve problems involving fluid
When the path of the integration is closed curve, this fact is indicated by f x, y, z ds in place of
C
f ( x, y, z )ds .
Theorem: The value of a line integral over a smooth curve C is independent of the parametric
representation of the curve C.
Example 1. Evaluate the following line integral f ( x, y)ds, f ( x, y)dx, f ( x, y)dy
C C C
x 4t 1
where f x, y x 2 y 2 ; C : , 1 t 1
y 3t 1
Method 1. By parametrizing the variables.
x 4t 1
f ( x, y ) x 2 y 2 ; C : , 1 t 1
y 3t 1
C 1 1
25t 3 1
25 25 50 310
5 t 2t 5 1 2 1 2 5 4
2
3 3 3 3 3
1
50 62
1
C
f ( x, y )dy 25t 2 2t 2 3dt 3 4 3 62
1 3 3
x
y 2 dx 2 xydy along the path consisting of the line
2
Example 2: Evaluate the line integral
segment from (0, 0) to (0,2) followed by the segment from (0, 2) to (1, 2)
Solution:
y (1, 2)
C1
C2
C C 1C 2
C1 : x 0, dx 0 x
y 2 dx 2 xydy 0
2
C2
1
x3
x
1
11
C 2 : y 2, dy 0 2
y dx 2 xydy
2
x 4 dx 0 4 x
2
C2 0 3 0 3
x 11 11
Therefore, 2
y 2 dx 2 xydy 0
C 3 3
x 2t
Example 3. Sketch the path and evaluate the line integral (2 x 3 y)dx, C : , 0t 2
C y 3t
Y (4, 6)
x 2t 2
C ( 2 x 3 y ) dx , C :
y 3t
, 0 t 2 C ( 2 x 3 y ) dx t 0(4t 9t )(2dt )
2 2
10tdt 5t 2
20 answer : 20
t 0 0
F dr F r t r t dt F Tds
'
C a C
Therefore,
i. If F x, y, z Pi Qj Rk then
C
f ( x, y, z )ds = Pdx Qdy Rdz Pdx Qdy Rdz
C C C C
C
f ( x, y)ds = Pdx Qdy Pdx Qdy
C C C
Example 1: Evaluate F dS
where F x, y, z xi z 2 zx j xyk ,and S is the surface of the
S
xdydz z 2 zx dzdx xydxdy
Hence
F dS F ndS xdydz z
zx dzdx xydxdy
2
S S S
1
4 2 4 z 2 4 2 x 2 2 y
xdydz z 22
2
zx dzdx xydxdy
0 0 0 0 0 0
3
Exercise: Show that evaluating also F dS F ndS also gives the same result.
S S
Q(x, y) =y, C is the boundary of the region between the x-axis and the graph of y=sinx for
0 x
Solution:
Sin x
1
sin x
Corollary to Green’s Theorem: The area A of the region R bounded by the piecewise smooth
simple closed curve C is given by
1
2 C
A= ydx xdy ydx xdy
C C
Example 3. Use the corollary to Green’s theorem to find the area of the region between the x-
axis and one arch of the cycloid with parametric equations x=5(t-sint) and y=5(1-cost)
Cycloid
C2
C1
2 0
A ydx ydx ydx odx 5(1 cos t )(5)(1 cos t )
C C1 C2 0 2
2 2 2
1 cos 2t
25 1 cos t dt 25 1 2 cos t cos 2 t dt 25 1 2 cos t
2
dt
0
0 0
2
2
3 1 3
25 t 2 sin t sin 2t 25 2 75
2 4 0 2
Example 4. Use the corollary to Green’s theorem to find the area of the region between the
y x 2 and y x 3
1 0 1 1
Area xdy x(3x )dx x(2 x)dx 3x dx 2 x dx
2 3 2
C C1C 2 0 1 0 0
3 1
3x 4 2 x 3 2 1
4 3 0
4 3 12
Example 5. Find the area bounded by one loop of the curve x=sin2t, y=sint
Note:
x sin 2t x 2 sin t cos t x 2 4 sin 2 t cos 2 t x 2 4(sin 2 t )(1 sin 2 t )
x 2 4 y 2 (1 y 2 )
For the y int ercepts , x 0 4 y 2 (1 y 2 ) 0 y 0 y 1 or y 1
Y
1
-1
2 cos 3 t
Area xdy 2 sin t cos t cos tdt 2 cos t sin tdt
2
C 0 0
3 0
2
3
cos 3 cos 3 0
4
3
Example 6. Find the work done by the force F 5x 2 y 3 i (7 x 3 y 2 ) j in moving a particle
once around the triangle with vertices (0, 0), (3, 0) and ((0, 6)
6 y
3 x
Work Pdx Qdy 5 x 2 y 3dx 7 x 3 y 2 dy 21x 2 y 2 15 x 2 y 2 dA
C C R
3 2 x 6 3 3
dx 2 x 2 2 x 6 dx
2 x 6 972
6 x y dydx 2 x y
2 2 2 3 3
0 0 0
0
0
6
Vector form of Green’s Theorem or divergence theorem in the plane.
Theorem:(Divergence Theorem):Let R be a bounded region with complete boundary C
oriented in positive sense, and suppose F is a vector field defined on R and its boundary. Then
F nds divFdA
R
Proof:
dx dy dx dy
dt i dt j dt i dt j dx dy
1) T T T i j
v ds ds ds
dt
dy dx
2) Let N i j
ds ds
dx dy dy dx
3) T N 0, therefore T N and N is normal to C
ds ds ds ds
dy dx
4) F N Pi Qj i
dy dx
j P Q
ds ds ds ds
dy dx
5) F nds P Q ds Pdy Qdx
C
C
ds ds C
Green 's Theorem
P Q P Q
6) F nds Qdx Pdy x y dA x y dA
C C R R
divF dA
R
Example 1. Verify the divergence theorem for R the interior of x 2 y 2 1 and F=2xi+3yj
P Q
1) divF 2 3 5 F nds divFdA 5dA 5
x y C R Circle
x cos t
2) Let C : x 2 y 2 1 or , 0 t 2 , P 2 x, Q 3 y
y sin t
2
0 0 0
2
5 1 5
t sin 2t 2 5
2 4 0 2
Example 2. Use the divergence theorem to evaluate the line integral (4 x y)dx (3x 2 y)dy ;
C
P
4
P 4 x y x P Q
Q divF 2
Q 3x 2 y 2 x y
y
2 2 2 2
P Q
(4 x y)dx (3x 2 y)dy 2rdrd r d 3d 6
2
dA 4
C R
x y 0 1 0
1
Definition: Flux is the rate of flow of the fluid across C in the direction of vector n
Example 9. Use Green’s theorem to calculate the outward flux = F nds Where F=2xi+3yj
C
x2 y2
and C is the ellipse 1
9 4
x2
2 1
P Q
3 9
flux F nds divFdA
C R R
x y
dxdy 4
0
2 3dxdy
0
3 /2
x2 x
20 2 1 dx 40 1 sin 2 3 cos d , sin
0
9 0
3
/2 /2 /2
1 cos 2 1 1
120 cos d 120
2
d 120 sin 2 30
0 0
2 2 4 0
Stokes' Theorem:
Stokes' Theorem states that if is an oriented surface with boundary curve C, and F is a vector
) and T (the unit tangent vector to C) are chosen so that n T points inwards from C along .
4.5 Gauss Divergence Theorem
In this section we are going to relate surface integrals to triple integrals. We will do this with the
Divergence Theorem.
Theorem: Divergence Theorem
Let E be a simple solid region and S is the boundary surface of E with positive orientation. Let
be a vector field whose components have continuous first order partial derivatives. Then,
Example 1 Use the divergence theorem to evaluate F ndS where F x, y, z xyi
1 2
y j zk
S
2
The region E for the triple integral is then the region enclosed by these surfaces. Note that
cylindrical coordinates would be a perfect coordinate system for this region. If we do that here
are the limits for the ranges
0 z 4 3r 2 , 0 r 1 and 0
We’ll also need the divergence of the vector field so let’s get that.
divF y y 1 1
The integral is then,
Group Activity
x2 y2
1.Compute the surface area of the ellipsoid whose equation is z 2 1 . We may
4 9
parametrize this ellipse, as we have done in the past, using modified spherical coordinates
2. (a) Compute the surface area of the portion of the paraboloid z = 9 x2 y2 that lies above
the xy-plane.
3
(b) Evaluate z
2
dS , where is the surface whose area you found in part (a).
a) F x, y, z xyi yzj zxk . S is the part of the paraboloid z 4 x 2 y 2 that lies above
the square with vertices [-1,1] , [-1,1] with positive orientation.
c) A fluid has density of 15 and velocity v xi yj k . Find the rate of flow downward
through S.
d) Use the divergence theorem (Gauss’ Thm) to find F ndS
S
given F(x,y,z)=x3i + 2xz2j +
3y2zk where S is the surface of the solid bounded by the paraboloid z 4 x 2 y 2 and
the xy plane.
F (x, y, z) = M ( x, y, z ) i N ( x, y, z ) j P ( x, y, z ) k
div F (x, y, z) = F ( x, y, z )
M N P
= ( x, y, z ) ( x, y, z ) ( x, y, z )
x y z
Then the curl of F, which is denoted curl F or F is the function defined by
curl F (x, y, z) = F ( x, y, z)
P N M P N M
= i j k
y z z x x y
Then the line integral of F along C is
b
F dr F r t r t dt F Tds
'
C a C
Let E be a simple solid region and S is the boundary surface of E with positive orientation. Let
be a vector field whose components have continuous first order partial derivatives. Then,
Review Problems
1.Compute divergence, curl, grad
i. Compute gradients of
a) exp(x)cos(y) b)
Kassahun Nigatu(Msc),Mesfin Teshome(Msc) and Yitagesu Daba(Msc) 214
Applied Mathematics III
coordinates. Not that you can write down sˆ ˆ , ˆ ẑ and zˆ sˆ without any computations. You
may do this two ways, either (a) repeat the derivation the way we did it for A or (b) read
through the notes about the general form of the derivative operators in curvilinear coordinates
and compute A h
by identifying the functions s , h and hz for cylindrical coordinates.
3. Div, grad, curl on scalar
i. Given the scalar function T(x,y,z) (e.g. the temperature at any point in the room)
Which of the three operations (div, grad, or curl) can be sensibly operated on T?
For each which can:
a) give a formula for the result
b) explain in words how you would interpret the result.
c) is the result a vector or scalar?
ii. Given an arbitrary vector function V(x,y,z) (e.g. the velocity of a flowing liquid). Which of
the three operations (div, grad, or curl) can be sensibly operated on V? .For each which can:
a) give a formula for the result
b) explain in words how you would interpret the result.
c) is the result a vector or scalar?
4.Divergence and curl
r
Consider an electric field E = c (Please note the numerator is not rˆ : this is NOT the usual E
r2
r
field from a point charge at the origin, which would give c' , right?!)
r3
a) - Calculate the divergence and the curl of this E field.- Explicitly test your answer for the
divergence by using the divergence theorem. (Is there a delta function at the origin like there was
for a point charge field, or not?)
b) What are the units of c? What charge distribution would you need to produce an E field like
this? Describe it in words as well as formulas. (Is it physically realizable?)
Kassahun Nigatu(Msc),Mesfin Teshome(Msc) and Yitagesu Daba(Msc) 215
Applied Mathematics III
UNIT 5
5. COMPLEX ANALYSIS
Unit Introduction
Complex analysis extends the familiar 'real calculus' to complex calculus by introducing
complex numbers and functions. Problems which are difficult to solve in real calculus may be
solved by the complex analysis in a short period of time..The importance of complex analysis in
Applied mathematics has three main reasons.
1. Two dimensional potential problems can be modeled and solved by method of analytic
functions which we shall see in this unit. This reason is the real and imaginary parts of
analytic functions satisfy Laplace's equations in two variables.
2. Many difficult integrals (real or complex) that appear in applications can be solved by
complex integration.
3. Most functions in engineering mathematics are analytic functions ,and their study as
functions of complex variable leads to a deeper understanding of their properties and to
interrelations in complex that have no analog in real calculus.
Thus, this unit is devoted to the complex analysis and its applications.
5.1 Revision on complex numbers
The transition from real calculus to complex calculus starts with a discussion of complex
numbers and their geometric representation in the complex plane. But Students are familiarized
with most of the algebraic and geometric structure of complex numbers in their pre-university
education. To make the lesson ease and more understandable we shall revise the properties of
complex numbers along with some new concepts which are very important in the subsequent
sections of this chapter.
Definition 5.1. A complex number z is an expression which is written in the form z=x+yi where
x and y are real numbers called the real part, Re z, and imaginary part, Im z, of z respectively and
i=√−1. We can write z also as an ordered pair (x ,y) of real number x and y.
Example 1:For z=4-5i, Re z=4 and Im z=-5. For z=7i, Re z=0 and Im z=7
Definition 5.2
Two complex numbers are equal if and only if their real parts are equal and their imaginary parts
are equal.
Example 2 Let z1=4x-18i and z2=-36+3yi. if z1=z2 , then 4x=-36 and 3y=-18.Thus x=-9 and
y=-6.
5.1.1 Operations on complex numbers
Definition 5.3
If z1 and z2 are two complex numbers such that z1=x1+y1i and z2=x2+y2i, then the sum,
difference and product and division of the two complex numbers are defined as
i. z1+ z2=( x1+x2)+( y1+y2)i
ii. z1- z2=( x1-x2)+( y1-y2)i
iii. z1 z2=(x1x2- y1y2)+( x1y2+ y1x2)i.
𝑧1 x +y i (x +y i)(x−yi) (x1 x2 +y1 y2 𝑥2 𝑦1 −𝑦2 𝑥1
iv. = x1 +y1 i = (x1 +y1 i)(x−yi) = +
𝑧2 2 2 2 2 𝑥22 +𝑦22 𝑥22 +𝑦22
Complex or z plane
We can represent the complex number x+ yi=(x ,y) by a position vector in the xy plane whose
tail is at the origin and its head is at the point (x, y).When the xy plane is used for displaying
complex number ,it is called the complex plane ,or simply ,the z plane(Fig 5.1). Real numbers or
points corresponding to z=x =x+0i=(x, y) are represented by points on the x-axis; hence the x-
axis is called the Real axis. Purely imaginary numbers or points corresponding to z=iy=0+0i =
(0, y) are represented by points on the y-axis, and hence we call the y-axis the Imaginary axis.
b) If z=6=6+0i,then 𝑧̅ = 6 − 0𝑖 = 6
Theorem 5.1
Suppose that z, z1 and z2 are arbitrary complex numbers. Then
i. 𝑧̅̅ = 𝑧 iii. 𝑧1 𝑧2 ̅1 𝑧̅2
̅̅̅̅̅̅=𝑧 v. Re(z)=
𝑧+𝑧̅
2
̅̅̅̅̅̅̅̅̅ (̅̅̅̅
ii. 𝑧1 + 𝑧2 = 𝑧̅1 + 𝑧̅2 𝑧1 ̅̅̅
z1
iv. ) = ̅̅̅ if 𝑧2 ≠ 0. vi. Im(z)=
𝑧−𝑧̅
𝑧2 z2
2𝑖
called an argument of z and is denoted by arg z. The unique value of arg z lying in the range
−𝜋 < 𝜃 ≤ 𝜃 is called the principal argument of z and denoted by Arg z.If Arg z= 𝜃, then
arg z=Arg z + 2n𝝅, n∈ 𝒁 When z is a negative real number , Arg z has a value 𝜋 not − 𝜋.
Fig 5.2
Example1 . Express each of the following numbers in polar form.
a) 1 + 𝑖 b) −1 − 𝑖
𝜋
Solutions. a) r = √1 + 1 = √2. Since z is in first quadrant ,Arg z=tan−1 1 = .Therefore
4
𝜋 𝜋
z=1 + 𝑖=√2 (cos 4 + sin 4 )
This is only one of an infinite number of possibilities for the polar form of 1 +𝑖 (for n=0) :
𝜋 𝜋
1 + 𝑖=√2 (cos( 4 + 2𝑛𝜋) + sin( 4 + 2𝑛𝜋) (n=0, ±1, ±2, − − − )
3𝜋
b) r=√1 + 1 = √2 .Since the number is in third quadrant , Arg z=tan−1 1 = − .Then
4
3𝜋 3𝜋
-1-i = √2 (cos(− 4
) + sin(− 4
).
3𝜋 3𝜋
Note that -1-i = √2 (cos(− + 2𝑛𝜋) + sin(− + 2𝑛𝜋 ) (n=0, ±1, ±2, − − − ).
4 4
The symbol 𝑒 𝑖𝜃 𝑜𝑟 exp(𝑖𝜃) is defined by means of Euler formula as 𝑒 𝑖𝜃 = cos 𝜃 + 𝑖 sin 𝜃 where
𝜃 is to be measured in radian measure.
It enables us to write the polar form 𝑧 = 𝑟(cos 𝜃 + 𝑖 sin 𝜃 )more compactly in exponential form
as 𝑧 = 𝑟𝑒 𝑖𝜃 .
4𝜋
Example 2.For a complex number z=-4+4√3 , we have = 8 𝑎𝑛𝑑 𝐴𝑟𝑔 𝜃 = . Then the
3
4𝜋 𝑖
exponential form of the complex number is 𝑧 = 8𝑒 3 .
Remark : If 𝑧1 = 𝑟1 (cos 𝜃1 + 𝑖 sin 𝜃1 ) and 𝑧2 = 𝑟2 (cos 𝜃2 + 𝑖 sin 𝜃2 ), then
𝑟1 = 𝑟2 𝑎𝑛𝑑 𝜃1 = 𝜃2 + 2𝑛𝜋,n∈ 𝑧.
The polar representation of a complex number gives a simple method to multiply and divide
complex numbers.
THEOREM 5.2
If 𝑧1 = 𝑟1 (cos 𝜃1 + 𝑖 sin 𝜃1 ) and 𝑧2 = 𝑟2 (cos 𝜃2 + 𝑖 sin 𝜃2 ), then
i. 𝑧1 𝑧2= 𝑟1 𝑟2 (cos(𝜃1 + 𝜃2 ) + 𝑖 sin(𝜃1 + 𝜃2 )
𝑧1 𝑟
ii. = 𝑟1 (cos(𝜃1 − 𝜃2 ) + 𝑖 sin(𝜃1 − 𝜃2 )
𝑧2 2
From this we can generalize ,by mathematical induction ,that 𝑧 𝑛 = 𝑟 𝑛 (cos 𝑛𝜃 + 𝑖 sin 𝑛𝜃) for
any integer n .For |𝑧| = 𝑟 = 1 this formula becomes
(𝐜𝐨𝐬 𝜽 + 𝒊 𝐬𝐢𝐧 𝜽)𝒏 = 𝐜𝐨𝐬 𝒏𝜽 + 𝒊 𝐬𝐢𝐧 𝜽 which is called De Moivre's formula,in honor of
French Mathematician Abrham De moivre (1667-1754).
Example . Evaluate (−√3 − 𝑖)30 .
1 5𝜋
Solution . Since r=2 and Arg z=tan−1 =− , then
√3 6
−5𝜋 −5𝜋
(−√3 − 𝑖)30 = 230 (cos(30 ∗ ) + 𝑖 sin (30 ∗ )
6 6
= 230 (cos −25𝜋𝑖 + 𝑖 sin −25𝜋𝑖 = −230
Roots of complex numbers
= ±1, ±𝑖
These values lie the circle of radius 1 and center at the origin.
1
Example 4. Find all values of (−8i)3 .
1 𝜋 𝜋
𝜋 3 − +2𝑘𝜋 − +2𝑘𝜋
Solution . r = 8 and Arg z= − 2 (−8𝑖)3 = √8((cos( 2
)+ i sin( 2
)) (k =0,1,2)
3 3
1
Therefore, (−8𝑖)3 are √3 − 𝑖 , 2𝑖 𝑎𝑛𝑑 − √3 − 𝑖 for k=0,1 and 2 respectively.
Remark if w1 is any nth root of an arbitrary complex number 𝑧 ≠ 0 and w is the value
𝑛
corresponding to k=1, then the n values of √𝑧 are
w1 , w1w , w1w2---, w1wn-1
Quick Exercises .
1. Find all roots of the following complex numbers
3 5 4
a) √−𝑖 b) √−3 c) √1 − 𝑖
Definition 5.8
Let z be a complex number and k be a fixed number. An equation of the form |𝑧 − 𝑘| = 𝜌 is a
circle of radius a and center 𝜌.It is the set of all z whose distance |𝑧 − 𝑘| from the center 𝑘
equals 𝜌 whose figure is given below(Fig 5.3).
Fig 5.3
Example 1.|𝒛 − 𝟒| = 𝟐 is an equation of a circle with radius 2 and center at 4.
Example 2. |𝑧 + 2𝑖| = 1 is an equation of a circle with center at -2i and radius 1.
Example 3. |𝑧 + 1 − 3𝑖| = 5 is an equation of a circle with center at -1-2i = (-1,2) and radius 5.
An open circular disk which is given by |𝑧 − 𝑘| < 𝜌 is the set of all points z lying inside but not
on a circle with center at k.
A closed circular disk which is given by |𝑧 − 𝑘| ≤ 𝜌 is the set of all its interior points plus the
circle itself. An open circular disk |𝑧 − 𝑘| < 𝜌 is also called a neighbourhood of k or a 𝜌-
neighbourhood of k.
Examples 4.The solution set of the inequalities |𝑧| < 3, |𝑧 − 𝑖| < 5 and |𝑧 + 2 + 3𝑖| < 8 are
neighbourhods of the points 0,i and -2-3i with radii 3,5 and 8, respectively
Any set containing a 𝜌-neighbourhood of k is also called neighborhood of k.
The set of all z whose distance |𝑧 − 𝑘| from 𝑘 is greater than 𝜌1 ,but less than 𝜌1 is called an
open annulus(or circular ring)(Fig 5.4)and is given by the inequality 𝜌1 < |𝑧 − 𝑘| < 𝜌2
.Similarly the closed annulus 𝜌1 ≤ |𝑧 − 𝑘| ≤ 𝜌2 (Fig 5.4)includes the two circles.
An open set S is connected if each pair of points z1 and z2 in it can be joined by a polygonal line,
consisting of a finite number of line segments joined end to end, that lies entirely in S. The open
set |z| < 1 is connected. The annulus 1 < |z| < 2 is, of course, open and it is also connected. A
nonempty open set that is connected is called a domain. thus any neighborhood is a domain. An
open square with a diagonal removed is not a domain since it is not connected.
The complement of a set S in the complex plane is the set of all points of the complex plane that
do not belong to S. A set S is called closed if its complement is open. For example, the points on
and inside the unit circle form a closed set (“closed unit disk”) since its complement is open.
A boundary point of a set S is a point every neighborhood of which contains both points that
belong to S and points that do not belong to S. For example, the boundary points of an annulus
are the points on the two bounding circles. Clearly, if a set S is open, then no boundary point
belongs to S; if S is closed, then every boundary point belongs to S. The set of all boundary
points of a set S is called the boundary of S.
A region is a set consisting of a domain plus, perhaps, some or all of its boundary points.
Quick Exercises.
Sketch the following sets and determine and determine which are domains.
1. |𝑧 − 2 + 𝑖| ≤ 1 4. Im z = 1
𝜋
2. |3𝑧 + 4| > 5 5. 0 ≤ arg 𝑧 ≤ 4
3. Im z > 2
5.2 COMPLEX ANALYTIC FUNCTIONS
From the real calculus, a real function f defined on a set S of real numbers (usually an interval)
is a rule that assigns to every x in S a real number f(x), called the value of f at x.
Now in complex, S is a set of complex numbers and a function f defined on S is a rule that
assigns to every z in S a complex number w, called the value of f at z and we use the notation
w=f(z).
Here z varies in S and is called a complex variable. The set S is called the domain of definition
of f or, simply, the domain of f. The set of all values of a function f is called the range of f.
Example1: w=2z4+5z-4 is a complex function defined for all z; that is, its domain S is the whole
complex plane.
z
Example2:f(z)=z2 +4 is a complex valued function defined for all z except at z = ± i.
In general if u(x ,y) and v(x ,y) are real valued function of x & y defined on a region s of the
complex plane then
w=f(z)=u(x ,y)+iv(x ,y)
is a complex valued function defined on S.
Conversely each complex valued function w=f(z) can be written as
w=f(z)=u(x,y)+iv(x,y)
where u and v are the real valued functions and called the real part and the imaginary part of
the function f(z) respectively.
E x a m p l e 1 Find the real and imaginary part of the following function and evaluate the value
of f at the indicated point.
a) W=z2+1 at z=2-3i
Solution. Let z=𝑥 + 𝑖𝑦.Then w=( 𝑥 + 𝑖𝑦)2 +1=x2-y2+1+2xyi so that u=Re f(z)= x2-y2+1 , v=Im
f(z)=2xy and f(2-3i)=(2-3i)2+1=-4-12i.From this we can see that u(2,-3)=4 and v(2,-3)=-12
b) w=f(z)=4z-iz at z=2-i
Solution. f(z)=4(𝑥 + 𝑖𝑦)-i(𝑥 + 𝑖𝑦)=4x+y+(4y-x)i yields u(x,y)=4x+y and v(x,y)=4y-x
Also f(2-i)=4(2-i)-i(2-i)=8-4i-2i-1=7-6i
Remark. If the polar coordinates r and θ, instead of x and y, are used, then u + iv = f (𝑟𝑒 𝑖𝜃 )
where w = u + iv and z = reiθ In that case, we may write
f (z) = u(r, θ) + iv(r, θ).
Example 2. If f (z) = z2, then
f (𝑥 + 𝑖𝑦) = (𝑥 + 𝑖𝑦)2 = x2 − y2 + i2xy.
Hence
u(x, y) = x2− y2 and v(x, y) = 2xy.
When polar coordinates are used,
f(𝑟𝑒 𝑖𝜃 ) = 𝑟 2 𝑒 2𝑖𝜃 = 𝑟 2 (cos2 𝜃 + 𝑖 sin2 𝜃) so that u(x ,y)= 𝑟 2 cos 2𝜃 and v(x ,y)= 𝑟 2 sin 2𝜃
Group Activity
1. For each of the functions below, describe the domain.
z−4 z
a) f(z)= z2 −5z+6 ; c) f(z)=z+z̅
1 1
b) f(z)= Arg ( z) d) f(z)=. 1−|z|2
2. Find the real and imaginary part of the following function and evaluate the value of f at the
indicated points.
a) f(z)=4z2-5z+3 at z=4-5i c) f(z)=𝑧̅ 𝑅𝑒(𝑧) − 4𝑧 + 2𝐼𝑚(𝑧) at 𝑧 = −𝑖
2z 1
b) 𝑓(𝑧) = 3−z at z= 3 -2i d) f(z)=𝑧̅ 2 − (−4 − 2𝑖)𝑧 at z=1+i
3. Write the following function in the form f(z)= u(r, θ)+iv(r, θ).
1
a) f(z)= z+z (z≠ 0) b) f(z)=𝒛𝟑 + 𝒛−𝟑
a) f(0) b) f(𝜋3 𝑖) c) 𝜋
f(3 + 2 𝑖)
a) f(1-i) b) f(√3 + 𝑖)
5.2.1 Limit and Continuity
Definition 5.9: A function w=f(z) is said to have a limit l as z tends to z0 if given any ε > 0
∃ δ > 0 ∋ |f(z) − l| < ε whenever |z − z0 | < δ for all z ≠ z0 (Fig 5.5).In this case we write
limz→z0 f(z) = l.
Fig 5.5.
Geometrically it means that given any open disk with center l and radius ε there exist an open
disc with center z0 and radius δ for every point z0 in the disc |z − z0 | < δ
Formally, this definition is similar to that in calculus, but there is a big difference. In the real
case, x can approach an x0 only along the real line, whereas in the complex calculus z may
approach z0 from any direction in the complex plane.
Remark: If limz→z0 f(z) exists then it is unique.
Now let us state the following theorems on the sum, product and quotient of two complex
functions whose proof is analogous to the proof of real functions.
Theorem 5.3. Let f and g be two functions such that limz→z0 f(z)=l and limz→z0 g(z) = m.then
I. limz→z0 f(z) ± g(z) = l ± m f(z) l
III. limz→z0 g(z) = m provided m≠ 0
II. limz→z0 f(z) g(z) = lm
It is easy to see from definition (5.9) of limit that
limz→z0 c=c and limz→z0 z = z0
where z0 and c are any complex numbers; and, by mathematical induction, it follows that
limz→z0 z n = z0n (n = 1, 2, . . .)
So, the limit of a polynomial
P(z) = a0 + a1z + a2z2 + ·· ·+anzn
as z approaches a point z0 is the value of the polynomial at that point: that is
lim p(z) = p(z0 )
z→z0
p(z) p(z )
If p(z) and q(z) are polynomials q(z0)≠ 0 then limz→z0 q(z) = q(z0). If both p(z)=0 and q(z)=0,
0
then p and q can be factored as p(z)=(z-z0)p1(z) and q(z)= (z-z0)q1(z).If q1(z0) ≠ 0, then
𝑝(𝑧) 𝑝1 (𝑧0 )
lim =
𝑧→𝑧0 𝑞(𝑧) 𝑞1 (𝑧0 )
Example 1.Evaluate the following limits
a) limz→i 2z 3 − 4z + 1 b) limz→1+2i
z2 +6z−3
c) limz→i
z2 +1
3z−2 z2 −3iz−2
z2 +1 (z−i)(z+i) (z+i)
c) limz→i = limz→i = limz→i = −2
z2 −3iz−2 (z−i)(z−2i) (z−2i)
Quick Exercise.
Evaluate the following limits
1.limz→2−3i 2z 2 − 5z + 3 2.limz→−2i
z3 −3z2 +1
3.limz→1+i
z2 −2i
z4 +z−2 z2 −2z+2
CONTINUITY
Definition 5.10 A function f is continuous at a point z0 if all three of the following conditions
are satisfied:
1.limz→z0 exists
(2) f (z0) is defined
(3) limz→z0= f (z0)
A function of a complex variable is said to be continuous in a region R if it is continuous at each
point in R.
Theorem 5.4 . Suppose that f and g are continuous at z0. Then the the sum f+g, the difference f-
𝑓
g, the product fg, the quotient provided that g(z0)≠ 0 and the composition fog, where
𝑔
5.2.2 Derivative
Definition 5.11 The derivative of a complex function f at a point 𝑧0 is written f ′ (z0 ) and is
defined by
𝑓(𝑧0 +∆𝑧)−𝑓(𝑧0 )
𝑓 ′ (𝑧0 ) = lim∆𝑧→0 (1)
∆z
provided this limit exists. Then f is said to be differentiable at z0 . If we write ∆z = z − z0 ,
we have z = z0 +∆z and (1) takes the form
𝑓(𝑧)−𝑓(𝑧0 )
𝑓 ′ (𝑧0 ) = lim𝑧→𝑧0 (2)
𝑧−𝑧0
𝑑𝑤 ∆𝑤
If we let w=f(z) and ∆𝑤 = 𝑓(𝑧) − 𝑓(𝑧0 ), then we can use the Leibniz notation = lim∆𝑧→0
𝑑𝑧 ∆𝑧
Differentiability at z0 means that, along whatever path z approaches , the quotient in (2) always
approaches a certain value and all these values are equal.
Example3
The function f(z)= z 2 is differentiable for all z and has the derivative 𝑓 ′ (z) = 2z because
If ∆𝑦 = 0,this is 1.If ∆𝑥 = 0,this is -1.Thus (****) approaches 1 along path I in Fig and -1
along path II. Hence , by definition, the limit of **** as ∆z approaches to zero does not exist at
any z. Therefore, f(z)= 𝑧̅ is not differentiable at z, and since z is arbitrary, f(z) is nowhere
differentiable.
Fig 5.6
Quick Exercise
1. Show that 𝑓 ′ (𝑧) = 4𝑧 3 if f(z)=𝑧 4 .
2. Show that f(z)=|𝑧|2 is differentiable only at z=0.
3. Show that 𝑓 ′ (𝑧) does not exist at any when
a) f(z)= Re z b) f(z)=Im z
It is also easy to show that, by definition of limit of complex function,
𝐝 𝐝 𝐝
𝐜=𝟎, 𝐳 = 𝟏 𝐚𝐧𝐝 (𝐜𝐟(𝐳) = 𝐜𝐟 ′ (𝐳)
𝐝𝐳 𝐝𝐳 𝐝𝐳
Also, if n is a positive integer,
d n
z = nz n−1
dz
This formula remains valid when n is a negative integer, provided that z ≠ 0.
Theorem: Differentiation Formula
If the derivatives of two functions f and g exist at a point z, then
d
i. (f(z) + g(z)) = f ′ (z) + g ′ (z)
dz
𝑑
ii. (𝑓(𝑧)𝑔(𝑧)) = 𝑓 ′ (𝑧) 𝑔(𝑧) + 𝑓 (𝑧) 𝑔′ (𝑧)
𝑑𝑧
is called a rational function. This f is analytic except at the points where h(z)= 0:
Here we assume that common factors of g and h have been canceled.
Example 2.f (z) = 1/z is analytic at each nonzero point in the finite plane.
Example 3.The function f (z) = |z|2 is not analytic at any point since its derivative exists only at
z = 0 and not throughout any neighborhood which we have seen in the previous section.
Definition 5.14: A point 𝑧0 is said to be singular or singularity of f if the function f fails to be
analytic at 𝑧0 .
𝑧−5 𝑧−5
Example 4.f(z)=𝑧 2 −3𝑧+2 ==(𝑧−2)(𝑧−1) is not analytic at z=1 and z=2 and hence z=1 and z=2 are
the singularities of f.
Example 5.f(z)=|𝑧|2 has no singular point since f is nowhere analytic.
Theorem (L'HOPITAL rule)
Assume that f and g are analytic at 𝑧0 . If f(𝑧0 ) = 0 , g(𝑧0 ) = 0 and 𝑔′ (𝑧0 ) ≠ 0, then
𝑓(𝑧 ) 𝑓 ′ (𝑧 )
lim𝑧→𝑧0 𝑔(𝑧0 )= lim𝑧→𝑧0 𝑔′ (𝑧0 )
0 0
Solution
Let f(z)= 𝑧 6 + 1 and g(z)= 𝑧 2 + 1 .Thus 𝑓 ′ (𝑧) = 6𝑧 5 and 𝑔′ (𝑧) = 2𝑧.Since f(i)= g(i)=0 and
𝑔′ (𝑖) = 2𝑖 ≠ 0, then by the L'HOPITAL rule,
𝑧 6 +1 6𝑧 5 −6𝑖
lim𝑧→𝑖 𝑧 2 +1= lim𝑧→𝑖 = = −3
2𝑧 2𝑖
Quick exercises
1. Find all the singularities of the following functions
𝑧 𝑧+6 𝑧+6
a) f(z)=𝑧 3 −8 b) f(z)=𝑧 2 (𝑧 2 +9) c) f(z)=𝑧 3 −𝑧 2 +4𝑧−4
[𝑢(𝑥+∆𝑥,𝑦+∆𝑦)+𝑖𝑣((𝑥+∆𝑥,𝑦+∆𝑦)]−[𝑢(𝑥,𝑦)−𝑖𝑣(𝑥,𝑦)]
𝑓 ′ (𝑧) = lim∆𝑧→0 (3)
∆𝑥+𝑖∆𝑦
We first choose path I in Fig 5.6. Thus we let first ∆𝑦 → 0 and then ∆𝑥 → 0. After ∆𝑦 = 0is
zero, ∆𝑧 = ∆𝑥 . Then (3) becomes, if we first write the two u-terms and then the two v-terms,
𝑢(𝑥+∆𝑥,𝑦)−𝑢(𝑥,𝑦) 𝑣(𝑥+∆𝑥,𝑦)−𝑣(𝑥,𝑦)
𝑓 ′ (𝑧) = lim∆𝑥→0 + i lim∆𝑥→0
∆𝑥 ∆𝑥
Since 𝑓 ′ (𝑧) exists, the limits on the right exist and give the partial derivatives of u and v with
1
respect to y; noting that 𝑖 = −𝑖. we thus obtain
Thus
ux = 2x =vy , uy = −2y =−vx ,
EXAMPLE 2. Is 𝑧̅analytic?
Solution. Since 𝑧̅=𝑥 − 𝑖𝑦,we have u=x and v=-y .Then
𝑢𝑥 = 1 , 𝑢𝑦 = 0, 𝑣𝑥 = 0 & 𝑣𝑦 = −1 𝑤ℎ𝑖𝑐ℎ 𝑠ℎ𝑜𝑤𝑠 𝑡ℎ𝑎𝑡 𝑢𝑦 = −𝑣𝑥 𝑏𝑢𝑡 𝑢𝑥 ≠ 𝑣𝑦.
∴f(z)=𝑧̅ 𝑖𝑠 𝑛𝑜𝑡 𝑎𝑛𝑎𝑙𝑦𝑡𝑖𝑐
The Cauchy–Riemann equations are fundamental because they are not only necessary but also
sufficient for a function to be analytic. More precisely, the following theorem holds.
Theorem 5.6 Cauchy–Riemann Equations
If two real-valued continuous functions u(x, y)and v(x, y) of two real variables x and y have
continuous first partial derivatives that satisfy the Cauchy–Riemann equations in some domain
D, then the complex function f(z)=u(x, y)+iv(x, y) is analytic in D and 𝑓 ′ (z) = 𝑢𝑥 + 𝑖𝑣𝑥 in D.
Using Theorems 5.5 and 5.6 we can now easily find out whether or not a given complex
function is analytic or not.
Example 3. Is f(z)=𝑒 −2𝑥 (𝑐𝑜𝑠 2𝑦 − 𝑖𝑠𝑖𝑛 2𝑦) anyltic ?
Solution. We have u=e−2x cos 2y and v=− e−2x sin 2y and by differentiation
𝑢𝑥 = −2 e−2x cos 2y 𝑣𝑥 = 2 e−2x sin 2y
𝑢𝑦 = −2 e−2x sin 2y 𝑣𝑦 = −2 e−2x cos 2y
Since the Cauchy Riemann equations are satisfied, that is 𝑢𝑥 = 𝑣𝑦 𝑎𝑛𝑑 𝑢𝑦 = −𝑣𝑥 , we
conclude that f(z) is analytic for all z and
𝑓 ′ (z) = 𝑢𝑥 + 𝑖𝑣𝑥 =−2 e−2x cos 2y + 2 e−2x sin 2y
Quick exercises :Are the following functions analytic ?If so, find 𝑓 ′ (z).
1. f(z) = cos x cosh y − i sin x sinh y.
2. f (z) = 2x + ix𝑦 2
3. 𝑓(𝑧) = 𝑥 3 − 3𝑥𝑦 2 + 𝑖(3𝑥 2 𝑦 − 𝑦 3 )
4. 𝑓(𝑧) = 𝑒 −𝑦 cos 𝑥 − +𝑖𝑒 −𝑦 sin 𝑥
Remark. If we use the polar form z=r(cos𝜃 + 𝑖𝑠𝑖𝑛𝜃)and set f(z)=𝑢(𝑟, 𝜃) + 𝑖𝑣(𝑟, 𝜃), then the
Cauchy–Riemann equations are
1 1
ur = vθ , vr = − uθ (r > 0).
r r
1
and f ′ (z) = f ′ (reiθ ) = r e−iθ (vθ − uθ ).
Example 1.
1 1 1 1
𝑓(𝑧) = = 𝑖𝜃
= 𝑒 −𝑖𝜃 = (𝑐𝑜𝑠𝜃 − 𝑠𝑖𝑛𝜃) (𝑧 ≠ 0)
𝑧 𝑟𝑒 𝑟 𝑟
1 1
from which u(r,𝜃) = 𝑐𝑜𝑠𝜃 𝑎𝑛𝑑 𝑣(𝑟, 𝜃) = − 𝑟 𝑠𝑖𝑛𝜃 and
𝑟
𝑢𝑟 = cos 𝜃 , 𝑣𝑟 = − sin 𝜃 ,
1 1
𝑢𝜃 = − 𝑟 𝑠𝑖𝑛𝜃, 𝑣𝜃 = − 𝑟 𝑐𝑜𝑠𝜃 .
𝟏 𝟏
Since the Cauchy-Riemann equations 𝐮𝐫 = 𝐫 𝐯𝛉 𝐯𝐫 = − 𝐫 𝐮𝛉 are satisfied, then f is analytic at
Because comparing 𝑣𝑥 = 3𝑥𝑦 2 + +ℎ′ (x) and -𝑢𝑦 = −𝑥 3 + 3𝑦 2 𝑥 where -𝑣𝑥 =-𝑢𝑦 then it is seen
that ℎ′ (x) =−𝑥 3 .
𝑥4 3 1 𝑥4
Therefore h(x)= − + 𝐶 giving v(x,y)=2 𝑥 2 𝑦 2 − 4 𝑦 4 + − + 𝐶 and the corresponding
4 4
analytic function is
3 1 𝑥4
f(z)= 𝑥 3 𝑦 − 𝑦 3 𝑥 + 𝑖 (2 𝑥 2 𝑦 2 − 4 𝑦 4 + − + 𝐶).
4
Quick Exercises.
Are the following functions harmonic? If your answer is yes, find a corresponding analytic
function f(z)=u(x ,y)+iv(x ,y)
1. u=𝑥 3 − 3𝑥𝑦 2 2. u=𝑒 𝑥 sin 2𝑦
Group Exercise
1. Determine whether the following functions(1-11) are Harmonic or not? If so, find a
corresponding analytic function f(z)=u +iv.
1. u=𝑥 3 − 3𝑥 2 7. v=(2x+1)y
𝑥
2. u=𝑥 2 +𝑦 2 8. u=x cos 3x cosh 3y + y sin 3x sinh 3y
9. 𝑢 = 𝑥 3 − 3𝑥𝑦 2 + 2𝑥 + 𝑦
3. v=𝑒 𝑥 sin 2𝑦
10. u=𝑒 𝑥 (𝑥 cos 𝑦 + 𝑦 𝑠𝑖𝑛𝑦)
4. u= sinx coshy
−(𝑥2 −𝑦2 )
5. u=𝑒 −𝑥 cos 𝑦 11. 𝑒 2 cos 𝑥𝑦
6. u=𝑥 2 − 2𝑥 − 𝑦 2
2. Find the values of the constants a, b and c that will guarantee that the function f(z)=𝑎𝑥 2 +
𝑏𝑥𝑦 + 𝑐𝑦 2 is Harmonic?
3. Determine the values of a and b so that the following functions are Harmonic and find a
Harmonic conjugate
Definition 5.15 The exponential function of complex analysis is defined for all z by the function
f(z)=𝑒 𝑧 = 𝑒 𝑥+𝑖𝑦 = 𝑒 𝑥 (𝑐𝑜𝑠𝑦 + 𝑖𝑠𝑖𝑛𝑦) (*)
when y is taken to be in radian measure. For convenience we use the notation exp 𝑧 instead of
𝑒 𝑧.
Example1: Show that f(z)=𝑒 𝑧 is analytic for all z and 𝑓 ′ (𝑧) = 𝑒 𝑧 using Cachy-Riemann
equations.
Solution. From f(z)= 𝑒 𝑥 (𝑐𝑜𝑠𝑦 + 𝑖 𝑠𝑖𝑛𝑦) we have
u(x, y)= 𝑒 𝑥 𝑐𝑜𝑠𝑦 ,v(x, y)= 𝑒 𝑥 𝑠𝑖𝑛𝑦; and by differentiation
𝑢𝑥 = 𝑒 𝑥 𝑐𝑜𝑠𝑦 , 𝑣𝑥 = 𝑒 𝑥 𝑠𝑖𝑛𝑦
𝑢𝑦 = −𝑒 𝑥 𝑠𝑖𝑛𝑦, 𝑣𝑦 = 𝑒 𝑥 𝑐𝑜𝑠𝑦.
Since 𝑢𝑥 = 𝑣𝑦 and 𝑢𝑦 = −𝑣𝑥 then f(z)= 𝑒 𝑧 is analytic for all z by the Cachy-Rieman equations
and 𝑓 ′ (𝑧) = 𝑢𝑥 + 𝑖𝑣𝑥 = 𝑒 𝑥 𝑐𝑜𝑠𝑦 + 𝑒 𝑥 𝑠𝑖𝑛𝑦
= 𝑒 𝑧 ∀𝑧.
Therefore 𝑒 𝑧 is an entire function
If z is a pure imaginary number ,that is z= iy, then 𝑒 𝑖𝑦 = 𝑐𝑜𝑠𝑦 + 𝑖 𝑠𝑖𝑛𝑦 which is called a Euler
formula.
Example 1. 𝑒 2𝜋𝑖 = 𝑐𝑜𝑠2𝜋 + 𝑖𝑠𝑖𝑛2𝜋 = 1 + 0 = 0
Recall that the polar form of a complex number z is z=r(cos 𝜃 + 𝑖 sin 𝜃) and can be written more
compactly as an exponential form z=r𝑒 𝑖𝜃 .
Example 2 . Write the exponential form of z=1+i.
𝜋
𝜋
Solution. r= √2 and 𝜃 = 4
and z=√2 𝑒 𝑖 4 .
Remarks.
1. 𝑒 𝑧 is a periodic function with a period of 2𝜋𝑖 since f(z+2𝜋𝑖)= 𝑒 𝑧+2𝜋𝑖 = 𝑒 𝑧 =f(z).This
property is not shared by the real fuctio f(x)=x.
2. In the real case, f(x)=ex is never negative whereas in the complex case ez can be
negative.
Example 3. Solve ez = -1
Solution . ez= ex( cos y + i sin y)= -1+ 0i and 𝑠𝑖𝑛𝑐𝑒 |𝑒 𝑧 | = 𝑒 𝑥 = 1 𝑡ℎ𝑒𝑛 𝑥 = 0. As ex=1,
ex cos y= -1 and ex sin y=0 ,then x=0 and y=(2𝑛 + 1)𝜋, 𝑛 = 0, ±1, ±2, − − −.
Therefore z=(2𝑛 + 1)𝜋, 𝑛 = 0, ±1, ±2, − − −.
Example .𝑒 𝑧 = 1 + 𝑖
𝜋𝑖
𝜋
Solution . 𝑒 𝑥+𝑖𝑦 =𝑒 𝑥 𝑒 𝑦𝑖 = √2𝑒 4 implies that 𝑒 𝑥 = √2 and y= 4 + 2𝑛𝜋 𝑛 = 0, ±1, ±2, − − −.
Quick Exercises.
1. Write an exponential form of
a) z=1-i b) z=3-4i
2. Solve 𝑒 𝑧 = 2 + 3𝑖
Group Exercise.
1. Show that
a) exp(4+𝜋𝑖)=-𝑒 4 b) exp(𝜋𝑖 + 𝑧) = −𝑒𝑥𝑝𝑧
2. Find the real and imaginary part of
𝜋𝑧
a) exp(− ) c) exp(𝑧 2 )
3
1 d) exp(𝑧 3 )
b) exp(𝑧)
̅̅̅̅̅̅̅̅̅̅ = 𝑒𝑥𝑝(𝑖𝑧
4. Show that 𝑒𝑥𝑝(𝑖𝑧) ̅ ) if and only if z=n𝜋 (𝑛 = 0, ±1, ±2,---.
5. Show that if ez is real ,then Im z= n𝜋 (𝑛 = 0, ±1, ±2,---.
1
6. Write Re(𝑒 𝑧 ) interms of x and y.
7. Show that f(z)=exp𝑧̅ is not analytic anywhere ,using the Cauchy-Riemann equations.
Trigonometric function.
From Euler's formula 𝑒 𝑖𝑧 = cos 𝑧 + 𝑖 sin 𝑧 and 𝑒 −𝑖𝑧 = cos 𝑧 − 𝑖 sin 𝑧 for all z. Adding and
subtracting these equations we obtain the following definition.
Definition 5.16. The sine and cosine functions of a complex variable z is defined by as
1 1
sin z=2𝑖 (𝑒 𝑖𝑧 − 𝑒 −𝑖𝑧 ) and cos z=2 (𝑒 𝑖𝑧 + 𝑒 −𝑖𝑧 )
The other trigonometric functions tan z, cot z , sec z and csc z can be defined in terms of sin z
and cos z as of the real case. and moreover
Example 1. Show that sin z=sin x coshy +icos xsinh y .
1
Solution . sin z=2𝑖 (𝑒 𝑖𝑧 − 𝑒 −𝑖𝑧 )= sin z
1
=2𝑖 (𝑒 𝑖(𝑥+𝑖𝑦) − 𝑒 −𝑖(𝑥+𝑖𝑦) )
1
= (𝑒 −𝑦+𝑖𝑥 − 𝑒 𝑦−𝑖𝑥 )
2𝑖
1
= (𝑒 −𝑦 (cos 𝑥 + 𝑖𝑠𝑖𝑛 𝑥) − 𝑒 𝑦 (cos 𝑥 − 𝑖𝑠𝑖𝑛 𝑥)
2𝑖
𝑒 𝑦 +𝑒 −𝑦 𝑒 𝑦 −𝑒 −𝑦
= sin x( ) + 𝑖 cos 𝑥( )
2 2
Note that Re(sin z)= sin x cosh y and Im(sin z)= cos x sinh y.
Similarly, we can show that cos z= cosx coh y - isin x sinh y.
Here are some properties of the trigonometric functions.
sin (z1+z2)= sin z1cos z2+cos z1sin z2 ∀𝑧 sin2z +cos2z=1 ∀𝑧.
cos (z1+z2)= cos z1cos z2+sin z1sin z2 ∀𝑧 sin 2z=2sin z cos z ∀𝑧
sin -z= -sin z ∀𝑧. cos 2z=cos2z -sin2z ∀𝑧
cos -z=cos z ∀𝑧.
Definition 5.17 A solution to the equation f(z)=0 is called a zero of the function f.
Example 2. Solve cos z=0.
Kassahun Nigatu(Msc),Mesfin Teshome(Msc) and Yitagesu Daba(Msc) 240
Applied Mathematics III
Similarly, we can show that sin z=0 iff z=n 𝜋 for any integer n
Example 3 . Solve sin z=2
Solution . sin z=sin x cosh y +icos x sinh y=2+0i which implies sin x cosh y=2 and
𝜋
cos x sinh y=0.If cosx sinh y=0,then x= 2 +n𝜋 for any integer z or y=0. If y=0,the first equation
𝜋
sin x cosh y= sin x cosh 0=sin x=2 which is impossible because sin x ≤ 1 for all x. If x= 2 +n𝜋
for any integer z, then sin x=±1 and the first equation becomes
cosh y=±2. Since cosh y≥ 1,then cosh y=2 from which y=cosh-1 2.
𝜋
Therefore, z= 2 +n𝜋 + 𝑖 cosh-1 2
HYPERBOLIC FUNCTIONS
Definition 5.18 .The hyperbolic sine hyperbolic cosine functions of a complex variable z are
defined as
𝑒 𝑧 −𝑒 −𝑧 𝑒 𝑧 + 𝑒 −𝑧
sinh z = and cosh z = .
2 2
The other hyperbolic functions tanh z, sech z, csch z and coth z can be defined in terms of sinh z
and cosh z as of the real case.
Since ez and e-z are entire functions ,then sinh z and cosh z are entire functions. Furthermore,
𝑑 𝑑
sinh 𝑧 = cosh 𝑧 𝑎𝑛𝑑 cosh 𝑧 = sinh 𝑧
𝑑𝑧 𝑑𝑧
The derivatives of the remaining hyperbolic functions follow the same rule as in real calculus:
𝑑 𝑑
tanh 𝑧 = 𝑠𝑒𝑐ℎ2 𝑧 and coth 𝑧 = −𝑐𝑠𝑐ℎ2 𝑧
𝑑𝑧 𝑑𝑧
𝑑 𝑑
sech 𝑧 = − sech 𝑧 tanh 𝑧 and 𝑑𝑧 csch 𝑧 = − csch 𝑧 coth 𝑧
𝑑𝑧
Exercises
1. Prove that Im cos z and Re sin z are Harmonic.
2. Write the following in terms of u+ iv
a) sin 2𝜋𝑖 d) cosh(2+3i)
𝜋
b) cos i e) sin(𝜋 +
c) sin(1-3i)
3. Show that sinz is odd and cosz is even function.
4. Solve the following equations.
a) sin z=5 c) sinh z=0 e) sinh z=i
b) cosh z=0 d) cosh z=-1 1
f) cosh z=2
Finally we try to introduce a complex logarithm which is somewhat complicated compared to the
real logarithm.
Definition 5.19. The natural logarithm of z=x+iy denoted by lnz (or sometimes by log z) and is
defined as the inverse of the exponential function, that is, w=lnz is defined for z≠ 0 by the
equation ew=z.
If we write z=rei𝜃 , r > 0, then lnz=lnr+i𝜃 ,r=|𝑧| > 0, 𝜃 = arg 𝑧.Since arg 𝑧 is integer multiples
of 2𝜋,then ln z is a multiple valued function with infinitely many values.
The principal value of ln z denoted by Ln z is defined by
Ln z=ln|𝑧|+i Argz where −𝜋 < 𝜃 ≤ 𝜋.
The function Ln z is single-valued function as Argz is unique.
If 𝜃 is the particular value of arg z such that −𝜋 < 𝜃 ≤ 𝜋, then
ln z= ln|𝑧| + 𝑖(𝜃 + 2𝑛𝜋), 𝑛 𝑖𝑠 𝑎𝑛𝑦 𝑖𝑛𝑡𝑒𝑔𝑒𝑟.If z is positive real ,then Arg z=0 and Ln z becomes
identical with the real natural logarithm of z. If z is negative real,then
Arg z= 𝜋 and Ln z= ln|𝑧|+i 𝜋
Examples
1. ln1=0,±2𝜋𝑖, ±4𝜋𝑖, − − − 𝐿𝑛1 = 0
2. ln-1=±𝜋𝑖, 3𝜋𝑖, ±𝜋𝑖 𝐿n-1= 𝜋𝑖
3. ln-4=1.386294±(2𝑛 + 1)𝜋𝑖 𝑓𝑜𝑟 𝑎𝑛𝑦 𝑖𝑛𝑡𝑒𝑔𝑒𝑟 𝑛 Ln-4= ln-4=1.386294+𝜋𝑖
𝜋𝑖 3𝜋 5𝜋𝑖 𝜋𝑖
4. lni= 2 , , ,--- Lni= 2
2 2
𝐫
𝐟 ′ (𝐳) = 𝐳(𝑢𝑟 + 𝑖𝑣𝑟 )(𝑤ℎ𝑦)
𝐫 1 1
=𝐳( + 𝑖0) =
𝑟 𝑧
𝑑 1
∴ 𝑙𝑛𝑧 =
𝑑𝑧 𝑧
General power
Definition 5.19. The General powers of a complex number z=x+iy are defined by the formula
𝑧 𝑐 = 𝑒 𝑐𝑙𝑛𝑧 =𝑒 𝑐𝑙𝑛|𝑧|+𝑖(𝜃+2𝑛𝜋) where where c is complex,z≠ 0 and any integer n.
𝑧 𝑐 is multivalued function(why).The particular function
𝑧 𝑐 = 𝑒 𝑐𝐿𝑛𝑧
is called the principal value of 𝑧 𝑐 .
𝜋 𝜋 𝜋
Example 1: 𝑖 𝑖 = 𝑒 𝑖𝑙𝑛𝑖 = 𝑒 𝑖(2 𝑖±2𝑛𝜋𝑖) 𝑒 − 2 ±2𝑛𝜋) and the principal value (n=0) is 𝑒 − 2
1 1 𝜋 1 𝜋 𝜋𝑖 𝑛𝜋
1⁄ (𝑙𝑛|𝑖|)+𝑖( +2𝑛𝜋)= ( 𝑖( +2𝑛𝜋)=
Example 2: 𝑖 2 = 𝑒 (2𝑙𝑛𝑖) = 𝑒 (2 2 𝑒 2 2 𝑒4𝑒 for k∈ 𝑍.Since 𝑒 𝑛𝜋 = ±1,
1⁄ 𝜋𝑖 𝑛𝜋 1+𝑖
𝑖 2 = ±1𝑒 4 𝑒 = ±1 ( ).
√2
1+𝑖
The principal value is when n=0.
√2
Quick exercise:
Find the principal value of
1. (1 − 𝑖)1+𝑖 2. (1 + 𝑖)2−𝑖
The inverse trigonometric and Hyperbolic functions.
Definition 5.20. The inverse sine w=arcsin 𝑧 is the relation that sinw=z.
Example 1. Show that arcsin 𝑧 =-i ln(𝑖𝑧 ± √1 − 𝑧 2 ))
𝑒 𝑖𝑤 −𝑒 −𝑖𝑤
Solution . w=arcsin 𝑧 implies that z=sin w= from which
2𝑖
1 𝑖+𝑧
3. arccosh 𝑧 = ln(√𝑧 ± √1 − 𝑧 2 ) 5. arctanh 𝑧 =2 ln(𝑖−𝑧)
4. arcsinh 𝑧 = ln(√𝑧 + √1 + 𝑧 2 )
5.3 Complex Integration
The previous two sections establish the basic or fundamental for the study of complex analysis
covered complex numbers, limits and differentiation and introduce the most important concept of
analyticity. Now we shall consider the next part of complex analysis, complex integration
First, we shall show how to evaluate integrals of a complex functions by defining the integral of
a complex valued function of a real variable
Definition 5.21. Let f(t)=u(t) + iv(t) where u and v are real-valued function of the real variable t
for a≤ 𝑡 ≤ 𝑏.Then
𝑏 𝑏 𝑏
Solution.
1. Let u= 5𝑡 4 + 1 and v= -4t so that
2 2 2
4 4
∫ (5𝑡 − 4𝑖𝑡 + 1) 𝑑𝑡 = ∫ (5𝑡 + 1)𝑑𝑡 + 𝑖 ∫ −4𝑡𝑑𝑡
1 1 1
Quick exercises
Evaluate the following integrals
𝝅 𝟐 𝜋
1. ∫𝟎𝟒 𝒕𝒆𝒊𝒕 𝒅𝒕 2. ∫−𝟏(𝟒𝒕 + 𝟐𝒊)𝟐 𝒅𝒕 3.∫0 𝑒 (𝑡+𝑖𝑡) 𝑑𝑡
The following are some properties of complex integrals which can easily be verified.
Let f(t)=u(t)+iv(t) and g(t)=r(t)+is(t) be two continuous functions on[a,b].Then
𝑏 𝑏 𝑏
1. ∫𝑎 (𝑓(𝑡) + 𝑔(𝑡))𝑑𝑡 = ∫𝑎 𝑓(𝑡)𝑑𝑡 + ∫𝑎 𝑔(𝑡)𝑑𝑡
𝑏 𝑐 𝑏
2. ∫𝑎 𝑓(𝑡)𝑑𝑡 = ∫𝑎 𝑓(𝑡)𝑑𝑡 + ∫𝑐 𝑓(𝑡)𝑑𝑡 where a≤ 𝑐 ≤ 𝑏
𝑏 𝑏
3. ∫𝑎 𝑐𝑓(𝑡)𝑑𝑡 = 𝑐 ∫𝑎 𝑓(𝑡)𝑑𝑡 where c is a complex consant.
𝑏 𝑎
4. ∫𝑎 𝑓(𝑡)𝑑𝑡 = − ∫𝑏 𝑓(𝑡)𝑑𝑡
Now ,we shall see the integral of complex 𝑣𝑎𝑙𝑢𝑒𝑑 𝑓𝑢𝑛𝑐𝑡𝑖𝑜𝑛𝑠 which are defined on curves in the
complex plane called contour integral or line integral.
5.3.1 Contour integral: Line integral in the complex plane
If z moves along the curve C in the z-plane(complex plane) and at each position z has associated
with it a function of z ,i.e. f(z),then summing up f(z) for all such points between A & B(Fig 5.7)
means that we are evaluating a line integral in the z-plane between A(z=z1) & B(z=z2) along the
Fig 5.7
The evaluation of line integral in the complex plane is known as contour integral or complex
point coincides with initial point).Here the integrand f(z) is integrated over a given curve C,
called the path of integration. We may represent a curve C by a parametric representation
z(t)=x(t)+iy(t) ,a≤ 𝑡 ≤ 𝑏
The sense of increasing t is called the positive sense on C. We assume C to be a piecewise
smooth curve, that is they consist of finitely many smooth curves joined end to end.
Basic properties of line integrals
1. (c f ( z) c g ( z))dz c
C
1 2 1
C
f ( z )dz c2 g ( z )dz (Linearity)
C
2.
C
f ( z )dz f ( z )dz (Sense reversal)
C
3.
C
f ( z )dz f ( z )dz f ( z )dz where C=𝐶1 ∪ 𝐶2
C1 C2
( Partition of path)
Complex integral is rich in methods for evaluating integrals. We shall discuss the first two in this
sections and other follows in the section 5.4.
Before we state the first theorem let's define a terminology called simple connected domain
which is very important in evaluating integrals of complex functions.
Definition 5.22. A domain D is called simply connected if every simple closed curve(closed
curve without self intersections) encloses only points of D.
For example , a circular disk is simply connected ,whereas an annulus is not simply
connected(why?).
First method
Integration by the use of the path
Now we shall see the second method of evaluating complex integral that is not restricted to
analytic function but applied to any continuous complex function.
Theorem 5.10 Let C be a piecewise differentiable curve given by the equation z=z(t),where
𝑎 ≤ 𝑡 ≤ 𝑏.Let f(z) be a continuous complex valued function defined in a region containing
C.Then
b
C
f ( z )dz f ( z (t )) z ' (t )dt
a
Proof. Exercise
Steps in Applying Theorem 5.10
(A) Represent the path C in the form z(t) (a≤ 𝑡 ≤ 𝑏)
𝑑𝑧
(B) Calculate the derivative 𝑑𝑡
(C) Substitute z(t) for every z in f(z) (hence x(t) for x and y(t) for y).
(D) Integrate f(z(t)𝑧 ′ (𝑡) over t from a to b
Note that the orientation of the curve is counterclockwise if it is not specified.
1
Example 1.Using theorem Prove that C z p
dz =2𝜋𝑖 where C is the circle of radius r and center
p, i.e,|𝑧 − 𝑝| = 𝑟
Solution . The parametric representation of C is given by z(t)=𝑝 + 𝑟𝑖𝑒 𝑖𝑡 and 0≤ 𝑡 ≤ 2𝜋.
it
1 2 ire dt 2
dz = idt it 0 2i
2
Since dz=ir𝑒 𝑖𝑡 𝑑𝑡,we have
C z p 0 re it 0
1
In particular, if p=0,
C z
dz 2i
Example 2.Evaluate zdz where C is the right hand half of a circle of radius 2 center at the
C
origin.
−𝜋 𝜋
SOLUTION. The parametric representation of C is given by z(t)=2𝑒 𝑖𝑡 ( 2 ≤ 𝜗 ≤ 2)
𝑧̅ = ̅̅̅̅̅
2𝑒 𝑖𝑡 = 2𝑒 −𝑖𝑡 and dz=2𝑖𝑒 𝑖𝑡 dt
𝜋 𝜋
Therefore zdz =∫
C
2
−𝜋
2
2𝑒 −𝑖𝑡 𝑖𝑡
2𝑖𝑒 𝑑𝑡=4∫ 𝑑𝑡 = 4𝜋𝑖
2
−𝜋
2
−𝜋 𝜋
It is also possible to write z(t)=2𝑒 𝑖𝑡 =2(cos 𝑡 + isin 𝑡) ≤𝜗≤ and 𝑧̅ =2(cos 𝑡 − isin 𝑡)
2 2
EXAMPLE 3. Evaluate C
f ( z )dz where f(z)=y-x-3𝑥 2 i C is the line segment from z=0 to
z=1+i.
Solution. The equation of the line segment C determined by the points z=0 and z=1+i is given
by y=x (check it).The parametric equation of C can be given as x= t & y= t where 0≤ 𝑡 ≤ 1 so
that
z(t)=x(t)+i y(t)= t+it and dz=(1+i)dt. Now f(t)= t-t-3𝑡 2 = −3𝑖𝑡 2 and 1+i
1 𝑡3 1
C
f ( z )dz =∫0 −3𝑖𝑡 2 (1 + 𝑖)𝑑𝑡 = −3 𝑖(1 + 𝑖) [ =
3 0
1−𝑖
Note that we can write 𝑧0 and 𝑧1 instead of C ,since we can get the same result for all those C
from 𝑧0 to 𝑧1 .
If f(z) is entire , we can take for D the complex plane (which is certainly simply connected)
Quick exercises
1. Evaluate x 2 yi dz
C
where C is the parabola y=𝑥 2 from z=1+2i to z=2+4i
1 1
2. .Evaluate ∫−1 𝑧 𝑑𝑧 along the curve C
We turn now to an inequality involving contour integrals that is extremely important in various
applications.
Definition 5.23. Let C be a piecewise differentiable curve given by the equation z=z(t) where
a≤ 𝑡 ≤ 𝑏. Then the length L of C is defined by
𝑏
L=∫𝑎 |𝑧 ′ (𝑡)| 𝑑𝑡
EXAMPLE 1. Find the length L of the circle C with center z0 and radius r.
Solution. The parametric equation of C is given by he equation z(t)= z0+ r𝑒 𝑖𝑡 where
0≤ 𝑡 ≤ 2𝜋 and 𝑧 ′ (𝑡)= ir𝑒 𝑖𝑡 .
∴ 𝐿 = ∫ |𝑧 ′ (𝑡)| 𝑑𝑡 =
0
|∫ 𝑓(𝑡)𝑑𝑡| ≤ ∫|𝑓(𝑡)|𝑑𝑡
𝑎 𝑎
| f ( z )dz | ≤ 𝑀𝐿
C
Where L is the length of the contour C & M is the upper bound of the modulus |𝑓(𝑧)| ,that
is |𝑓(𝑧)| ≤ 𝑀
𝑏 𝑏
Proof. | f ( z )dz | = |∫𝑎 𝑓(𝑧(𝑡)𝑧 ′ (𝑡)𝑑𝑡| ≤ ∫𝑎 |𝑓(𝑧(𝑡)𝑧 ′ (𝑡)|dz (why?)
C
𝑏 𝑏
≤ ∫𝑎 𝑀|𝑧 ′ (𝑡)| 𝑑𝑧 = 𝑀 ∫𝑎 |𝑧 ′ (𝑡)|𝑑𝑧 = 𝑀𝐿 (𝑤ℎ𝑦)
EXAMPLE 2. Let C be the arc of the circle |𝑧|=2 from z=2 to z=2i that lies in the first
z4 6𝜋
quadrant. Show that | dz | ≤
C z 3 1 7
then
z4 6𝜋
| dz | ≤ 𝑀𝐿 =
C z 1
3
7
Quick exercise
dz
1. Without evaluating the integral ,show that | dz | ≤ 4√2 where C is the line segment
C z4
joining z=i to z=1
C
f ( z )dz (udx vdy) i vdx udy
C C
Applying Green’s theorem to each of the two integrals on the right hand side in turn, we have
C
f ( z )dz 0
1 ±𝜋 ±3𝜋 ±5𝜋
where C is the unit circle, secz = 𝑐𝑜𝑠𝑧 is not analytic at z= , , ,….but all these points lie
2 2 2
outside C; none lies on C or inside C. Similarly for the second integral, whose integrand is not
analytic at z=±3𝑖.
2 z sin z
Example 3. If C denotes any closed contour lying in the open disk |𝑧| < 4 , then C z 2 25
dz =0.
This is because the disk is a simply connected domain and the two singularities z = ±5i of the
integrand are exterior to the disk.
Remark. Analyticity is sufficient but not necessary condition for Cauchy theorem to be true.
1 1
Example 4.
C ( z 1) 2
0 where C is a circle of radius 2 and center 1.But f(z)=(𝑧−1)2 is not
analytic at z=1.
Quick exercise.
Apply the Cauchy–Integral theorem to show that
f ( z )dz 0
C
when the contour C is the unit circle |z| = 1, in counterclockwise direction, and when
𝑧 1
1. 𝑓(𝑧) = 𝑧 2 +2𝑧+2 2. 𝑓(𝑧) = 𝑠𝑒𝑐ℎ 𝑧 3. f(z)=tan(4 𝑧)
The following theorem is an adaptation of the Cauchy Integral theorem to multiply connected
domains.
5.3.3 Cauchy’s Integral Formula
Cauchy’s integral theorem leads to Cauchy’s integral formula which is very important in
i. evaluating integrals as we shall see in this section
ii. proving that analytic functions have derivatives of all orders and
iii. Showing that all analytic functions have a Taylor series representation.
Theorem 5.15. Cauchy’s Integral Formula
Let f(z)be analytic in a simply connected domain D. Then for any point z 0 in D and any simple
closed path C in D that encloses z0 (Fig.5.13),
f ( z)
C z z0
2if ( z 0 ) ( 1) (Cauchy integral formula)
the integration being taken counterclockwise. Alternatively (for representing f(z0) by a contour
integral, divide (1) by 2𝜋𝑖),
𝟏 f ( z)
f(z0)=𝟐𝝅𝒊 (Cauchy’s integral formula).
C z z0
for any contour enclosing z0=π (since cos z is entire), and zero for any contour for which z0=π
lies outside (by Cauchy’s integral theorem).
Example 2.
Kassahun Nigatu(Msc),Mesfin Teshome(Msc) and Yitagesu Daba(Msc) 253
Applied Mathematics III
1 2
z 7
z 21
2
1 190
C 3z i dz
3 2i z 2 7[ i
i 3 z 27
z 3
3
𝑖
When z0=3 lies inside C.
zdz
Example 3. Evaluate
C (9 z 2 )( z i )
Let C be the positively oriented circle|𝑧| = 2.since the
𝑧
function f(z)=9−𝑧 2 is analytic within and on C and since the point z0= −i is interior to C, the
theorem
z 3 1dz 𝑧 3 + 1⁄ 9𝜋𝑖
C z 2 4 =2𝜋𝑖 ( 𝑧 + 2 ) [ 𝑧=2 = 2
Quick exercise:
𝑍2
1. Integrate by Cauchy’s Integral formula counterclockwise around the circle.
𝑍 2 −1
𝜋
𝑖. |𝑧 + 1| =1 𝑖𝑖. |𝑧 + 𝑖| = 1.4 𝑖𝑖𝑖. |𝑧 − 1 − 𝑖| = 2
dz
ii ) 2 C : 4 x 2 ( y 2) 2 4
C z 4
cosh 2 ( z i )
iii ) dz C the boundary of the square with vertices 2,4i
C z i
z
iv ) 2 dz , C : z 1 4 (Use partial fraction )
C z 4
Group Activity
Evaluate the Integrals 1-15 using either the first or second method.
1
Re z dz , C the parabola y 1 2 ( x 1) from 1 i to 3 3i.
2
1.
C
z
2. e 4 dz , C the shortest path from 4i to 8i
C
i
sec
2
4. z dz , C any path from to .
C
4 4
Im z
2
5. dz , counterclo ckwise around the triangle with vertices 0,1, i.
C
(z z
1
6. ) dz , C the uni tcircle , counterclo ckwise
C
1 6
7. ( z 2i ( z 2i)
C
2
)dz , C the circle z 2i 4, clockwise.
8. z exp z dz where C is the line segment from 1 2 i to 2 i
C
1 z
10. dz where C is the line segment from 1to i
C
z
dz
11. where C is the line segment from 2 to 2 i
C
z z
2
13. Re z 2 dz , clockwise aroud the boundary of the square with vertices 0, i ,1 i,1
C
z dz , where C is given by C : z (t ) t i t 2 , 0 t 1
2
14. Evaluate
C
Fig.5.14
EXAM PLE2.For any contour enclosing the point -2i we obtain by counterclockwise integration
z 3 2 z 2 1 2i 3
C ( z 2i) 3 2! (z 2 z 1) 12i 4
2 ''
z 2 i
Example3.Evaluate
cos z
C z ( z 2 9)
2
dz where C is a circle of radius 3 and center 1 counterclo ckwise , that is |𝑧 − 1|=3.
cos z
Solution. The function is not analytic at z=0 and 𝑧 = ±3𝑖 and the contour C encloses
z ( z 2 9)
2
𝑐𝑜𝑠𝑧
the point 0 but not ±3𝑖.If we let f(z)=𝑧 2 +9 , then by the Cauchy Integral domain
Example 4.Evaluate
ze z
C 4 z i 3 dz where C is z 1
1 z
ze
ze z ze z 1 z
Solution. dz = dz
64 dz so that f(z)= ze and
C
4 z i
3 3 3
C
C
64
4( z i z i
4 4
1 z 𝜋𝑖
𝑓 ′′ (𝑧) = e (2 z ) . The singular point z = −. is also inside C.
64 4
ze z 𝜋 1 i i 1 2 2 i
Therefore C 4 z i 3 dz =2𝜋𝑖(𝑓 ′′ (− 4 ) = 64 e 4 (2 4 ) 64 ( 2 2 )(2 4 ) .
Quick exercise . Evaluate then following integrals
sin z
1. C z4
where C is a unit circle counterclockwise
1
2. C i
dz , where C is a unit circle ,
( z ) 2 ( z 2i) 2
3
z 3 sin z
3. C ( z i) 3 dz , where C is the boundary of the square whose vertices are ±2, ±2𝑖
counterclockwise.
2
ez
4. C dz ,C:|𝑧 − 3𝑖| = 2 , clockwise
z ( z 2i ) 2
We turn now to the most powerful consequences of the Cauchy integral formula .
Theorem 5.17. CAUCHY’S INEQUALITY
Let 𝑧0 be a fixed complex number. If f is analytic within and on a circle |𝑧 − 𝑎| = 𝑅 in
counterclockwise direction , then
n!M
f ( n) ( z0 )
Rn
Proof. Applying the ML-inequality with |𝑓(𝑧)| ≤ 𝑀 on C and the length of the given circle is
2πR ,we obtain from derivative of analytic function
𝑛! 𝑀. 2𝜋𝑅 𝑛! 𝑀
|𝑓 (𝑛) (𝑧0 | ≤ = 𝑛
2𝜋 𝑅 𝑛+1 𝑅
Now let us proof the famous theorem on entire function called Liouville’s theorem using the
above Cauchy’s Inequality theorem.
T HEOREM 5.18. Liouville’s Theorem
If an entire function is bounded in absolute value in the whole complex plane, then this function
must be a constant.
Proof. By assumption, |𝑓(𝑧)| is bounded, say |𝑓(𝑧)| ≤ 𝑘, for all z. Using Cauchy’s inequality,
𝑘
we see that |𝑓 ′ (𝑧0 )| ≤ 𝑅 . Since f(z) is entire, this holds for every r, so that we can take r as large
as we please and conclude that 𝑓 ′ (𝑧0 ) = 0 .Since 𝑧0 is arbitrary, 𝑓 ′ (𝑧) = 𝑢𝑥 + 𝑖𝑣𝑥 = 0 for all z
(why?). Thus u= const, v= const and f= 𝑢 + 𝑖𝑣 = const for all z. This completes the proof.
The following theorem is also another very interesting consequence of Cauchy Integral formula
T H E O R E M 5.19 Morera’s Theorem (Converse of Cauchy’s Integral Theorem)
If is continuous in a simply connected domain D and if
C
f ( z )dz 0
tan z
1. 2 dz, C the boundary of the square whose sides lie along the line lines x 2 and y 2
C ( z x0 ) 2
and -2< x0 < 2.
cosh z
2. C z4
dz, C : z 1
exp( z 2 )
3. C ( z 1) 4 dz , C : z 3 2
1
4. C ( z 1) 2
2
dz where C:|𝑧 − 3| = 3 counterclokwise
4z 3 6
5. C z( z 1 i) 2
dz, C consists of z 3 counterclo ckwise and z 1 clockwise.
1
C z ( z 2 9)
2
dz, where C is
3
6. (i ) z (ii ) z 1
2
3
(iii ) z 2i (iv ) z 2i 3 (v) z 3
2
5.4 Complex series,Taylor and Laurent series, residues and poles
In the next section we shall see a new approach of evaluating complex integral called Residue
integral. But this method needs a thorough understanding of Taylor and Laurent series.
Therefore, we shall discuss these series in such a way that they play a leading role in evaluating
complex integral using residues.
Definition 5.24. If a complex function f is analytic at z=0, then the Maclaurin series expansion
of f is
∞
′ (0)
𝑧 2 𝑓 ′′ (0) 𝑧 2 𝑓 ′′ (0) 𝑓 (𝑛) (0)𝑧 𝑛
𝑓(0) + 𝑧𝑓 + + +⋯= ∑
2! 2! 𝑛!
𝑛=0
Everything that has been said about the Maclaurin series expansion of an expression involving a
real variable x can equally be said about an expression involving a complex variable z.
𝑧3 𝑧5 𝑧 2𝑛+1
Example 1. The Maclaurin series expansion of sinz=z− 3! + − ⋯ ((−1)𝑛 (2𝑛+1)!
5!
Because
f(z)=sinz and f(0)=0
𝑓 ′ (𝑧) = 𝑐𝑜𝑠𝑧 and 𝑓 ′ (0) = 1
𝑓′′ (𝑧) = −𝑠𝑖𝑛𝑧 and 𝑓 ′′ (0) = 0
𝑓 ′′′ (𝑧) = −𝑐𝑜𝑠𝑧 and 𝑓 ′′′ (0) = −1
.
.
.
Therefore, the Maclaurin series expansion of f is
′ (0)
𝑧 2 𝑓 ′′ (0) 𝑧 2 𝑓 ′′ (0) 𝑧3 𝑧5 (−1)𝑛 𝑧 2𝑛+1
𝑓(0) + 𝑧𝑓 + + +⋯=z− + −⋯= ∑
2! 2! 3! 5! (2𝑛 + 1)!
𝑛=0
Furthermore, applying the Ratio test the series is valid for all finite values because
𝑎𝑛+1 (𝑧)
lim𝑧 0 | |= 0 < 1 (verify it)
𝑎𝑛 (𝑧)
.
.
.
𝑓 (𝑛) (𝑧) = (−1)𝑛+1 𝑛! (1 + 𝑧)−𝑛 and so 𝑓 (𝑛) (0) = (−1)𝑛+1 𝑛!
Therefore, the required Maclaurin series expansion is
∞
𝑧2 𝑧3 (−1)𝑛+1 𝑧 𝑛
𝑧− + −⋯= ∑
2 3 𝑛
𝑛=1
𝑎𝑛+1 (𝑧)
since lim𝑧 0 | |=|𝑧|, then the series is valid for |𝑧| < 1 by the ratio test
𝑎𝑛 (𝑧)
𝑧 2𝑛 1
2.cosz= ∑∞ 𝑛 4. 1+𝑧 = ∑∞ 𝑛 𝑛
𝑧=0(−1) 𝑧
𝑛=0(−1) (2𝑛)!
𝑧 2𝑛+1 𝑧3 𝑧5
2. Sinhz =∑∞
𝑛=0 (2𝑛+1)! = 𝑧 + + 5! +…. For all z.
3!
𝑧 2𝑛+1 𝑧3 𝑧5
3. arc tanz =∑∞ 𝑛
𝑛=0(−1) 2𝑛+1 = 𝑧 − + − ⋯ |𝑧| < 1
3 5
4. Binomial series:
∞
1 −𝑚 𝑛
= (1 + 𝑧)−𝑚 = ∑ ( )𝑧
(1 + 𝑧) 𝑚 𝑛
𝑧=0
Fig 5.15
This means that the expansion is valid for all z within this circle. The radius of the circle within
which a series expansion is valid is called the radius of convergence and the circle is called the
circle of convergence.
Taylor series
Theorem 5.20. Let f(z) be analytic inside and on a simple closed curve C.Then the Taylor
expansion of f(z) about the point 𝑧0 which is interior to C is given by
𝑓 (𝑛) (𝑧0) 𝑓 ′ (𝑧0 )(𝑧−𝑧0 ) 𝑓 ′′ (𝑧0 )(𝑧−𝑧0 )2 𝑓 ′′′ (𝑧0 )(𝑧−𝑧0 )3
f(z)=∑∞
𝑛=0 (𝑧 − 𝑧0 )𝑛 =𝑓(𝑧0 ) + + + + ⋯+
𝑛! 1! 2! 3!
𝑓 (𝑛) (𝑧0)
(𝑧 − 𝑧0 )𝑛 …
𝑛!
where 𝑧0 is the center of the circle of convergence and the center of convergence is given by
|𝑧 − 𝑧0 | = 𝑅 where R is the radius of convergence.
Remark. The Maclaurin series is a special case of Taylor series about 𝑧0 =0.
1
Example 1.The Taylor series of (z) = 𝑧 at z=1 is given by
Now
1
f(z) = 𝑧 f(1)=1
1
𝑓 ′ (𝑧) = − 𝑧 2 𝑓 ′ (1) = −1
2
𝑓 ′′ (𝑧) = 𝑧 3 𝑓 ′′ (1) = 2
6
𝑓 ′′′ (𝑧) = − 𝑧 4 𝑓 ′ (1) = −6
.
.
.
1
Therefore the Taylor expansion of about z=1is
𝑧
∞
1
= 1 − (𝑧 − 1) + (𝑧 − 1)2 − (𝑧 − 1)3 + ⋯ = ∑(−1)𝑛 (𝑧 − 1)𝑛
𝑧
𝑛=0
𝑎𝑛+1
and lim𝑛→∞ | | = |𝑧 − 1| so that its circle of convergenc is |𝑧 − 1| < 1.
𝑎𝑛
convergence.
𝑧−2 𝑧−2 𝑧−2 𝑧−2 1 𝑧−2 (𝑧−2)𝑛
Solution. f(z)= 𝑧+2 = 4+(𝑧−2) = 𝑧−2 = 𝑧−2 = ∑∞
𝑛=0(−1)
𝑛
(𝑤ℎ𝑦? )
4(1+ ) 4 1+ 4 4𝑛
4 4
(𝑧−2)𝑛+1
=∑∞
𝑛=0(−1)
𝑛
4𝑛+1
𝑧−2
By the ratio test the region of convergence is given by | | < 1 which is the circular disc
4
|𝑧 − 2| < 4.
Example 3. Find the Taylor series of f(z)=𝑧 4 𝑒 3𝑧 at z=0 and determine its region of convergence.
𝑧𝑛 (3𝑧)𝑛 3𝑛 𝑧 𝑛
Solution. Since 𝑒 𝑧 = ∑∞
𝑛=0 𝑛! , 𝑒
3𝑧
= ∑∞
𝑛=0 = ∑∞
𝑛=0 .Therefore
𝑛! 𝑛!
3𝑛 𝑧 𝑛+4
𝑧 4 𝑒 3𝑧 = ∑∞
𝑛=0 and by the ratio test the series converges for all z,i.e |𝑧| < ∞.
𝑛!
Quick exercise. Find the Taylor series of the following functions and determine its region of
convergence.
𝑧
1. f(z)= 𝑧+5 about the point z =1
𝜋
2. f(z)=cosz with center z= 3
Laurent Series
If a function f fails to be analytic at a point 𝑧0, one can’t apply the Taylor theorem at that point.
It is often possibly ,however to find a new series representation of f involving both positive and
negative powers of (z-z0)(or constant) called Laurent series. Laurent series is the generalization
of Taylor series.
Definition 5.25. A Laurent series is a series of positive and negative integers powers of (z-z0)
by which we can represent a given function f(z) in an annulus (a circular ring with center z0) in
which f(z) is analytic, f(z) may have singularities outside the ring as well as in its hole.
The series (or finite sum of the negative powers is called the principal part(or singular part) of
the Laurent series. The principal part is used
for the classification of singularities
in powerful integration method(Residue integration) that we shall discuss in the next
section.
Theorem 5.21. (Laurent’s Theorem )
Let f(z) be analytic in a domain containing two concentric circles and with center z0 and the
annulus between them ( Fig.5.16). Then f(z) can be represented by the Laurent series
𝒃
𝒇(𝒛) = ∑∞ ∞ 𝒏
𝒏=𝟎 𝒂𝒏 + ∑𝒏=𝟏 {𝒛−𝒛 𝒏 =
𝟎)
𝒃𝟏 𝒃𝟐
𝒂𝟎 + 𝒂𝟏 (𝒛 − 𝒛𝟎 ) + 𝒂𝟐 (𝒛 − 𝒛𝟎 )𝟐 + … . + + +⋯ (𝟏)
𝒛−𝒛𝟎 (𝒛−𝒛𝟎 )𝟐
consisting of nonnegative and negative powers. The coefficients of this Laurent series are given
by the integrals
1 f (z ) 1
𝑎𝑛 = 2𝜋𝑖 C
z z0
dz n=0, ±1, ±2, …, bn
2I
C
f ( z )( z z 0 ) n1 f ( z )dz (2)
taken counterclockwise around any simple closed path C that lies in the annulus and encircles the
inner circle, as in Fig. 5.16.
This series converges and represents f(z) in the enlarged open annulus obtained from the given
annulus by continuously increasing the outer circle C1 and decreasing C2 until each of the two
circles reaches a point where f(z) is singular.
In the important special case that z0 is the only singular point of f(z) inside C2 , this circle can be
shrunk to the point z0 giving convergence in a disk except at the center. In this case the series
(or finite sum) of the negative powers of (1) is called the principal part or singular part of f (z)
at z0 or of that Laurent series (1).
Solutions.
𝑧 2𝑛
1. Since cos z= ∑∞ 𝑛
𝑛=0(−1) (2𝑛)! and f (z) is analytic at all points z≠ 0, we have
𝑐𝑜𝑠 𝑧 1 𝑧2 𝑧4 𝑧6 1 1 𝑧 𝑧3
= 𝑧 3 (1 − + − + ⋯)=𝑧 3 − 2𝑧 + 4 − +… (|𝑧| > 0)
𝑧3 2! 4! 6! 6!
Here the annulus of convergence is the whole complex plane without the origin and its principal
1 1
part is − 2𝑧
𝑧3
𝑧𝑛
2. Since 𝑒 𝑧 = ∑∞
𝑛=0 𝑛! for all z and f(z) is analytic at all points z≠ 0 we have
1 1
2𝑛 1 1 1
f(z)=𝑧 3 𝑒 𝑧2 = 𝑧 3 ∑∞ 3
𝑛=0 𝑛! = 𝑧 (1 + 𝑧 2 +2 𝑧 4 + 6 𝑧 6 … for all z and its principal part is
𝑧
1 1 1
+ 6𝑧 3 + 24𝑧 5 + ⋯
2𝑧
3. Since f(z) is analytic at all points z≠ 0,1,then f is analytic in annulus 0 < |𝑧 − 1| < 1
and hence can be expressed as a Laurent series in powers of z-1 as
1 1 1 1
= 𝑧−1 (1+(𝑧−1)) = 𝑧−1 (1 − (𝑧 − 1) + (𝑧 − 1)2 − (𝑧 − 1)3 + ⋯
𝑧(𝑧−1)
1
=𝑧−1 − 1 + (𝑧 − 1) − (𝑧 − 1)2 + ⋯ since |𝑧 − 1| < 1
𝑒𝑧 𝑒 𝑢+1 𝑒𝑢 𝑒 𝑢2 𝑢3 𝑢4
4. Let z-1=u. Then 𝑓(𝑧) = (𝑧−1)2 = (𝑢)2
= 𝑒 𝑢2 = 𝑢2 (1 + 𝑢 + + + +⋯
2! 3! 4!
𝑒(𝑧−1)2
+ + ⋯ which is valid for 0 < |𝑧 − 1| < ∞.
4!
1
5.a) 1−𝑧 = ∑∞ 𝑛
𝑛=0 𝑧 which is valid for |𝑧| < 1 by the ratio test
1 −1 −1 1 1 1 1
𝑏) = 1 = ∑∞
𝑛=0 = − ∑∞
𝑛=0 𝑧 𝑛+1 = − 𝑧 − 𝑧 2 − ⋯ which is valid for |𝑧| > 1
1−𝑧 𝑧(1− ) 𝑧 𝑧𝑛
𝑧
Quick exercise.
1
1. Find the Laurent series that represents the function 𝑓(𝑧) = 𝑧 2 sin 𝑧 2 in the domain 0
for
i) |𝑧| < 1 ii) 1< |𝑧| < 2 iii) |𝑧| > 2
𝑧
3. Find the Laurent series of f(z)= (𝑧+1)(𝑧+2) with center z0= -1
Group Activity
1. Find the Laurent series representations of the following function that converges for 0<|𝑧| <
𝑅 and determine its region of convergence.
𝑠𝑖𝑛𝜋𝑧 1
a) f(z)= d) f(z)= 𝑧 3 𝑐𝑜𝑠ℎ 𝑧
𝑧2
1 𝑒𝑧
b) f(z)= 𝑧 2 −𝑧 3 e) f(z)= = 𝑧 2 −𝑧 3
𝑠𝑖𝑛ℎ3𝑧
c) f(z)= 𝑧2
2. Find the Laurent series representations of the following function that converges for 0<|𝑧 −
𝑧0 | < 𝑅 and determine its region of convergence.
𝑒𝑧 𝑒 2𝑧
a) f (z)=(𝑧−2)2 , 𝑧0 = 2 c) f(z)=(𝑧−1)2 , 𝑧0 = 1
cos 𝑧 1
b) f(z)=(𝑧−𝜋)2, 𝑧0 = 𝜋 d) f(z)=(𝑧 2 (𝑧−𝑖), 𝑧0 =i
𝑧−2
3. Find the Laurent series of (𝑧+2)(𝑧+5) that is valid for
Singularities
Recall that a function f(z) is singular or has a singularity at a point z=z0 if f(z) is not analytic
(perhaps not even defined) at z=z0 but every neighborhood of z=z0 contains points at which f(z)
is analytic. We also say that is z=z0 singular point of f(z).
Definition 5.25: A singular point z=z0 is called isolated singular point of f(z) if it has a
neighborhood without further singularities of f(z) .Otherwise it is called non-isolated singularity.
𝜋 3𝜋 5𝜋
Example 1. f(z)= tan z has isolated singularities at ± 2 , ± ,± , 𝑒𝑡𝑐 .
2 2
1 sin1⁄
Example 2 . Let f(z)= tan 𝑧 = cos1⁄𝑧 .The singular points of f(z) are 0 and those points of z for
𝑧
1 1 𝜋 2
which cos 𝑧 = 0, 𝑡ℎ𝑎𝑡 𝑖𝑠 = (2𝑛 + 1) 2 𝑎𝑛𝑑 𝑡ℎ𝑒𝑛 𝑧 = (2𝑛+1)𝜋 , 𝑛 = 0, ±1, ±2, …
𝑧
But the singular point z = 0 is not isolated because every neighborhood of the origin contains
other singular points of the function .Therefore, z=0 is non-isolated singular point.
Laurent series expansion of the function f(z) can be used to classify the isolated singular points.
Let z=z0 be an isolated singular points of f(z).Then there exist a neighborhood of the point z 0
,0 < |𝑧 − 𝑧0 | < 𝑅 inside this f(z) is analytic.Hence in this region we can expand the function f(z)
as a Laurent series
𝑏
f(z)=∑∞ 𝑛 ∞ 𝑛
𝑛=0 𝑎𝑛 (𝑧 − 𝑧0 ) + ∑𝑛=1 (𝑧−𝑧 )𝑛
0
valid in the neighborhood of the singular point z =𝑧0 except at z0 itself, that is ,in the region of
the form 0 < |𝑧 − 𝑧0 | < 𝑅.
Removable Singularity
Definition 5.26. If f (z) has an isolated singularity at z=z0 but lim𝑧→𝑧0 𝑓(𝑧) exists , then the point
z=z0 is called a removable singularity.
Note that in this case the principal part of the Laurent series is zero and lim𝑧→𝑧0 𝑓(𝑧) = 𝑎0
Example 1.
𝑠𝑖𝑛𝑧 1 𝑧3 𝑧5 𝑧2 𝑧4
Let f(z)= = 𝑧 (𝑧 − + −⋯= 1− + − ⋯ .Obviously , z=0 is an isolated
𝑧 3! 5! 3! 5!
singularity. Since the principal part is zero and lim𝑧→0 𝑓(𝑧) = 1 = 𝑎0 , then z=0 is a removable
singularity.
Alternatively(without using Laurent series expansion), since z=0 is the isolated singular point of
𝑠𝑖𝑛 𝑧
f and lim𝑧→0 = 1, then z=0 is the removable singularity.
𝑧
1−cos 𝑧
Example 2. The point z=0 is a removable singular point of the function f(z)=
𝑧
because
1 𝑧2 𝑧4 𝑧6 𝑧 𝑧3 𝑧5
f(z) = [1 − (1 − + − + − − −)] = − + − ----
𝑧 2! 4! 6! 2! 4! 6!
1−cos 𝑧
whose principal part is zero and moreover lim𝑧→0 = 0 by L'hopital rule.
𝑧
1−cos 𝑧
Alternatively, since z=0 is an isolated singular point of f and lim𝑧→0 exists (≠ 0), then z=0
𝑧
POLE
Definition 5.27. If the principal part of the Laurent series of the function f(z) has only a finite
number of terms of the form,
𝒃𝟏 𝒃𝟐 𝒃𝟐
+
𝒛−𝒛𝟎 (𝒛−𝒛𝟎 ) 𝟐 + ⋯+ (𝒛−𝒛𝟎 )𝒏
where n is a finite integer and 𝑏𝑛+1 = 𝑏𝑛+2 = ⋯ = 0 then
z=z0 is called a pole of order. In particular, if n=1(Poles of the first order) , we call a simple
pole.
Alternatively, an isolated singular point z=z0 is said to be a pole if lim𝑧→𝑧0 |𝑓(𝑧)| = ∞ and
lim𝑧→𝑧0 (z − z0 )n f(z) exists and different from zero . The smallest values of n for which this
limit exists define the order of the pole.
Example 1. The function
2 𝑧−4
f(z) = 𝑧(𝑧+3)4 + (𝑧−5)2 (𝑧+6)3
has a simple pole at 0 , a pole of fourth order at -3, a pole second order at 5 and a pole of third
order at -6.In each case we can easily show that lim𝑧→𝑧0 (𝑧 − 𝑧0 )𝑛 𝑓(𝑧) ≠ 0.
Example 2. The function
sin 𝑧
f(z)= has an isolated singular point at z=0 and thus can be expressed by the Laurent series as
𝑧4
𝑠𝑖𝑛𝑧 1 𝑧3 𝑧5 1 1 𝑧 𝑧3 1 1
= 𝑧 4 (𝑧 − + − ⋯ = 𝑧 3 − 3!𝑧 + 5! − +⋯ whose principal part is − 3!𝑧.Therfore
𝑧4 3! 5! 7! 𝑧3
third order.
Essential singularity.
Definition 5.28. If the principal part of the Laurent expansion of f(z) has infinite number of
terms, then the point z=z0 is called an essential singular point of f(z).
1⁄
Example 1 . f(z)= 𝑒 𝑧 has an essential singularity at z=0 since the principal part of the Laurent
series expansion about z=0 has infinite number of terms. That is
∞ ∞
1 (1⁄𝑧)𝑛 1 1 1 1
𝑒 ⁄𝑧 =∑ = ∑ = 1 + + + +⋯
𝑛! 𝑛! 𝑧 𝑛 𝑧 2! 𝑧 2 3! 𝑧 𝑛
𝑛=0 𝑛=0
Quick exercises. Determine the location of the singularities of the the following function and for
pole state the order.
1−𝑐𝑜𝑠𝑧 1−𝑒𝑥𝑝2𝑧 1
1. 𝑓(𝑧) = 2. 𝑓(𝑧) = 3. 𝑓(𝑧) = sin 𝑧
𝑧2 𝑧5
Group Activity
1. Locate the singularities of the following functions and state the orders for the poles.
1 1⁄
1. (𝑧) = 𝑧 3 (𝑧 2 +9)2 4. 𝑓(𝑧) = 𝑧𝑒𝑥𝑝 𝑧
𝑧
2. 𝑓(𝑧) = 𝑡𝑎𝑛𝑧 5. 𝑓(𝑧) = sin 𝑧
𝑧2 cos 𝑧−cos 2𝑧
3. 𝑓(𝑧) = 𝑧−sin 𝑧 6. 𝑓(𝑧) = 𝑧4
theorem, we have f ( z )dz 0 . However, when the integral f(z) has one or more isolated
C
singular points inside a closed contour C, then the Cauchy integral theorem can’t be used and the
inside C contribute to the value of the complex integral. These contributions are called residues.
Residues of analytic functions at an isolated singular point.
Definition 5.29. Let f(z) be analytic at all points z except at z=𝒛𝟎 .Let 𝒛𝟎 be an isolated singular
point. Then f(z) can be expanded as a Laurent series about z=𝒛𝟎 in the form
𝒃𝟏 𝒃
𝒇(𝒛) = 𝒂𝟎 + 𝒂𝟏 (𝒛 − 𝒛𝟎 ) + 𝒂𝟐 (𝒛 − 𝒛𝟎 )𝟐 + … . + + (𝒛−𝒛𝟐 )𝟐 + ⋯
𝒛−𝒛𝟎 𝟎
Which converges in some region R: 0 < |𝑧 − 𝑧0 | < 𝑟. The coefficient bn are defined by
1
bn
2i C
f ( z )( z z 0 ) n1 f ( z )dz
Where n=1,2,3… and C is a simple closed curve in the annulus 0< |𝑧 − 𝑧0 | < 𝑟 .The
coefficient of (z-z0)-1 ,that is, b1 in the Laurent series expansion of f(z) is called Residue of f(z)
at z=z0 and written as
b1 Re s( f ( z ); z0 )
Therefore , if the residue b1 at z=z0 can be determined by some method then the value of the
contour integral is given by
f ( z)dz 2ib . 1
sin 𝑧
Example 1. Integrate the function f(z)= counterclockwise around the unit circle C.
𝑧4
1 1 1 𝑧3
− 3!𝑧 + 5! − + ⋯ which converges for |𝑧|>0 .From the series we can see that z=0 is a pole
𝑧3 7!
1 1
of order 3 and the residue b1=− 3! = − 6 .
𝜋
∴ C
f ( z )dz 2ib1 =− 3
1 1
Solution. = z3 (1−z) has isolated singular points z=0 and z=1 of which z=1 lies outside C.
z3 −z4
f ( z)dz 2i(0) 0 .
𝑠𝑖𝑛3𝑧
Example . z=0 is a removable singular point of the function f(z)= because its Laurent series
𝑧
1 (3𝑧)3 (3𝑧)5 27𝑧 2 243𝑧 4
f(z)= 𝑧 (3𝑧 − + − ---) =3− + − --- has no principal part and thus
3! 5! 3! 5!
b1=0.
2. Residue at a simple pole
Let z0 be a simple pole of f(z).In this case
𝒃𝟏
𝒇(𝒛) = ∑∞ 𝒏
𝒏=𝟎 𝒂𝒏 (𝒛 − 𝒛𝟎 ) + .
𝒛−𝒛𝟎
4𝑧−3𝑖 5𝑖
Res(f(z);2i) 𝐥𝐢𝐦𝒛→𝟐𝒊 (𝒛 − 𝟐𝒊) =−8
𝑧(𝑧+2𝑖)(𝑧−2𝑖)
4𝑧−3𝑖 11𝑖
Res(f(z);-2i) 𝐥𝐢𝐦𝒛→−𝟐𝒊 (𝒛 + 𝟐𝒊) 𝑧(𝑧+2𝑖)(𝑧−2𝑖) = 8
p( z) p( z )
b1 Re sf ( z) Re s '
0
z z0 q( z) q ( z ) 0
z z0
𝑝(𝑧)
assuming that f(z)= 𝑞(𝑧) with p(z0)≠ 0 and f(z) has a simple pole at z0.
Proof.
𝑝(𝑧)
Res(f(z);z0)=lim𝑧→𝑧0 (𝑧 − 𝑧0 )(𝑓(𝑧) = lim𝑧→𝑧0 (𝑧 − 𝑧0 ) 𝑞(𝑧) =
𝑧−𝑧0 0 𝑧−𝑧 1
lim𝑧→𝑧0 𝑝(𝑧) lim𝑧→𝑧0 = lim𝑧→𝑧0 𝑝(𝑧) lim𝑧→𝑧0 𝑞(𝑧)−𝑞(𝑧 )
= 𝑝(𝑧0 ) 𝑞′ (𝑧 ) (why?)
𝑞(𝑧) 0 0
4𝑧−3𝑖
Example 2. From the above example f(z)=𝑧 3 +4𝑧 has simple poles at z=0 and z=±2𝑖 and its
(z − z0 )m f(z)= ∑∞
𝒏=𝟎 𝒂𝒏 (𝒛 − 𝒛𝟎 )
𝒏+𝒎
+ 𝒃𝟏 (𝒛 − 𝒛𝟎 )𝒎−𝟏 + 𝒃𝟐 (𝒛 − 𝒛𝟎 )𝒎−𝟐 + ⋯
then differentiating m-1 times and taking the limit as z→ 𝑧0 we obtain
𝟏 𝒅𝒎−𝟏
𝒃𝟏 = (𝒎−𝟏)! 𝐥𝐢𝐦𝒛→𝒛𝟎 ((𝒛 − 𝒛𝟎 )𝒎 𝒇(𝒛).
𝒅𝒛𝒎−𝟏
𝑑𝑧
Example 1 .The function 𝑓(𝑧) = 𝑧(𝑧−2)3 has a pole of order three at z=2 (verify!) and its
residues there is
1 𝒅𝟐 𝟏 𝟏 𝟐 𝟏
Res(f(z);2)=b1= (3−1)! 𝐥𝐢𝐦𝒛→𝟐 𝒅𝒛𝟐 ((𝒛 − 𝟐)𝟐 𝒛(𝒛−𝟐)𝟐 = 𝟔 𝐥𝐢𝐦 𝒛𝟑 = 𝟐𝟒
𝒛→𝟐
Group Activity
Find all the singularities in the finite plane and the corresponding residues of the functions 1-8.
1 4. cot 𝜋𝑧 1⁄
1. sin
𝑧
8. 𝑒 1−𝑧
cos 𝑧
sinh 𝑧 5.
2. 𝑧4
𝑧4
𝑧
1 6. ( )3
3. 2𝑧+1
1−𝑒 𝑧 exp 𝑧
7.
𝑧 2 +𝜋2
𝜋cot(𝜋𝑧) 𝜋2
9. Show that the residue of f(z)= at z=0 is -
𝑧2 3
We have seen how to find the integral of f(z) which is analytic in the contour C at single
singularity .What if the contour C contains a finite number of singularities? Here is the answer.
Theorem 5.30 The residue Theorem
Let f(z) be analytic inside a simple closed path C and on C, except for finitely many singular
points 𝑧1 , 𝑧2 , 𝑧3 ,…, 𝑧𝑘 inside C. Then the integral of f(z) taken counterclockwise around C
equals 2πi times the sum of the residues of f(z) at 𝑧1 , 𝑧2 , 𝑧3 ,…, 𝑧𝑘 :
Kassahun Nigatu(Msc),Mesfin Teshome(Msc) and Yitagesu Daba(Msc) 272
Applied Mathematics III
f ( z )dz 2i Re s( f ( z ); z j )
C
j 1
Proof. Since 𝑧1 , 𝑧2 , 𝑧3 ,…, 𝑧𝑘 are isolated singularities ,then there is a set of non-overlapping
circles Ck with center 𝑧𝑘 ,k=1,2,3,…,n(for example,Fig.5.17 where k=3) such that each circle Ck
enclosed only one singular point zk and lies inside C. By Cauchy integral theorem for multiply
connected domain we write,
We know ck
f ( z )dz =2𝜋𝑖Res(f(z); 𝑧𝑘 ).Substituting this into (*) we have
f ( z )dz 2i Re s( f ( z ); z j )
C
j 1
Solution. The integrand has two simple poles at z=0 and z=3, both of which are interior to C.
Also
3𝑧+2 2 3𝑧+2 8
Res(f(z),z=0)=lim𝑧→0 𝑧 𝑧(𝑧−3) = − 3 𝑎𝑛𝑑 Res(f(z),z=3)=lim𝑧→3 ( 𝑧 − 3)(𝑧(𝑧−3) = 3 ,then by the
Residue theorem
3z 2 2 8
C z ( z 3)
dz =2𝜋𝑖(− 3 + 3) = 4𝜋𝑖
e z 1
Example 2. Evaluate dz where
C z ( z 1)( z i ) 2
1 C2: |𝑧| = 2
i. C1: |𝑧| = 2 ii.
Solutions. z=0,1,2 are isolated singularities of f(z).Then z=0 is a removable singularity, z=1 is a
simple pole and z=0 is a pole of order 2.
i. z=0 is the only singularity which lies in C1 and Res(f(z);z=0)=0 since z=0 is a
removable singular point and thus
e z 1
C z( z 1)( z i) 2 dz 2i(0) 0
ii. All the three singular points lie in C2 and
𝑒 𝑧 −1 1−𝑒
Res(f(z);z=1)= lim𝑧→1 ( 𝑧 − 1)(𝑧(𝑧−1)(𝑧−𝑖)2 = and
2𝑖
𝑑 𝑒 𝑧 −1 1
Res(f(z);z=i)= lim𝑧→𝑖 𝑑𝑧 ((𝑧 − 𝑖)2 (𝑧(𝑧−1)(𝑧−𝑖)2 )) = 2 (3𝑒 𝑖 − 𝑖) (verify it!)
𝜋 3𝜋
Solution. tan z is not analytic at z=± 2 , ± , …. ,all of which are outside the contour C. The
2
𝑡𝑎𝑛𝑧
integrand has simple pole 2 and -2 since z2-4=0.z=0 is also an essential singularity of the
𝑧 2 −4
1⁄
integrand 𝑧𝑒 𝑧 which lies inside C. Then
𝑡𝑎𝑛𝑧 𝑡𝑎𝑛𝑧 𝑡𝑎𝑛2
Res(𝑧 2 −4; z= 2)=2𝜋𝑖 lim𝑧→2 (𝑧 − 2) (𝑧−2)(𝑧+2) = 4
and
𝑡𝑎𝑛𝑧 𝑡𝑎𝑛𝑧 𝑡𝑎𝑛−2 𝑡𝑎𝑛2
Res(𝑧 2 −4; z= -2)=2𝜋𝑖 lim𝑧→−2 (𝑧 + 2) (𝑧−2)(𝑧+2) = = (𝑤ℎ𝑦) .
−4 4
1⁄
Since z=0 is an essential singularity ,the only method to find the residue of 𝑧𝑒 𝑧 is finding its
Laurent series.So
1⁄ 1 1 1
𝑧𝑒 𝑧 =z(1 + 𝑧 + 2!𝑧 2 + 3!𝑧 3+….)
1 1 1 1
= z +1+ 2!𝑧 + 3!𝑧 2 + ⋯ from which its residue is the coefficient of , that is 2.
𝑧
Quick exercise.
zez
2.
C
(
z 16
4
ze z )dz , where C is the ellipse 9𝑥 2 + 𝑦 2 = 9
z 1
3.
C ( z 1)( z 3) 4
dz, C: |𝑧 − 2| =2
Group Exercise
1. Find Res(f,0) for
a) 𝑓(𝑧) = d) 𝑓(𝑧) = 𝑐𝑠𝑐 2 𝑧
f) 𝑓(𝑧) = 𝑧
1
𝑧 −3 cosh 4𝑧 𝑧 4 sin(1 + 𝑧)
b) 𝑓(𝑧) = csc 𝑧 exp(4𝑧)−1
e) 𝑓(𝑧) =
1 𝑠𝑖𝑛2 𝑧
c) 𝑓(𝑧) = exp(1 + 𝑧
1 e. cot(𝜋𝑧)
b. 1−𝑒 𝑧
1⁄ f. tanz
c. 𝑒 1−𝑧
3. Let p(z) be a polynomial of degree at most 2.Show that if a,b and c are distinct complex
numbers, then
𝑝(𝑧) 𝐴 𝐵 𝐶
f(z)=(𝑧−𝑎)(𝑧−𝑏)(𝑧−𝑐) = 𝑧−𝑎 + 𝑧−𝑏 + 𝑧−𝑐
𝑝(𝑎)
where A=Res(f, a)=(𝑎−𝑏)(𝑎−𝑐)
𝑝(𝑏)
B=Res(f, b)=(𝑏−𝑎)(𝑏−𝑐) and
𝑝(𝑐)
C=Res(f, c)=(𝑐−𝑎)(𝑐−𝑏)
1
1
c.
z 3e z
C 1 z3
dz, C:|𝑧| = 3 j. C z z 3 2z 2
4
dz, 𝐶: |𝑧| = 3
(z 2) 1dz, 𝐶: |𝑧 − 1| = 2
4
3z 2 2 z 5 k.
C (3z 2) 2 (2 z 1)dz , 𝐶 𝑖𝑠 𝑡ℎ𝑒 𝑢𝑛𝑖𝑡 𝑐𝑖𝑟𝑐𝑙𝑒
C
d.
1
z 5
l. C z 4
4
dz, C: |𝑧 + 1 − 𝑖| = 1
e.
C z 3z 3
4
dz, C: |𝑧 − 𝑖| = 4
sin z
cos z
m. C 4z 2 2
dz, C: |𝑧| = 2
f.
C z5
dz, C:|𝑧 − 1| = 1.5
1
n. 3z 10 z 2 3
4
dz, C: |𝑧 −
tan zdz ,
C
g. C:|𝑧| = 2
C
𝐼
𝑖√3| = 1 , C:|𝑧 − |=1
dz √3
h. , C:|𝑧| = 2
C sinh z
z cosh z
i.
C z 13z 2 36
4
dz, C: |𝑧| = 𝜋
5. Show that
𝑧−𝑠𝑖𝑛ℎ𝑧 𝑖
a) Res( ; 𝜋𝑖) =
𝑧 2 𝑠𝑖𝑛ℎ𝑧 𝜋
exp(𝑧𝑡) exp(𝑧𝑡)
b) Res( ; 𝜋𝑖) + Res( ; −𝜋𝑖) = −2𝑐𝑜𝑠(𝜋𝑡)
𝑠𝑖𝑛ℎ𝑧 𝑠𝑖𝑛ℎ𝑧
2𝜋
dz
∫ 𝐹(𝑐𝑜𝑠𝜃, 𝑠𝑖𝑛𝜃)𝑑𝜃 =
C
f ( z)
iz
0
and, as 𝜃 ranges from 0 to in 2𝜋 , the variable z=𝒆𝒊𝜽 ranges counterclockwise once around the
unit circle |𝑧| = 1.
Example 1 . Evaluate the following integrals.
2𝜋 𝑑𝜃
1. ∫0 5+4𝑠𝑖𝑛𝜃
𝑑𝑧 1 1
Solution. Let 𝑧 = 𝑒 𝑖𝜃 .Then 𝑑𝜃 = and using 𝑠𝑖𝑛𝜃 = (z − ) the integral becomes
𝑖𝑧 2i z
2𝜋 𝑑𝜃 dz
∫0 = where C: |𝑧| = 1
5+4𝑠𝑖𝑛𝜃 C z z 1
iz[5 4( )]
2i
dz dz
= .
C 2 z 5iz 2
2 C i
2( z 2i)( z )
2
𝑖
Since z=-2i and z=− 2 are simple poles of the integrand of which -2i is outside the contour
𝑖
C,then we compute the residue at − 2 as
𝑖 𝑖 1 1
Res(f(z);z=− 2)=lim𝑧→− 𝑖 (𝑧 + 2) 𝑖 = 3𝑖.Hence by the Residue theorem
2 2(𝑧+2𝑖)(𝑧+ )
2
2𝜋 𝑑𝜃 1 2𝜋
∫0 =2𝜋𝑖(3𝑖)= 3
5+4𝑠𝑖𝑛𝜃
Example 2.
2𝜋 𝑑𝜃
2. ∫0 2+𝑐𝑜𝑠𝜃
1 1
Solution. Letting 𝑧 = 𝑒 𝑖𝜃 and 𝑐𝑜𝑠𝜃 = 2 (z + z),the given integral becomes
dz
C z z 1
where C: |𝑧| = 1.
iz (2 )
2
dz dz 2idz
Therefore , C zz 1
=
C i ( 4 z z 1)
2
C ( z 2) 2 3
iz (2 )
2
2idz
=
( z 2 3 )( z 2 3 )
2idz 𝑖 2𝜋
(z 2 3 )( z 2 3 )
=2𝜋𝑖 (− ) =
√3 √3
polynomials in x and the degree of h(x) exceeds that of g(x) by at least two.To evaluate this type
𝑔(𝑧)
of integrals we let 𝑓(𝑧) = ℎ(𝑧).The poles of f(z) are determined by the zeros of the equation
h(z)=0.
Case I : No poles of f(z) lies on the real axis:
In this case we choose the curve C consisting of the interval [-r, r] on the real axis and the semi
circle 𝐶1 : |𝑧| = 𝑟 lying the upper half of the plane.Here we choose r large enough so that all the
poles lying in the upper half of the plane are in the interior of C.Then we have
r
C
f ( z )dz f ( x)dx f ( z )dz where C1 is the semicircle. Since the degree of h(x)-the
r C1
C
f ( z )dz f ( x)dx . Therefore
f ( x)dx can be evaluated by evaluating C
f ( z )dz by the
residue theorem.
f ( x)dx =2𝜋𝑖 ∑ 𝑅𝑒𝑠(𝑓(𝑧), the sum is over all residues of the corresponding
∴
Suppose that z0 is a pole lying on the real axis. In this case we indent the real axis by a semi
circle C2 of radius 𝜀 with center z0 lying on the upper half plane where 𝜀 is chosen to be
sufficiently small.
the value of
∞
∫ 𝑓(𝑥)𝑑𝑥 = 𝜋𝑖𝑅𝑒𝑠(𝑓(𝑧); 𝑧 = 𝑧0 )
−∞
∞ 1
Example 1. Evaluate ∫−∞ 𝑥 4 +1
𝜃+2𝑘𝜋
1
. The f(z) has four simple pole at z=√−1 = 𝑒 𝑖(
4
Solution. Let f(z)= 𝑛
𝑧 4 +1
𝜋𝑖⁄ 3𝜋𝑖⁄ −𝜋𝑖⁄ −3𝜋𝑖⁄
,n=4,𝜃 = 𝜋,k=0,1,2,3,namely z1=𝑒 4 , 𝑧2 =𝑒 4 , 𝑧3 =𝑒 4 𝑎𝑛𝑑 𝑧4 = 𝑒 4
But only 𝑧1 𝑎𝑛𝑑 𝑧2 lie in the upper half of the plane.Let h(z)=1 and k(z)=𝑧 4 + 1 so that f(z)
ℎ(𝑧)
= 𝑘(𝑧).
𝜋𝑖⁄ −3𝜋
ℎ(𝑧1 ) 1 1 1 1
Res(f(z);z=𝑒 4 )= = 4𝑧 3 =4 𝑒 4 = − 4√2 − 4√2 𝑖
𝑘 ′ (𝑧1 ) z z1
3𝜋𝑖⁄ −9𝜋
ℎ(𝑧1 ) 1 1 1 1
Res(f(z);z=𝑒 4 )= = 4𝑧 3 =4 𝑒 4 = 4√2 − 4√2 𝑖
𝑘 ′ (𝑧1 ) z z 2
∞ 1 1 1 1 1 𝜋
Therefore ∫−∞ 𝑥 4 +1 = 2𝜋𝑖 (− 4√2 − 4√2 𝑖 + 4√2 − 4√2 𝑖) =
√2
∞ 𝑑𝑥
Example 2. Evaluate ∫−∞ (𝑥 2 −5𝑥+6)(𝑥 2 +4)
𝑑𝑧
Solution. Let f(z)= (𝑧 2 −5𝑧+6)(𝑧 2 +4). f(z) has four simple poles at z= ±2𝑖, 𝑧 = 2 𝑎𝑛𝑑 𝑧 = 3 of
which 𝑧 = −2𝑖 is in the lower half plane which is of no interest here and z=2 and z=3 are on the
real axis.
1 1
Res(f(z);z=2)=lim𝑧→2 (z − 2) (z−2)(z−3)((z2 +4) = − 8
1 1
Res(f(z);z=3)=lim𝑧→3 (z − 3) (z−2)(z−3)((z2 +4) = 13
1 1 40−8i 5−i
Res(f(z);z=2i)=limz→2i (z − 2i) (z−2)(z−3)(z+2i)(z−2i) = 40+8i = = 208
1664
∞ dx 5−i 1 1 π
∴ ∫−∞ (x2 −5x+6)(x2 +4) = 2πi(208) + πi (− 8 + 13) = 104
∞ 𝑥 2 𝑑𝑥 𝜋 ∞ 𝑥 4 𝑑𝑥 √3𝜋
1. ∫−∞ (𝑥 2 +1)2 = 2. ∫0 =
2 𝑥 6 −1 6
If f(z) is analytic in a simply connected domain D, then we can evaluate (8) as in calculus by
indefinite integration and substitution of limits, that is,
(9)
f ( z)dz F ( z ) F ( z ) where F ( z ) f ( z )
'
1 0
for every path C in D from a point 𝑧0 to a point 𝑧1 . These assumptions imply independence of
C
path, that is, (9) depends only on 𝑧0 and 𝑧1 (and on f(z) of course) but not on the choice of C.
A general method of integration, not restricted to analytic functions, uses the equation z=z(t) of
C, where 𝑎 ≤ 𝑡 ≤ 𝑏,
Kassahun Nigatu(Msc),Mesfin Teshome(Msc) and Yitagesu Daba(Msc) 281
Applied Mathematics III
10
b
C
f ( z )dz f ( z (t )) z ' (t )dt
a
Cauchy’s integral theorem is the most important theorem in this chapter. It states that if f(z) is
analytic in a simply connected domain D, then for every closed path C in D
f ( z)dz 0 .
C
Under the same assumptions and for any 𝑧0 in D and closed path C in D containing 𝑧0 in its
interior we also have Cauchy’s integral formula
1 f ( z)
f ( z0 )
2i C z z 0
dz. (11)
Furthermore, under these assumptions f(z) has derivatives of all orders in D that are themselves
analytic functions in D and
𝑛! f ( z)
𝑓 𝑛 (𝑧0 ) = 2𝜋𝑖 (z z 0)
n 1
dz , n 1, 2 ,.... (12)
This implies Morera’s theorem (the converse of Cauchy’s integral theorem) and Cauchy’s
inequality , which in turn implies Liouville’s theorem that an entire function that is bounded in
the whole complex plane must be constant.
A Laurent series is a series of the form
∞ ∞
𝑏𝑛
𝑓(𝑧) = ∑ 𝑎𝑛 (𝑧 − 𝑧0 )𝑛 + ∑ (13)
(𝑧 − 𝑧0 )𝑛
𝑛=0 𝑛=1
or, more briefly written
1 f (z* )
𝑓(𝑧) = ∑∞ 𝑛
𝑛=−∞ 𝑎𝑛 (𝑧 − 𝑧0 ) , 𝑎𝑛 = 2𝜋𝑖 ( z * z ) n 1
dz *
C 0
where n=0,1,2, ... .This series converges in an open annulus (ring) A with center 𝑧0. In A the
function f(z) is analytic. At points not in A it may have singularities. The first series in (12) is a
power series. In a given annulus, a Laurent series of f(z) is unique, but f(z) may have different
Laurent series in different annuli with the same center.
Of particular importance is the Laurent series (12) that converges in a neighborhood of z0 except
at z0 itself, say, for 0 < |𝑧 − 𝑧0 | < 𝑅 𝑓𝑜𝑟 𝑠𝑢𝑖𝑡𝑎𝑏𝑙𝑒 𝑅 > 0. The series (or finite sum) of the
negative powers in this Laurent series is called the principal part of f(z) at z0. The coefficient
1
𝑏1 of 𝑧−𝑧 in this series is called the residue of f(z) at z0 and is given by
0
1
2i C
𝒃𝟏 = 𝑹𝒆𝒔(𝒇(𝒛); 𝒛 = 𝒛𝟎 ) = f ( z * ) dz * (𝟏𝟒). Thus f ( z * )dz * 2i Re s( f ( z ), z z0 ).
C
𝒃𝟏 can be used for integration as shown in (14) because it can be found from
1 𝒅𝒎−𝟏
𝑹𝒆𝒔(𝒇(𝒛); 𝒛 = 𝒛𝟎 ) = 𝐥𝐢𝐦 ( 𝒎−𝟏 [(𝒛 − 𝒛𝟎 )𝒎 𝒇(𝒛)]), (𝟏𝟓)
(m 1)! 𝒛→𝒛𝟎 𝒅𝒛
provided f(z) has z0 at a pole of order m; by definition this means that principal part has
1
as its highest negative power. Thus for a simple pole (m-1)
(𝑧−𝑧0 )𝑚
𝑝(𝑧) 𝑝(𝑧0 )
𝑅𝑒𝑠(𝑓(𝑧); 𝑧 = 𝑧0 ) = lim (𝑧 − 𝑧0 )𝑓(𝑧); 𝑎𝑙𝑠𝑜, 𝑅𝑒𝑠 ( ; 𝑧 = 𝑧0 ) = ′ .
𝑧→𝑧0 𝑞(𝑧) 𝑞 (𝑧0 )
If the principal part is an infinite series, the singularity of f(z) at 𝑧0 is called an essential
singularity.
Residue integration may also be used to evaluate certain classes of complicated real integrals
1. (𝟑 − 𝟒𝐢)𝟐 3. √𝐢
𝟐−𝟑𝐢 𝟐𝛑𝐢
2. 4. 𝐞 𝟑
𝟏=𝟔𝐢
3. Find all values of the following functions(5-6)
4 3
5. √−24𝑖 6. √−1 , √8
Verify that the given functions (7-9 ) are Harmonic and find its Harmonic conjugates
7. 𝑢 = 𝑒 −𝑥 [(𝑥 2 − 𝑦 2 ) cos 𝑦 + 2𝑥𝑦 sin 𝑦)
8. u=𝑥 3 − 3𝑥𝑦 2 + 3𝑥 + 2
9. 𝑣 = 𝑦 sinh 2𝑥 cos 2𝑦 + 𝑥𝑐𝑜𝑠ℎ 2𝑥 sin 2𝑦
Find the value of the functions(10-11)
10. sin(2 − 𝑖) 11. cosh(𝜋 + 𝜋𝑖)
Solve the following equations(12-15)
12. 𝑒 𝑧 = 3𝑖 13. ln 𝑧 =
𝜋𝑖
2
18. Re z dz
C
from 0 to 3 27i along y x 3
Lnz 1
19.
C ( z 2i ) 2
dz counterclo ckwise around z 1
2
2 1
20. ( ( z 2i) z 4i )dz
C
clockwise around z 1 2.5
References
Contents
Module Introduction ....................................................................................................................... 1
UNIT I............................................................................................................................................. 3
1. ORDINARY DIFFERENTIAL EQUATIONS .......................................................................... 3
1.1 Introduction to Differential Equations .................................................................................. 4
1.1.1 Basic Concepts, terminology and Physical examples of Differential Equations ........... 4
1.1.2 Nature of Solutions of ODE: particular and general solutions ....................................... 8
1.2 First Order Ordinary Differential Equations ....................................................................... 12
1.2.1 Separable Differential Equations; Reduction to Separable Form ................................. 13
1.2.2 Homogeneous equations ............................................................................................... 19
1.2.3 Exact Differential Equations ........................................................................................ 27
1.2.3.1 Inexact equations; Integrating Factors ...................................................................... 32
1.2.4 Linear First Order Differential Equations; Reduction to Linear Form ......................... 37
1.2.5 Application Involving First Order Differential Equations ........................................... 44
1.3 Ordinary Linear Differential Equations of the Second order ......................................... 48
1.3. 1 Homogenous Equations .............................................................................................. 50
1.3.2 Non Homogeneous Linear Differential Equations ................................................. 71
1.4. Systems of Differential Equations.................................................................................. 88
1.5 Unit Summary ................................................................................................................ 90
UNIT II ......................................................................................................................................... 95
2. LAPLACE TRANSFORMS.................................................................................................. 95
2.1 The Laplace Transform; Inverse Transform and Linearity ................................................. 96
2.2 Laplace Transforms of Derivatives and Integrals ............................................................. 109
2.2.1 Transforms of Derivatives .......................................................................................... 109
2.2.2 Application To Differential Equations; Initial Value Problems ................................. 111
2.2.3 Laplace Transforms of Integrals . ............................................................................... 114
2.3 Unit Step Function and Second Shifting Theorem............................................................ 116
2.4 Differentiation and Integration of Transforms .................................................................. 123
2.4.1 Differentiation of Transforms ............................................................................... 123
2.4.2 Integration of Transforms ..................................................................................... 128
2.5. Convolution and Integral Equations ............................................................................. 130
2.5.1. Convolution ............................................................................................................. 130
2.5.2 Integral Equation .............................................................................................. 134
2.6 Unit Summary ................................................................................................................... 137
UNIT III ...................................................................................................................................... 140
3. FOURIER SERIES; FOURIER INTEGRALS AND .......................................................... 140
FOURIER TRANSFORMATIONS ........................................................................................... 140
3.1 Periodic and Orthogonal Funcions ............................................................................... 141
3.1.1. Periodic Functions and Trignometric Series ............................................................. 141
3.1.2. Orthogonal functions ................................................................................................. 144